You are on page 1of 244

केन्द्रीय विद्यालया संगठन ( मंबई संभाग )

KENDRIYA VIDYALAYA SANGATHAN (MUMBAI


REGION)

आध्यान सामाग्री / Study Material


शैक्षेवनक सत्र / Session – 2023-24
जीि विज्ञान / BIOLOGY
कक्षा XII / Class- XII
(Based on NCERT)

1
छत्रों के लिए सोंदेश

छात्ररं की अध्ययन सामग्री हमारे विषय वशक्षकरं द्वारा विवभन्न स्तररं पर विषय विशेषज्ञरं की
दे खरे ख में वकए गए एक घरे लू शैक्षविक अभ्यास का उत्पाद है , जर छात्ररं कर आपकी पढाई
के समेकन के वलए एक व्यापक वशक्षि सहायता उपकरि प्रदान करता है ।
इसमें पाठ अिधारिा मानवित्र, प्रिाह िार्ट , अध्यायरं का सवित्र प्रवतवनवधत्व, पाठरं का सार,
क्षमता आधाररत प्रश्न, एमसीक्यू प्रश्न, अवभकथन और तकट प्रश्न, केस आधाररत प्रश्न, बरर्ट के
वपछले िषों के प्रश्न, अभ्यास पत्र और सीबीएसई नमूना प्रश्न पत्र शावमल हैं ।
सामग्री कर निीनतम सीबीएसई पाठ्यक्रम और प्रश्न पत्र वर्जाइन कर ध्यान में रखते हुए
विकवसत वकया गया है । मझे उम्मीद है वक यह सामग्री त्वररत पनरीक्षि के वलए एक अच्छा
उपकरि सावबत हरगी और छात्ररं के आत्मविश्वास के स्तर कर बढाकर उन्हें बेहतर प्रदशटन
करने में मदद करे गी।
यरजनाबद्ध अध्ययन, कडी मेहनत, अच्छा समय प्रबंधन और ईमानदारी छात्ररं कर बडी
सफलता तक पहुुँ िने में मदद करे गी।

श्रीमती सरना सेठ


उपायक्त
केन्द्रीय विद्यालया संगठन
मंबई संभाग

2
लिय लिद्यालथियरों के लिए एक शब्द

मझे छात्ररं के वलए जीि विज्ञान में अध्ययन सामग्री जारी करते हुए खशी हर रही है , वजसे हमारे प्रवतवित क्षेत्र
के विद्वान वशक्षकरं के समूह द्वारा सािधानीपू िटक तैयार वकया गया है । उन्हरंने आप सभी कर अत्याधवनक
जानकारी प्रदान करने के वलए कक्षा संिालन के अपने अनभिरं कर संवक्षप्त नरर्् स, संकल्पना मानवित्र,
प्रश्नरं के संग्रह, सूिना तावलकाओं और आदशट नमूना पेपर के सेर् के रूप में रखा है । एक अच्छा संसाधन
सबसे अच्छा साथी हरता है । यह वनवित रूप से आपकर प्रे ररत करे गा और आपकर अपना सिटश्रेि प्रदशटन
करने के वलए मागटदशटन करे गा।
मझे लगता है वक अध्ययन सामग्री छात्ररं और वशक्षकरं के वलए समान रूप से उपयरगी सावबत हरगी। इससे
वशक्षकरं के वलए तैयारी का समय कम हर जाएगा और उन्हें समय पर मूल्ां कन करने में सहायता वमलेगी।

आगामी परीक्षा में सफलता और उज्ज्वल कररयर के वलए शभकामनाएं ।

श्रीमती प्रवमला पल
प्रािायट दे हु ररर् क्रमां क 1
पिे

3
STUDY MATERIAL TEAM

ADVISOR:
Mrs. Sona Seth, Deputy Commissioner, KVS Mumbai Region.

COORDINATOR :
Mrs. Pramila Pal, Principal, KV Dehu Road No.1, Pune.
REVIEW TEAM
Mr. Ashok Viswakarma, PGT(Biology) KV Army Area Pune.
Dr. Manoj Baliram Jagdale, PGT (Biology) KV AFS Devlali.

CONTENT UPDATION TEAM


Mrs. Sandhya Mehrotra, PGT(Biology) KV DIAT Pune.
Mrs. Archana Kumari, PGT(Biology) KV ONGC Panvel.
Mrs. Deepa Parihar, PGT(Biology) KV1 Colaba.
Mr. Nitin Dongre, PGT (Biology) KV Pulgaon Camp
Mrs. Kanak Lata, PGT (Biology) KV1, Majri
Mrs. Sheeba K M, PGT (Biology) KV2 Vasco.

4
INDEX

SN CONTENT PAGE NO

1 Weightage of marks 5

2 CBSE Syllabus 6

3 Chapter 2: Sexual Reproduction in Flowering Plants 8

4 Chapter 3: Human Reproduction 12

5 Chapter 4: Reproductive Health 21

6 Chapter-5: Principles of Inheritance and Variation 27

7 Chapter-6: Molecular Basis of Inheritance 35

8 Chapter-7: Evolution 44

9 Chapter-8: Human Health and Disease 50

10 Chapter-10: Microbes in Human Welfare 59

11 Chapter-11: Biotechnology: Principles and Processes 65

12 Chapter-12: Biotechnology and its Applications 71

13 Chapter-13: Organisms and Populations 76

14 Chapter-14: Ecosystem 81

15 Chapter-15: Biodiversity and Conservation 86

5
CLASS XII (2023-24) (THEORY)
Time: 03 Hours Max. Marks: 70
Unit Title Marks
Unit-VI Reproduction 16

Chapter 2: Sexual Reproduction in Flowering Plants


Chapter-3: Human Reproduction
Chapter 4: Reproductive Health
Unit-VII Genetics and Evolution 20

Chapter-5: Principles of Inheritance and Variation


Chapter-6: Molecular Basis of Inheritance
Chapter-7: Evolution
Unit-VIII Biology in Human Welfare 12

Chapter-8: Human Health and Disease


Chapter-10: Microbes in Human Welfare
Unit-IX Biotechnology 12

Chapter-11: Biotechnology: Principles and Processes


Chapter-12: Biotechnology and its Applications
Unit-X Ecology 10

Chapter-13: Organisms and Populations


Chapter-14: Ecosystem
Chapter-15: Biodiversity and Conservation
Chapter-16: Environmental Issues

Total marks 70

6
Detail syllabus (https://cbseacademic.nic.in/curriculum_2024.html )
Unit-VI Reproduction

Chapter-2: Sexual Reproduction in Flowering Plants Flower

Flower structure; development of male and female gametophytes; pollination - types, agencies and examples; out
breeding devices; pollen-pistil interaction; double fertilization; post-fertilization events - development of endosperm
and embryo, development of seed and formation of fruit; special modes- apomixis, parthenocarpy, polyembryony;
Significance of seed dispersal and fruit formation

Chapter-3: Human Reproduction


Male and female reproductive systems; microscopic anatomy of testis and ovary; gametogenesis -spermatogenesis and
oogenesis; menstrual cycle; fertilization, embryo development upto blastocyst formation, implantation; pregnancy and
placenta formation (elementary idea); parturition (elementary idea); lactation (elementary idea)

Chapter-4: Reproductive Health

Need for reproductive health and prevention of Sexually Transmitted Diseases (STDs); birth control - need and
methods, contraception and medical termination of pregnancy (MTP); amniocentesis; infertility and assisted
reproductive technologies - IVF, ZIFT, GIFT (Elementary idea for general awareness).

Unit-VII Genetics and Evolution

Chapter-5: Principles of Inheritance and Variation


Heredity and variation: Mendelian inheritance; deviations from Mendelism – incomplete dominance, co-dominance,
multiple alleles and inheritance of blood groups, pleiotropy; elementary idea of polygenic inheritance; chromosome
theory of inheritance; chromosomes and genes; Sex determination - in humans, birds and honey bee; linkage and
crossing over; sex linked inheritance - haemophilia, colour blindness; Mendelian disorders in humans - thalassemia;
chromosomal disorders in humans; Down's syndrome, Turner's and Klinefelter's syndromes.

Chapter-6: Molecular Basis of Inheritance


Search for genetic material and DNA as genetic material; Structure of DNA and RNA; DNA packaging; DNA replication;
Central Dogma; transcription, genetic code, translation; gene expression and regulation - lac operon; Genome, Human
and rice genome projects; DNA fingerprinting.

Chapter-7: Evolution

Origin of life; biological evolution and evidence for biological evolution (paleontology, comparative anatomy,
embryology and molecular evidence); Darwin's contribution, modern synthetic theory of evolution; mechanism of
evolution - variation (mutation and recombination) and natural selection with examples, types of natural selection;
Gene flow and genetic drift; Hardy - Weinberg's principle; adaptive radiation; human evolution.

Unit-VIII Biology and Human Welfare

Chapter-8 Human Health and Diseases


Pathogens; parasites causing human diseases (malaria, dengue, chikungunya, filariasis, ascariasis, typhoid, pneumonia,
common cold, amoebiasis, ring worm) and their control; Basic concepts of immunology - vaccines; cancer, HIV and
AIDS; Adolescence - drug and alcohol abuse.

Chapter-10: Microbes in Human Welfare

Microbes in food processing, industrial production, sewage treatment, energy generation and microbes as bio-control
agents and bio-fertilizers. Antibiotics; production and judicious use.

Unit-IX Biotechnology and its Applications


7
Chapter-11: Biotechnology

Principles and Processes Genetic Engineering (Recombinant DNA Technology).

Chapter-12: Biotechnology and its Applications

Application of biotechnology in health and agriculture: Human insulin and vaccine production, stem cell technology,
gene therapy; genetically modified organisms - Bt crops; transgenic animals; biosafety issues, biopiracy and patents.

Unit-X Ecology and Environment

Chapter-13: Organisms and Populations

Population interactions - mutualism, competition, predation, parasitism; population attributes - growth, birth rate and
death rate, age distribution. (Topics excluded: Organism and its Environment, Major Abiotic Factors, Responses to
Abiotic Factors, Adaptations)

Chapter-14: Ecosystem

Ecosystems: Patterns, components; productivity and decomposition; energy flow; pyramids of number, biomass,
energy (Topics excluded: Ecological Succession and Nutrient Cycles).

Chapter-15: Biodiversity and Its Conservation

Biodiversity-Concept, patterns, importance; loss of biodiversity; biodiversity conservation; hotspots, endangered


organisms, extinction, Red Data Book, Sacred Groves, biosphere reserves, national parks, wildlife, sanctuaries and
Ramsar sites.

8
CHAPTER -1

SEXUAL REPRODUCTION IN FLOWERING PLANTS

--THE WHOLE CHAPTER AT YOUR FINGER TIP

Key words
Flower Reproductive part of a plant

Sporogenous tissue Compactly arranged homogenous cells in the center of microsporangia,


undergo meiosis (Microsporogenesis) to form tetrads of microspores

Germ pore Apertures in the pollen grain, facilitate the exchange of gases and water, help in
emerging of pollen tube.

Autogamy When pollination occurs between the same flower of the same plants

Egg apparatus Consists of synergids and filiform apparatus, help in the entry of pollen tube
into the embryo sac.

Synergid Present in the embryo sac, two in number

Filiform apparatus Present in synergids, guider pollen tube entry into the embryo sac

Geitnogamy Transfer of pollen grains from the anther to the stigma of another flower of
the same plant

Xenogamy Transfer of pollen grains from the anther to the stigma of a different plant

Triple fusion Male gamete fuses with two polar nuclei to form the triploid endosperm

Embryogeny Formation of embryo

Cotyledons the embryonic leaf in seed-bearing plants

Scutellum Cotyledons of monocotyledon plants

Dormancy State of inactiveness

Parthenocarpy Development of fruit without fertilization ex- banana, orange

Polyembryony Occurrence of more than one embryo in seed Ex- lemon

K.V.S.MUMBAI REGION
Pre Fertilization event Fertilization Post Fertilization event

FORMATION OF POLLEN GRAIN

K.V.S.MUMBAI REGION
• It involves the formation of male and female gametes (gametogenesis) and gamete transfer.
• Gamete transfer- Bringing together male & female gamete
Male reproductive structure- Stamen
• Stamens consists of – Filament, Stalk and Anther
• Anther- it is Bilobed, Dithecous and each lobe has 2 pollen sacs
(microsporangia)
• Microsporangia have four layers Epidermis, Endothecium, Middle layers and
Tapetum (provides nourishment to the pollen grains).
Microsporogenesis (formation of microspore)
• Microsporogenesis is the process in which sporogenous tissues in microsporangia form meiocytes (2n). The
latter divide meiotically and form microspore tetrad (n).
• Each microspore matures into Pollen grains (male gametophyte)
Pollen grains
Inline (inner layer): it is composed of cellulose & pectin
Exine (outer layer): it is composed of sporopollenin.
Female reproductive structure (Carpel)
• Carpel has stigma, style and ovary. The ovary encloses ovules (megasporangium) in it.
• Ovule consists of Funicle, Hilum, Integuments, Micropyle, Chalaza, Nucellus and embryo sac. The embryo sac is 7
celled and 8-nucleate structure-
Female reproductive structure (Carpel)
• Carpel has stigma, style and ovary. The ovary encloses ovules (megasporangium) in it.
• Ovule consists of Funicle, Hilum, Integuments, Micropyle, Chalaza, Nucellus and embryo sac. The embryo sac is 7
celled and 8-nucleate structure-
• 7 cell- 3 antipodal cell + 2 synergids + egg + central cell
• 8 nucleate- 3 antipodal cell + 2 synergids + egg + 2 polar nuclei

Megasporogenesis (formation of megaspore)


• Inside ovule Megaspore Mother Cell (MMC) is formed. MMC undergoes meiosis and 4 haploid megaspores are
formed (Megaspore tetrad).
• Only one of the megaspores develops into a female gametophyte (Monosporic development).

K.V.S.MUMBAI REGION
POLLINATIONThe process of the transfer of pollen grains from the anther to the stigma of a flower
is called pollination.
Types of Pollination

Type Description
Autogamy ● Pollen grains are transferred from the anther to the stigma of the same flower.
● Some plants produce two types of flowers−chasmogamous (open flowers) and
cleistogamous (closed flowers).
● In case of chasmogamous flowers, the anthers and stigma are brought together by
growth, bending or folding.
● In case of cleistogamous flowers, the anther and stigma lie close to each other.
Geitonogamy ● Pollen grains are transferred from the anther of one flower to the stigma of other
flower of the same plant.
Xenogamy ● Pollen grains are transferred from the anther of one flower to the stigma of other
flower of a different plant.

PRE-FERTILISATION EVENT:

K.V.S.MUMBAI REGION
K.V.S.MUMBAI REGION
K.V.S.MUMBAI REGION
Adaptation in pollination
Wind pollen grains are light, non-sticky, stamen well-exposed, large often-feathery stigma, often
monolocular ovule, numerous flowers packed into an inflorescence.
Water Pollen grains are long, ribbon like, and protected from wetting by a mucilaginous covering.
Insect Flowers are large, colourful, fragrant and rich in nectar. When the flowers are small, several flowers
are clustered into an inflorescence to make them conspicuous.

FERTILIZATION (Fusion of male and female gametes)


• Fusion of male gamete (in pollen) and female gamete (egg) in the embryosac.
• In some plants there are two male gametes. One fuses with an egg (syngamy) and the other fuses with two
polar nuclei (triple fusion). This is known as double fertilization.
POST FERTILIZATION EVENT

PEC Zygote Ovule Ovary

Endosperm Embryo Seed Fruit

Endospermic / Albuminous seeds Seeds that have endosperm wheat, rice


Non-endospermic / non-albuminous seeds Seeds that lack endosperm at maturity pea, bean
EMBRYO
• Zygote undergoes mitosis division to form 2- cell stage, 8-cell stage, Globular, Heart shaped stage, Torpedo
stage and finally the mature embryo.
Structure of a dicotyledonous embryo
Embryonal axis The main axis of the embryo which
divides it into different regions
Cotyledons Food storage and nourishment

Epicotyl and Part of the embryonal axis above and


hypocotyl below the cotyledons respectively
Plumule a radicle Stem tip and root tip respectively
Root cap Covering of root tip

Structure of monocot embryo

Coleorrhiza Sheath enclosing radicle & root cap


Coleoptile Sheath enclosing plumule

SEEDS:

Non-endospermicseeds
Endospermicseeds or
(Albuminousseeds) (Non-albuminousseeds)
Ex-Maize, Wheat Ex- gram, Beans
OUTBREEDING DEVICES

OUT BREEDING DEVICES ( to avoid inbreeding depression some plants develop devices to discourage self-pollination).
Pollen release and stigma receptivity are not synchronized.
Stigma and another position in the same flower are such in a way that they can’t come in contact.
Self - incompatibility
Production of the unisexual flower.

ARTIFICIAL HYBRIDIZATION

Emasculation- removal of anther before attaining maturity


Bagging- bagging of emasculated flower with a paper bag
Dusting- removal of bag and dusting with desired pollen on the stigma
Advantage- to prevent self-pollination and inbreeding depression
POLLEN PISTIL INTERACTION
Compatibility- Pollen of the same species as the stigma
Self-incompatible - Pollen of the wrong type, either from other species or from the same plant
The pistil can recognize Compatible or Incompatible pollen.
Wrong pollen is rejected and stigma and style
Monocot
SEED

Seed coat Radicle (root) Plumule (shoot) Cotyledon

Dicot
Importance of seeds
Protect embryo, provide nourishment to young seedlings, undergo dormancy to overcome the stressed situation
APOMIXIS
• Formation of seed without fertilization is known as apomixis.
• Diploid nucellar cells divide and enter inside the embryo sac and later develop into embryos.
• Sometimes, the egg cell is diploid due to the failure of meiosis. Such diploid egg cell later directly develops
into an embryo.
Polyembryony- the occurrence of many embryos in value. E.g. Citrus, mango, etc.

Important questions:
ASSERTION AND REASONING:
Questions

In the following questions a statement of assertion followed by a statement of reason is given. Choose the
correct answer out of the following choices.
(a) Both assertion and reason are true, and reason is the correct explanation of the assertion.
(b) Both assertion and reason are true, but reason is not correct explanation of the assertion.
(c) Assertion is true but reason is False.
(d) Bath assertion and reason are false.
1. Assertion (A) Tapetum is the innermost wall layer It performs the function of providing nourishment to the
developing pollen grains.
Reason (R): The cells of tapetum have dense cytoplasm and usually have more than one nucleus
Answer: (a)
2. Assertion (A) The pollen mother cell undergoes meiosis form microspores. to and this process is called as
microsporogenesis.
Reason (R): Some cells of the spororogenous tissue are potential pollen mother cells (PMC)
Answer: (c)
3. Assertion (A) Pollen grains are well preserved and can stay viable for long periods of time
Reason (R): The hard outer layer of pollen grains called exine is made up sporopollenin
Answer: (a)
4. Assertion (A): Pollen grains are shed at 3-celled stage in some angiosperms
Reason (R): In some species, the vegetative cell of pollen grain divides mitotically to form two male gametes
Answer: (a)
5. Assertion: When the two genes in a dihybrid cross are situated on the same chromosome, the proportion of
parental gene combinations is much higher than non-parental type.
Reason: Higher parental gene combinations can be attributed to crossing over between two genes.
Answer: (c)

MCQs
1. Flowers with both androecium and gynoecium are called
1. Bisexual flowers 2. Anther 3. Stamens 4. Unisexual flowers
2. The transfer of pollen from the anther to stigma is called
A. Pollination B. Fertilization C. Adoption D. Diffusion
3. The fusion of female reproductive nucleus with the male reproductive nucleus is known as
A. Adoption B. Excretion C.Fertilization D.Regeneration
4. The two nuclei at the end of the pollen tube are called
A. Tube nucleus and a generative nucleus B. sperm and ovum
1. Generative nucleus and stigma D. Tube nucleus and sperm
5. Generative nucleus divides forming
A. 2 male nuclei B. 3 male nuclei C. 2 female nuclei D. 3 female nuclei
6. Embryo sac is located inside the
A. Stigma B. Ovule C. Micropyle D. Style
7. One nucleus of the pollen tube and secondary nucleus of the ovum grow into
A. Stigma B. Endosperm C. Anther D. Stamen
8. The stalk of Datura flower at its base is known as
A. Pedicel B. Corolla C. Sepals D. Thalamus
9. The male reproductive parts of a flower, the stamens, are collectively known as
A. Androecium B. Filament C. Anther D. Gynoecium
10. The other name for gynoecium is
A. Pistil B. Stigma C. Androecium D. Style
11. Functional megaspore in a flowering plant develops into
A. Endosperm B. Ovule C. Embryo-sac D. Embryo
12. Which of the following is similar to autogamy, but requires pollinators?
A. Geitonogamy B. Cleistogamy C. Apogamy D. Xenogamy
13. What is the function of the filiform apparatus?
A. Guide the entry of pollen tube B. Recognize the suitable pollen at the stigma
1. Produce nectar D. Stimulate division of the generative cell
14. A mass of nutritive material outside the embryo sac is called _____
A. Protoplasm B. Pericarp C. Ectoderm D. Perisperm

15. Which of the following statements is correct?


A. Sporogenous tissue is haploid B. The hard outer layer of pollen is called intine
1. Tapetum nourishes the developing pollen D. Microspores are produced by endothecium
16. Which of the following fruit is produced by parthenocarpy?
A. Brinjal B. Apple C. Banana D. Jackfruit
17. The process of formation of seeds without fertilization in flowering plants is known as
A. Budding B. Apomixis C. Sporulation D. Somatic hybridization
18. Functional megaspore in an angiosperm develops into
A. Endosperm B. Embryo C. Embryo-sac D. Ovule
19. Rewards and attractants are required for
A. Entomophily B. Cleistogamy C. Anemophily D. Hydrophily
20. A dioecious flowering plant prevents
A. Geitonogamy and xenogamy B. Autogamy and xenogamy
C. Autogamy and geitonogamy D. Cleistogamy and xenogamy

Answer Key
1- 1 2- 1 3- 3 4- 1 5- 1

6- 2 7- 2 8- 1 9- 1 10- 1
11- 3 12- 1 13- 1 14- 4 15- 3

16- 3 17- 2 18- 3 19- 1 20- 3

1. In a young anther, a group of compactly arranged homogenous cells was observed


in the center of each microsporangium. What is the name given to these cells?
Ans: Sporogenous tissue
2. Give the scientific name of a plant that came to India as acontaminantwith imported
wheat and causes pollen allergy.
Ans: Parthenium
4. Why are pollen grains produced in enormous quantities in Maize?
Ans: To make sure pollination is happening as the Maize is pollinated by wind.
5. In the same species of Asteraceae and grasses, seeds are formed without the fusion
of gametes. Mention the scientific term for such forms of reproduction.
Ans: Apomixis
6. Arrange the following in the correct developmental sequence: Male gamete, Potential
pollen mother cell, sporogenous tissue, Pollen grains, Microspore tetrad.
Ans: Sporogenous tissue, Potential pollen mother cell, microspore tetrad, Pollen grain male.
7. If the diploid number of chromosomes in an angiospermic plant is 16. Mention the
number of chromosomes in the endosperm and antipodal cell.
Ans: Chromosomes are in the endosperm and 16 chromosomes are present in the antipodal
cell.
8. What kind of structures are formed at the end of microsporogenesis and
megasporogenesis?
Ans: Microsporogenesis comes about into the arrangement of four haploid dust grains
orchestrated usually in the tetrahedral tetrad whereas Megasporogenesis produces four
megaspores orchestrated in a linear tetrad.
9. What is funiculus?
Ans: The stalk of the ovule is known as the funiculus.
10. Define parthenocarpy.
Ans: Parthenocarpy is the production and development of seedless fruit.
11. What is microsporogenesis?
Ans: Microsporogenesis is the process which leads to the production of microspores from
pollen mother cells through meiosis is referred to as microsporogenesis.
12. Why is emasculation done in the process of hybridization?
Ans: Emasculation is when the stamens are separated before artificial hybridization to secure
that no unwanted pollen lands on the stigma and the flower can be pollinated with the wanted
pollen grains.
13. What do you understand about double fertilization?
Ans: Fertilization in female gametophytes occurs at two locations and that are the egg cell &
the generative cell; the vegetative cell and polar nuclei. This is known as double fertilization.
14. What is sporopollenin?
Ans: The exine of the pollen grain is made of a quite resistant organic chemical known as
sporopollenin.
15. Name one plant each where pollination occurs with the help of
a) Water.
Ans: Water pollinated: Vallisneria and Hydrilla.
Ans: Bat pollinated: Anthocephalus and Bauhinia megalandra.
16. Why do most zygotes develop after a certain amount of embryo is formed?
Ans: As an adaptation to give guaranteed sustenance to the developing embryo, the zygote
divides only when a particular amount of endosperm is created.
17. What is polyembryony?
Ans: Polyembryony is the process of formation of more than one embryo during the formation
of a seed.
18. Name the type of cross pollination in Vallisneria& Bougainvillea.
Ans: (i) vallisneria - Hydrophily (ii) Bougainvillea - Entomophily
19. How many haploid nuclei and haploid cells are present in the female gametophyte
of angiosperm?
Ans: 8-haploid nuclei and 7-haploid cells.
20. Mention the scientific term for the type of pollination which ensures Genetic
Recombination.
Ans: Allogamy or Xenogamy
21. Which are the nuclei that fuse to form an endosperm?
Ans: The second male gamete combines with a secondary nucleus, which is produced by the
union of two polar nuclei to form a triploid primary endosperm.
22. Give an example of a Bat-Pollinated flower.
Ans: Adansoniadigitata.
23. Why are pollen grains produced in enormous quantities in maize?
Ans: Pollen grains are produced in enormous quantities in maize because, in maize, pollen
grains are transferred through the air. A large number of pollen grains are produced but only a
few of them are air-borne, Pollen grains are entangled by protruding stigma.
24. Name the part of an angiosperm flower in which development of male & female
gametophyte takes place.
Ans: The advancement of male gametophyte takes place in microspores in pollen grains and
the development of female gametophyte happens in the megaspore of the ovule.
25. Why is an apple called a false fruit? Which part of the plant forms the fruit?
Ans: Apple is known as a false fruit as it develops from the ovary together with the accessory
floral plants such as the Thalamus
26. Name the part of the plant producing seed & fruit after fertilization.
Ans: After the fertilization, the ovary develops into fruit & the ovule develops into a seed.

2 Marks Questions
1. In Angiospermic plants before the formation of microspore sporogenous tissue
undergo cell division.
Ans: The polyembryony is known as the presence of more than one embryo in a seed,
Example- Sometimes more than one embryo is created inside an embryo sac by splitting of
egg or by cleavage, endosperm, synergid, or antipodal
(a) Name the type of cell division.
Ans: Meiosis division
(b) What would be the ploidy of the cells of tetrad?
Ans: Haploid
2. Outer envelope of pollen grain made of a highly resistant substance. What is that
substance? At which particular point the substance is not present?
Ans: Sporopollenin; at germpore sporopollenin is not present.
3. Fruits generally develop from the ovary, but in few species, the thalamus contributes
to fruit formation.
(a) Name the two categories of fruits
Ans: Two categories of fruits are (i) True fruits (ii) False fruit
b) Give one example of each fruit
Ans: (i) True fruits e.g., Mango (ii) False fruit e.g., Apple
4. Among the animals, insects, particularly bees are the dominant pollinating agents.
List any four characteristic features of the insect-pollinated flower.
Ans: Four characteristic features of the insect-pollinated flower are:-
i. Flowers are large.
ii. Colorful petals are there in flowers.
iii. Presence of fragrance in flowers.
iv. They are rich in nectar.
5. Differentiate between geitonogamy and xenogamy.
Ans: Below given are the differences between geitonogamy and xenogamy:-

Geitonogamy Xenogamy

Transfer of pollen grains from the anther to the Transfer of Pollen grains from the anther
stigma of another flower of the same plant. to the stigma of different plants.
Does not provide an opportunity for genetic Provide an opportunity for gametic
recombination. recombination.
6. In the given figure of a dicot embryo, label the parts and give their function.

Ans:
A =Plumule To form shoot system
B= Cotyledons Storage of food
7. Name the parts A, B, C, and D of the anatropous ovule (Figure 2) given above.
Ans: A = Micropyle, B = Outer integument, C = Nucellus, D = Embryo sac
8. Given below is an incomplete flow chart showing the formation of gamete in
angiosperm plants. Observe the flow chart carefully and fill in the blank A, B, C, and D.

Ans: In the given flow chart


A represents Ovule/Megasporangium B represents Megaspore mother cell
C represents Tapetum D represents Pollen grains

9. Name the blank spaces a, b, c, and d in the table given below: Item What it
represents in the plant.
Part What it Represents in the Plant
(i) Pericarp a
(ii) b Cotyledon in seeds of grass family.

(iii) Embryonal axis C

(iv) d Remains of nucellus in a seed


Ans: In the given table a, b, c, and d represents the following:
(i) Pericarp
Ans: a- wall of fruit
(ii) Cotyledon in seeds of the grass family
Ans: b-scutellum
(iii) Embryonal axis
Ans: c-shoot and root tip
(iv) Remains of nucellus in a seed.
Ans: d-perisperm
10. Even though each pollen grain has two male gametes. Why are at least 10 pollen
grains and not 5 pollen grains required to fertilize 10 ovules present in a particular
carpel?
Ans: This is because only one male gamete is involved in syngamy. ie fusion of male gamete
with the egg cell.
11. Describe the structure of a microsporangium with a neatly labeled diagram.
Ans: The structure of the microsporangium is as follows:
-It is almost circular and it also has four wall layers.
-The outer three layers are epidermis, endothecium and middle layers are protective in nature
and help in the dehiscence of anther so that it can release pollen grains.
-The inner tapetum provides food and nourishes the developing embryo. Sporogenous tissue
remains at the central position.

12. Why can pollen grains remain well preserved as fossils?


Ans: Pollen grains are well preserved as fossils because the exine of the pollen is composed
of a chemical, sporopollenin which can withstand high temperature, strong acids and alkalis,
and strong enzymes
13. How are the cells arranged in an embryo sac?
Ans: An embryo sac is a 7 celled and 8 nucleated structure. A group of three cells is present
at the micropylar end which are two synergids and one egg cell. The chalazal end has three
cells, which are called antipodals. There is a central cell with 2 polar nuclei.
14. Why are cleistogamous flowers invariably autogamous?
Ans: In a cleistogamous flower, the flower never opens and when the anther dehiscence in
the bud the pollen grains rest on the stigma of the same flower and therefore it is totally
autogamous.
15. State any one advantage and disadvantage of pollen grains to humans.
Ans: Advantage: Pollen grains are rich in nutrients and therefore in lots of western countries,
they( pollen tablets are used as food supplements.
Disadvantage: Pollens of many species of plants can cause severe bronchial afflictions and
allergies which will usually lead to severe diseases like chronicle respiratory disorder.
16. State the characteristics of insect-pollinated flowers.
Ans: An entomophilous flower has the following characteristics:
• Petals and sepals are well developed with attractive colors to invite insects.
• Flowers are normally bigger in size with a strong odor.
17. Differentiate between chasmogamous and cleistogamous flowers
Ans:
Chasmogamous Cleistogamous
The flowers are conspicuous. The anthers The flowers are small and inconspicuous. The
and the stigmas are exposed. anthers and stigmas are never exposed.
Both self and cross-pollination can occur. Only self-pollination is possible
18. Which type of pollination ensures the arrival of genetically different pollen grains 9
to stigma?
Ans: In xenogamy pollens from another plant of the same species pollinate the stigma and
thereby ensure the coming of genetically different types of pollen grains onto the stigma.
19. What relationship exists between a species of moth and Yucca plant?
Ans: There exists a relationship between the Yucca plant and the moth. The moth deposits its
egg in the locule of the ovary and in return moth pollinates the flower of the yucca plant. As
the seeds begin to mature, the larvae hatch from the eggs.
20. Differentiate between Geitonogamy & Allogamy.
Ans: Below given are the Differences between Geitonogamy &Allogamy:-
Geitonogamy Allogamy

It takes place between anther & pistil of It takes place between two flowers of two
different flowers of the same plant. different plants of the same species.
Bisexual flowers are essential for
Unisexual flowers are essential for Allogamy.
geitonogamy.
Progenies do not show variation & are Progenies show variations & are genetically
genetically pure. impure.
21. Draw a diagram of L.S. of an anatropous ovule of an Angiosperm & label the
following parts:-
(i) Nucellus (ii) Integument (iii) Antipodal cells (iv) Secondary Nucleus.
Ans:
22. Why is the process of fertilization in flowering plants referred to as double
fertilization?
Ans: In flowering plants, the first male nuclei fuse with the egg to produce a diploid zygote,
and the second male nuclei fuse with the Secondary nucleus to produce the primary
endosperm nucleus, resulting in DOUBLE FERTILISATION.
23. What are cleistogamous flowers? Can cross-pollination occur in cleistogamous
flowers? Give a reason?
Ans: In some Angiosperm plants eg. Commelina, Oxalis, etc, flowers are bisexual and they
never open. This condition is known as cleistogamy and flowers from this condition are called
cleistogamous. The cleistogamous flowers are self-pollinated and to ensure this they never
open. Hence, cross-pollination is not possible.
24. Draw a labeled diagram of mature embryo sac & label the following i) Egg cell ii)
Antipodal cells iii) Synergids iv) Polar nuclei

25. Mention two strategies evolved lay flowers to prevent self-pollination


Ans: Two strategies evolved lay flowers to prevent self-pollination
(i) Dichogamy - In this process, two reproductive organs of a bisexual flower mature at
different- different times.
(ii) Self sterility – The Pollen grains couldn’t germinate on the stigma of the same flower or
on the flower of the same plant.
26. What is apomixis? What is its importance?
Ans: The development of the reproductive propagules without meiosis and syngamy is known
as apomixis. It is also known as asexual reproduction. apomixis is a method of reproduction
that produces new individuals from the vegetative part of the plant body.
27. Draw a well-labeled diagram of a longitudinal section of a pistil showing pollen
germination?

28. List the advantages of pollination to angiospermic plants?


Ans: Pollination leads to fertilization & production of seeds & fruits which are necessary for
continuity of life.
i) It is critical for the development of new plant varieties.
ii) It is necessary for hybrid seed production.
iii) It aids in plant genetic recombination.
3 Marks Questions
1. Continued self-pollination leads to inbreeding depression. List three devices, which
flowering plants have developed to discourage self-pollination?
Ans:
(a) Release of pollen and stigma receptivity is not synchronized in some species
(b) Anther and stigma are at different positions/heights in some plants
(c) Self-incompatibility is a genetic mechanism.
2. What will be the fate of the following structures in the angiospermic plant? Ovary
wall, Ovule, zygote, outer integument Inner integument, and primary endosperm
nucleus.
Ans: Ovary wall = Pericarp ; Ovule = Seed, Zygote - Embryo; Outer integument = Testa;
Inner integument = Tegmen; Primary endosperm nucleus = Endosperm.
3. Differentiate between microsporogenesis and megasporogenesis. What type of cell
division occurs during these events. Name the structure formed at the end of these two
events.
Ans: Microsporogenesis is the process of production of microspore from a Pollen mother cell.
Megasporogenesis is the process of production of megaspores from MMC. Meiotic division in
both Microsporogenesis causes the formation of pollen grains while megasporogenesis
causes the formation of megaspores.
4. Differentiate between microsporogenesis and megasporogenesis.
Ans: Below given are the differences between microsporogenesis and megasporogenesis:
Microsporogenesis Megasporogenesis
From the diploid microspore mother cell, haploid Megaspores are produced from the
microspores or pollen grains are formed. diploid megaspore mother cell.
The pollen grains are arranged in a tetrahedral The megaspores are arranged in a linear
tetrad. tetrad.
All the microspores are functional. Only one megaspore is functional.
Others degenerate.
5. Explain the stages involved in the maturation of a microspore into a pollen grain.
Ans: The microspore comprises a dense cytoplasm and a bigger nucleus in the center. As the
microspore develops the nucleus is shifted towards the side due to the production of vacuoles
in the upper end of the cytoplasm. The mitotic division causes the nucleus to produce two
nuclei which separate out into 2 cells; the lower generative cell and the upper bigger
vegetative cell. A mature pollen grain normally has 2 cells.
6. What is triple fusion? Where does it occur?
Ans: The nucleus of a vegetative cell of pollen grain combined with 2 polar nuclei of the
central cell of the female gametophyte joins to produce the primary endosperm. This fusion is
called vegetative fusion or triple fusion because it involves 3 nuclei. It happens in the central
cell of the egg apparatus.
7. Explain the structure of an anatropous ovule with a neat labeled diagram?
Ans: An anatropous ovule consists of: a stalk called a funicle attached to the placenta. Helium
is the name for the connection between the funicle and the ovule. One or more integuments
might enclose the ovule, with an aperture at the tip. The opening is called the micropyle. The
opposite end of the micropyle is referred to as the chalazal end, the basal part of the ovule.
Within the integuments that ordinarily contain a single embryo sac is a mass of cells known as
nucellus.

8. Describe the structure of a pollen grain.


Ans: The pollen grain is normally spherical with 2 wall layers.
• The outer layer is exine composed of a highly resistant organic substance called
sporopollenin which is absent at the aperture region called germ pore.
• The inner layer is called the intine which is made up of pectin and cellulose.
• A mature pollen grain has a vegetative cell and a generative cell.
9. Enlist the advantages offered by seeds to angiosperms.
Ans: The importance of seed formation:
• Seed formation is linked with fertilization and pollination that are independent of water
and therefore it is surely a more dependable process.
• Seed formation provides nutrition and protection to the budding embryo.
• Seeds are used for the multiplication of plants. Seeds are capable of perennation; they
can withstand different climates.
10. Give any three advantages of sexual incompatibility.
Ans: Advantages of sexual incompatibility:
• Sexual incompatibility inhibits self-pollination.
• Sexual incompatibility has made plants outbreeders and thereby maintains the vitality
and vigor of the plant’s rarace.
• In sexual incompatibility, variations occur because outbreeding provides adaptability for
the changes in the environment.
11. List any three differences between wind-pollinated flowers and insect-pollinated
flowers.
Ans: Below given are the differences between wind-pollinated flowers and insect-pollinated
flowers:-
Wind-Pollinated Flower Insect-Pollinated Flower
Flowers are small and colorless. Flowers are brightly colored
Flowers do not have scent or nectar Flowers possess nectar glands.
Pollen grains are dry and unwettable. Pollen grains are sticky or Spiny.
Stigma is a large well- exposed hairy and
Stigma is short and is present within the
branched
flower.

12. Trace the development of microsporocytes into mature pollen grains.


Ans: When the anther is at an early stage of development, the microsporangium has closely
arranged homogenous cells which form the Sporogenous tissues.
1. Each cell of sporogenous tissue will become a Pollen mother cell (PMC) and also forms
microspore tetrad or Pollen grains.
2. But Some of them lose this Potential and later get differentiated into a pollen or microspore
mother cell (MMC)
3. Every microspore mother cell (MMC) undergoes meiosis and produces a bunch of four
haploid cells, which is called microspore tetrad.
4. When the anther gets matured, the microspores get dissociated from the tetrad and
develop into pollen grains.
5. The mitosis happens in the nucleus of microspores to produce bigger vegetative cells and
smaller generative cells. They form a double-layered wall – outer exine is made up of
sporopollenin
And the inner intine is made up of cellulose and pectin. Generally, the pollen grains are
released at two-celled stages.
13. i) Explain the structure of maize grain with the help of a diagram.
Ans: In the grass family ( eg.Maize ) fruit is single-seeded where the pericarp and the seed
coat are fused together to produce the husk. Just below the husk, there is a layer of cells
known as the aleurone layer, this layer stores protein. There is a large endosperm that stores
starch. The embryo lies on one side of the endosperm & consists of a single cotyledon called
scutellum & embryonal axis. The region of the embryonal axis that points downward from the
point of attachment of cotyledons is a radicle and it is covered with a protective sheath known
as coleorhiza. The region of an embryonal axis that points upward from the point of
attachment of cotyledon is known as a plumule, it is covered by a foliaceous sheath called
coleoptile.
ii) Why cannot we use the term maize seeds for maize grains?
Ans: We cannot use the term seeds for maize grain because the seed is not completely
developed from the embryo but retains a part of the endosperm.
14. Trace the development of megasporocytes into the mature ovule.
Ans: (i) A single Megaspore mother cell is differentiated in the micropylar region of the
nucleus of an ovule & undergoes meiosis & forms a cluster of haploid cells called megaspore
tetrad. Of these, soon three degenerates & only one megaspore becomes functional
(ii) Functional megaspore enlarges to form embryo sac. Its nucleus undergoes mitotic division
& two nuclei move to opposite poles forming a 2-nucleate embryo sac.
(iii) Two successive mitotic divisions in each of these two nuclei result in information of 8-
nucleate embryo sac.
(iv) Three cells are grouped together at the micropylar end to form egg apparatus. consisting
of two synergids& a female egg cell.
(v) Three cells are grouped together at the chalazal end, they are called antipodal cells.
(vi) The remaining two nuclei are called Polar nuclei, they move to the center of the embryo
sac & fuse to form the secondary nucleus. Thus a typical angiosperm embryo sac is 8-
nucleate 7-celled.
15. “Incompatibility is the natural barrier in the fusion of gamete”. Justify this
statement.
Ans: Pollen grains of a plant species cannot germinate on the stigma of other non-related
species as both the species are incompatible and the process is called pollen – pistil
incompatibility. In many angiosperms plants, it is observed that pollen grains germinate on the
stigma of non-related species but male gametes produced in pollen tubes cannot fertilize
eggs. This is called gametic incompatibility. Self-incompatibility can be attained by utilizing
any of the following ways:-
1. Pollen Stigma interaction: - In this phenomenon, pollen grains fail to germinate on Stigma
because of incompatibility.
2. Pollen tube style interaction: - In this phenomenon, pollen grains become able to
germinate on stigma & pollen tube penetrate stigmatic surface but due to incompatibility
growth of pollen tube within stigma & style is inhibited.
3. Pollen – ovule interaction: - pollen tube successfully pierces & grows within style & its
growth is inhibited at micropyle of ovule.

16. How does pollination takes place in salivia. List any four adaptations required for
such type of pollination.
Ans: In salivia, entomophily/pollination of insects occur. The flowers of salivia are bilipped. Its
upper lip consists of two petals and its lower lip consists of three petals. The lower lip is
intended as a sitting pad for insects. In regular conditions, the connective stays upright. When
the insect penetrates the tube of the corolla in the direction of nectar sitting on the lower lip, it
pushes the sterile anther lobe which automatically brings about fertile anther to touch the
backside of insects and gets the blow of the fertile lobe. Pollen grains are dusted on the back
feathers and legs of insects
Adaptations for Entomophilous flower are :-
1. Flowers are brightly colored.
2. Flowers possess nectar glands.
3. pollen grains are usually sticky & spiny
4. flowers are large-sized & stout
5 Marks Questions
1. Draw the embryo sac of flowering plants and label:
(a) (i) Central Cell (ii) Chalazal end (iii) Synergids
Ans:

(b) Name the cell that develops into the embryo sac and explain how this cell leads to
the formation of the embryo sac.
Ans: Megaspore mom cell (MMC) undergoes meiotic department to provide four megaspores.
Out of these, three degenerates, and one useful megaspore develops into the embryo sac.
This form of improvement is known as a monosporic improvement.
(c) Mention the role played by various cells of the embryo sac.
Ans: Egg: The egg fuses with a male gamete to make a zygote or future embryo, Synergid:
Absorption of nutrients, it also attracts and guides pollen tubes. The Central Cell, after fusion
with the second male gamete, makes primary endosperm cell also which gives rise to the
endosperm.
(d) Give the role of filiform apparatus
Ans: Filiform apparatus guides the entry of pollen tubes.
2. Explain the formation of an embryo sac with diagrams.
Ans: The functional megaspore becomes bigger in size.
• The mitotic division occurs in the nucleus to make 2 nuclei that go in direction of
opposite poles.
• Every nucleus at the poles undergoes two mitotic divisions to form 4 nuclei in each pole
or a total of 8 nuclei.
• Two nuclei from each pole move to the center to produce the polar nuclei.
• The other nuclei, three at each pole, get surrounded by a bit of cytoplasm to form cells.
• The female gametophyte or the embryo sac thus has 7 cells and eight nuclei.
3. Explain the development of an embryo in a dicotyledonous plant with neatly labeled
diagrams.
Ans: The embryo develops at the micropylar end where the zygote is located. The zygote
begins developing after the particular part of the endosperm is developed to ensure nutrition
to the embryo. The zygote breaks mitotically to develop several stages including pro- embryo,
globular, heart-shaped, and eventually the developed embryo.
4. Describe the post-fertilization changes taking place in a flowering plant?
Ans:
(i) Development of Endosperm:- Endosperm development precedes embryo development.
The most common method of endosperm development is the nuclear type where triploid
endosperm (PEN) undergoes repeated mitotic divisions without cytokinesis – Subsequently
cell wall formation occurs from the periphery & endosperm store food materials.
(ii) Development of Embryo:-The zygote divides by mitosis in a pro-embryo first. Later
development causes the production of a globular and heart-shaped embryo & which ultimately
becomes a horseshoe-shaped embryo with one or more than one cotyledon. In a dicot
embryo, the portion of the embryonal axis about the level of attachment is epicotyl and it
reduces into plumule while part of the embryonal axis below the level of attachment is
hypocotyl and finally terminates into a radicle.
5. Trace the events that would take place in flower from the time of Pollen grain of
species fall on stigma up To completion of fertilization.
Ans:
Germination of Pollen Grains on Stigma
The pollen grains absorb fluid present on stigma and swell up. The exine ruptures at the place
of germ pore and intine come out in the form of a tube with its internal contents. This small
tubular structure is called a pollen tube and the process is called pollen germination.

ii) Entry of pollen tube into Ovule: - The entry of pollen tube into ovule happens by the
micropyle or chalaza or also by oblique sides of the ovule. Only 1 pollen tube begins inside
the embryo sac of an ovule. Usually, two synergids are eliminated by the entry of a pollen
tube into the embryo sac.
iii) Discharge of Mate Gametes:- Both male gametes are discharged into the embryo sac
after entering the pollen tube, either by generating two pores in the pollen tube and releasing
each male gamete through each pore or by the pollen tube rupturing and releasing the male
gametes into the embryo sac.

(4) Development of Embryo: - The zygote divides by mitosis into a pro-embryo first. After
development outcomes in production of heart and globular shaped embryo and also that will
eventually mature into an embryo of shapes of a horseshoe with one or more than one
cotyledon.
In a dicot embryo, the part of the embryonal axis about the level of addition is called epicotyl
and it eliminates into plumule while a portion of the embryonal axis under the level of
attachment is known as hypocotyl and it terminates into the radicle.
6. i) Why are zygotes dominant for some time in the fertilized ovule.
Ans: Zygote stays dominant for quite some time in a fertilized ovule because the embryo
develops after making of endosperm so zygote forms by the formation of endosperm which
supplies food material for developing embryo.
ii) What is polyembryony? Give an example.
Ans: Polyembryony is known as the presence of more than one embryo in a seed, Example-
Sometimes more than one embryo is created inside an embryo sac by the splitting of an egg
or by cleavage, endosperm, synergid, or antipodal.
iii) In fruits, what is formed from the following parts:-
a) Ovary wall b) Outer integument c) Inner integument d) Zygote
e) Primary endosperm f) Ovary g) Nucellus

Ans: In fruits, the following things are formed from given parts:
a Ovary wall Pericarp

b Outer integument Testa

c Inner integument Tegmen

d zygote embryo
e primary endosperm endosperm

f Ovary Fruit

g Nucellus perisperm.

Case based question


1. The pollen grains or microspores are the male reproductive bodies of a flower and are contained in the pollen sac or
microsporangia. Each pollen grain consists of a single microscopic cell, possessing two coats: the exine and the intine.
The exine of a pollen grain is made of chemically stable material. Because of this, pollen grains are often very well
preserved for thousands of years in soil and sediments.
(i) One of the most resistant biological material presents in the exine of pollen grain is:
A. Pectocellulosic B. Suberin C. Sporopollenin d. Cellulose.
ANS: I) sporopollenin
(ii) The exine possesses one or more thin places known as
A. Raphe B. Germ pores C. Hilum D. Endothecium.
ANS: ii) Germ pore
(iii) What is the function of germ pore?
A. Emergence of radicle B. Absorption of water for seed germination
C. Initiation of pollen tube D. All of these
ANS:iii) Initiation of pollen tube
(iv) What is the key advantage to the plant for having such strong pollen grain walls?
A. It protects the vital genetic material in the pollen grain.
B. It allows pollen to serve as a valuable fossil record for the study of ancient plants.
C. It prevents the pollen tube from growing out before the pollen grain reaches the stigma of a compatible
species.
D. It gives weight to the pollen grain, allowing it to cling better to the body surfaces of insect pollinators.
ANS:iv) it protects the vital genetic material in the pollen grain
(v)The number of germ pores in dicots and monocots respectively are
A. three and two B. two and three C. one and three D. three and one.
ANS:v)One and three
2. Read the following and answer any four questions from (i) to (iv) given below: Double Fertilisation 8
After pollen germination in flowering plants, the pollen tube penetrates through the stigma and the
tube grows through the style and reaches the ovary. Once it reaches the ovary, the tube penetrates
it and reaches the micropyle of the ovule and enters into the embryo sac. Here, one of the two
male nuclei fuse with the nucleus of the egg cell to form a zygote. This fusion of the male and
female gametes is known as syngamy. The other male gamete fuses with the two polar nuclei
located in the central cell to produce the triploid endosperm nucleus in a primary endosperm
cell(PEC). This fusion of three nuclei is termed as triple fusion. Since two types of fusions, syngamy
and triple fusion take place in an embryo sac, the phenomenon is termed double fertilization. The
zygote develops into an embryo while PEC develops into the endosperm. The cells of endosperm
tissue are filled withreserve food materials and are used for the nutrition of the developing embryo.
i. Name the diploid cell of the embryo sac.
a) Egg cell b) Ovary c) Ovule d) Central
cellAns. d) Central cell.
ii. Double fertilization is a phenomenon found in
a) Bryophytes b) Pteridophytes c) Angiosperms d) All of the
above.Ans. c) Angiosperms.
iii. At least how many pollen grains are required to fertilise 10 ovules in a
carpel?
a) 5 b) 10 c) 15 d) 20
Ans. b) 10
iv. The ploidy of a zygote formed as a result of syngamy is
a) Diploid b) Haploid c) Triploid d) None of the
above.Ans. a) Diploid.

3. Read the following and answer any four questions from (i) to (iv) given below: Pollinating agents The pollen-
pistil interaction begins with pollination, followed by pollen adhesion to the stigma. After it adheres, it
imbibes water and gets hydrated which initiates pollen tube germination. There are different agents of
pollination like wind, insects, birds and water.
Anemophilous flowers are pollinated by the agency of wind. These flowers are small andinconspicuous.. The
pollen grains are very light, non-sticky and sometimes winged.
Entomophilic flowers are pollinated by insects. These flowers are often attractive to look at with bright
petals and are fragrant to attract the insect visitors to them. They often have broad stigmas or anthers to
allow the insect to perch on it. Many of the insect-pollinated flowers also secrete nectar which attracts
bees, butterflies or other similar insects to the flowers. The pollen grain surface of such flowers produce
mucilaginous secretion. Hydrophilic flowers are pollinated by water. It is commonly found in algae,
bryophytes, pteridophytes and some angiosperms. The pollen grains may have a mucilaginous covering to
protect it from getting wet.
i. The pollinating agent of an inflorescence of small dull coloured flowers with well exposedstamens
and large feathery stigma is
a. Water b. Wind c. Insects d.
Birds.Ans. b. Wind.
ii. An example of biotic agent for pollination is
a. Air b. Water c. Honey bee d. All of the
above.Ans. c. Honey bee.
iii. The pollen grains in the flowers are generally sticky that help them to
a. stick on to the body of the insects b. float on water
c. float in the air d. fall on the ground safely.
Ans. a. stick on to the body of the insects
iv. Which of the following statements seem to describe the water-pollinated
submergedplants?
a. The flowers do not produce nectar. b. The flower petals are not brightly coloured.
c. The pollen grains have mucilaginous covering.
d. The female flowers have long stalk to reach the surface.
Ans. d. The female flowers have long stalk to reach the surface.
v. What are the problems with hybrid seeds?
a. Hybrids seeds has to be produced every year.
b. Seeds obtained from hybrid plants when grown, tend to segregate and loose the
hybridtraits.
c. Cost factor d. All of the above.
d. Ans. d. All of the above.

ASSERTION – REASON BASED QUESTIONS

Directions: In the following questions, statement of assertion is followed by a statement of reason.


Mark the correct choice as:
a) If both assertion and reason are true and reason is the correct explanation of assertion.
b) If both assertion and reason are true but reason is not the correct explanation of assertion.
c) If assertion is true but reason is false.
d) If both assertion and reason are false.
1. Assertion: An angiospermous flower represents the modified condensed shoot.
Reason: The fertile leaves of the shoot become modified into microsporophylls which bear ovules.
Ans: c)
2. Assertion: A typical microsporangium of angiosperms is generally surrounded by four wall layers-
epidermis, endothecium, middle layers and tapetum.
Reason: The outer three wall layers perform the function of protection and help in dehiscence of
anther.
Ans: b)
3. Assertion: In a microsporangium, the tapetal cells posses little cytoplasm and generally have a
single prominent nucleus.
Reason: During microsporogenesis, the microspore mother cells (MMCs) undergo meiosis divisions
to produce haploid microspore tetrads.
Ans: d)
4. Assertion: In most angiosperms, microspore of a tetrad grow and separate from one another
shortly after meiosis.
Reason: In the members of family Asclepiadaceae, all the pollen grains of a sporangium remain
united to form a compact structure called pollinium.
Ans: b)
5. Assertion: Exine of a pollen grain is made up of sporopollenin which is resistant to high
temperature, strong acid or alkali as wel as enzymatic degradation.
Reason: Sporopollenin is absent in the region of germpore.
Ans: b)
6. Assertion: The development of embryo sac from a single functional megaspore is termed as
monosporic development.
Reason: In monosporic (Polygonum) type of embryo sac development, usually the megaspore which
is situated towards micropylar end remains functional.
Ans: c)
7. Assertion: Although geitonogamy is a functionally cross pollination involving a pollinating agent,
genetically it is similar to autogamy.
Reason: In geitonogamy, pollen grains from the anthers of one flower are transferred to the stigma of
another flower.
Ans: a)
8. Assertion: Hydrophilly constitute a major mode of pollination in most of the aquatic angiospermous
plants.
Reason: Vallisneria and Zostera are examples of water pollinated plants.
Ans: b)
9. Assertion: Only pre-pollination growth of male gametophyte occurs inside the microsporangium
whereas the remaining growth occurs over the female reproductive organs.
Reason: Whole of the growth of female gametophyte occurs inside the megasporangium.
Ans: b)
10. Assertion: Self-incompatibility is a genetic mechanism which prevents self pollination by inhibiting
either pollen germination or pollen tube growth in the pistil.
Reason: In gametophytic self- incompatibility, the incompatibility reaction is determined by the
genotype of the sporophytic tissue.
Ans: c)
11. Assertion: In angiosperms, endosperm development precedes embryo development.
Reason: Double fertilization ensures that the nutritive tissue is formed before the zygote starts
cleaving.
Ans: a)
12. Assertion: In Cocos nucefera, coconut water represents the cellular endosperm and the
surrounding water kernel represents the free – nuclear endosperm.
Reason: Endosperm persists in some mature seeds.
Ans: d)
13. Assertion: During development of embryo in dicots, suspensor serves as the main nutritive tissue
for the embryo.
Reason: The last cell of the suspensor at the end adjacent to the embryo is known as hypophysis.
Ans: d)
14. Assertion: Ex-albuminous seed do not possess any residual endosperm as it is completely
consumed during embryo development.
Reason: Wheat, castor, pea and groundnut all are examples of ex-albuminous seed.
Ans: c)
15. Assertion: In plants, apomixes is a form of asexual reproduction that mimics sexual reproduction.
Reason: Apomixis involves the production of seeds without the fusion of gametes.
Ans: a)
16. Assertion: Autogamy is a transfer of pollen grains from an anther to the stigma of the same
flower on the same plant.
Reason: Xenogamy is pollination between two flowers on different plants.
Ans: b)
17. Assertion: Insects visit flower to gather honey.
Reason: Attraction of flowers prevents the insects from damaging other parts of the plant.
Ans: d)
18. Assertion: Pollen mother cells (PMCs) are the first male gametophytic cells.
Reason: Each PMC gives rise to two pollens.
Ans: d)
19. Assertion : Chasmogamous flowers require pollinating agents.
Reason: Cleistogamous flowers do not expose their sex organs.
Ans: b)
20. Assertion: Gynoecium consists of pistil.
Reason: It represents the male reproductive part in flowering plants.
Ans: c)
21. Assertion: Flowers are the structures related to sexual reproduction in flowering plants.
Reason: Various embryological processes of plants occur in a flower.
Ans: a)
22. Assertion: Geitonogamy is genetically similar to autogamy.
Reason: The pollen grains come from same plant.
Ans: a)
23. Assertion: Cleistogamous flowers produce assured seed set in the absence of pollinators.
Reason: These flowers do not open at all.
Ans: a)
24. Assertion: A typical microsporangium of angiosperms is generally surrounded by four wall layers.
Reason: The outer three wall layers perform the function of protection and help in dehiscence of
anther to release the pollen.
Ans: b)
25. Assertion: Exine of a pollen grain is made up of sporopollenins which are resistant to high
temperatures, strong acids or alkali as well as enzymatic degradation.
Reason: Sporopollenins are absent in the region of germ pores.
Ans: b)
26. Assertion: An angiospermous flower represents the modified condensed shoot which performs
the function of sexual reproduction.
Reason: The fertile leaves of the shoot become modified into microsporophylls and magasporophylls
which bear ovules and anthers respectively.
Ans: c)
27. Assertion: Although geitonogamy is functionally cross-pollination involving a pollinating agent,
genetically it is similar to autogamy since the pollen grains come from the same parent.
Reason: In geitonogamy, pollen grains from the anthers of one flower are transferred to the stigma of
another flower borne on the same plant.
Ans: a)
28. Assertion: The pre-pollination growth of male gametophyte occurs inside the microsporangium
whereas the rest of the growth occurs over the female reproductive organs.
Reason: Growth of the entire female gametophyte occurs inside the megasporangium.
Ans: b)
29. Assertion: Hydrophily is a major mode of pollination in most of the aquatic plants in angiosperms.
Reason: Almost all the aquatic dicot and monocot plants require water for the transport of male
gametes and for fertilisation.
Ans: d)
30. Assertion: Pollen grains from male parent are mostly transferred to the stigma in the female
parent by some external agency.
Reason: This is because the male flowers or male organs have no internal device to reach the female
organs in another flower.
Ans: a)
Important You tube link to understand contents--
https://youtube.com/playlist?list=PLMQhn0MTk6aLevo-Yw3wDG-rW6XtaIWMd
UNIT VI – REPRODUCTION
CHAPTER 2-HUMAN REPRODUCTION
Key Words
Scrotum It is a pouch-like structure outside the abdominal cavity in which the testers are situated.
It helps in maintaining the low temperature of the testes (2–2.5o C lower than the normal
internal body temperature) necessary for spermatogenesis.
Somniferous tubule In each testicular lobule 1- 3 highly coiled seminiferous tubules in which sperms are
produced.
Sertoli cells Sertoli cells provide nutrition to the male germ cells.
Leydig cell Present in interstitial space (regions outside the seminiferous tubules). It secretes
testicular hormones called androgens
Urethral meatus It is the external opening of the penis.
Seminal plasma Secretion of the male accessory glands (seminal vesicles, prostate and bulbourethral
gland) constitute the seminal plasma.
Male accessory glands Paired seminal vesicles, prostate and paired bulbourethral glands
Lactiferous duct Through which milk is sucked out.
Spermatogonia Immature diploid male germ cells (Spermatogonia) produce sperms by spermatogenesis.
These are present on the inside wall of the seminiferous tubule.
Acrosome Cap-like structure on sperm head. it is enzymes that help fertilization of the ovum.
Semen The seminal plasma along with the sperms constitutes the semen.
Oogonia Gamete mother cells (oogonia) are formed within each fetal ovary which later develops
into the egg.
Graafian follicle Mature follicles formed during oogenesis. When it own ruptures releases the secondary
oocyte (ovum) and reaming part forms the carpus luteum.
Zona pellucida The membrane on the secondary oocypreventsvent polyspermy
Carpus luteum After ovulation remaining parts of the Graafian follicle transform in the corpus luteum. It
secretes progesterone hormone, which is essential for the maintenance of the
endometrium.
Polar body It is a small haploid cell that is formed during oogenesis. It can’t be fertilized.
Cleavage The mitotic division in the zygote is called cleavage.
Placenta The chorionic villi and uterine tissue become interdigitated with each other and jointly
form a structural and functional unit called the placenta
Chorionic villi After implantation, finger-like projections appear on the trophoblast called chorionic villi
which are surrounded by the uterine tissue and maternal blood.
Stem cell Inner cell mass contains certain cells called stem cells which have the potency to give rise
to all the tissues and organs.
Implantation Implantation is the process in which the mammalian embryo (blastocyst) becomes
attached to the endometrium of the uterus.
Fetus ejection reflex Mild uterine contractions are generated by the placenta when the fetus is fully developed,
at the time of parturition.
Spermatogenesis and Oogenesis
FERTILIZATION
The fusion of sperm and ovum is called fertilization.
• Site- ampulla-isthmic junction.
• Zona-pellucida layer of the ovum block the entry of
the additional sperms thus only one sperm fertilizes the ovum.
IMPLANTATION

PREGNANCY AND EMBRYONIC DEVELOPMENT

STAGES OF EMBRYO DEVELOPMENT

One month Heart is formed


Two month Development of limbs and digits

12 weeks (first trimester) Most of the major organ systems are formed, for example, the
limbs and external Genital organs are well-developed.
Fifth month First movements and appearance of hair on the head

After 24 weeks (end of the second The body is covered with fine hair, eyelids separate, and
trimester) eyelashes are formed.

9 month Foetus is fully developed

PARTURITION

Gestation period: The average duration of human pregnancy (9 months).


Parturition: the process of delivery of the fetus.
Parturition is a neuroendocrine mechanism. The signals for parturition originate from the fully developed fetus and
placenta which induces mild uterine contractions called foetal ejection reflex. Due to this oxytocin hormone is released
from the maternal pituitary which causes stronger uterine contractions, which in turn stimulates further secretion of
oxytocin. Continuous secretion of oxytocin causes stronger and stronger contractions. This leads to the expulsion of
the baby out of the uterus through the birth canal – parturition.
LACTATION

• The mammary glands of the female starts producing milk towards the end of pregnancy through the process
called lactation.
• The milk produced during the initial few days of lactation is called colostrum which contains several
antibodies essential to develop resistance for newly born babies.

IMPORTANT QUESTIONS
Very Short Answer Type Questions/ MCQ
1- LH surge is responsible for-
a- Ovulation
b- Spermatogenesis
c- Oogenesis
d- Fertilization
Ans: a.
2- Spermiogenesis is
a- Transformation of spermatozoa into spermatids
b- Transformation of spermatids into spermatozoa
c- Both are incorrect
d- A is correct while b is incorrect
Ans: B
3- What is the role of polar bodies-
a- Maintain the number of chromosomes in the ootid
b- Prevent polyspermy
c- Release progesterone
d- All of these
Ans: a
4- Despite the presence of So many sperms in the vicinity of an egg cell, only one sperm enters the ovum. Why?
Ans: Because when sperm comes in contact with the ovum (Zona pellucida) & induces changes in the
membrane to block entry of other sperms
5- Define spermiogenesis. Where does it occur?
Ans: The transformation of non-motile spermatids into motile spermatozoa is called spermiogenesis. It occurs
inside the seminiferous tubules of the testes.
6- Name the cells which produce testosterone. What is the function of this hormone?
Ans: Interstitial cells (Leydig’s cells) of the testis secrete the testosterone hormone. Function. They control
secondary sexual characters.
7- The Spermatogonia cell has 46 chromosomes in human males. Give the number of chromosomes in (a)
Primary spermatocyte (b) Spermatid
Ans: (a) Primary spermatocyte – 46 (b) Spermatid- 23
8- How does colostrum provide initial protection against diseases in newborn infants? Give one reason.
Ans: Colostrum contains immunoglobin which provides immunity to the infants.
9- Name the embryonic stage which gets implanted in the uterine wall of a human female.
Ans: blastocyst stage
10- Name the hormone which causes strong uterine contraction during parturition. Does the parturition signal
originate from the mother or the fetus?
Ans: Hormone - is Oxytocin
The signal originates from the placenta and fully developed fetus.
11- Where are sperm stored in males?
Ans: Epididymis
Short Answer Type Questions

1- How is a primary oocyte different from a secondary oocyte?


Ans: The primary oocyte is a diploid cell (2n) whereas the secondary oocyte is a haploid cell (n). The primary
oocyte is formed when oogonia are at the prophase-I of the meiotic division in the foetal ovary whereas the
secondary oocyte is formed from the primary oocyte after meiosis-I – I division to produce ova in females
during the stage of puberty.
2- Name the muscular and glandular layers of the human uterus. Which one of these layers undergoes cyclic
changes during the menstrual cycle? Name the hormone essential for the maintenance of this layer.
Ans: Muscular layer – Myometrium
Glandular layer – Endometrium.
The endometrium undergoes cyclic changes during the menstrual cycle.
LH hormone maintains the lining of the uterus.
3- State the fate of the Trophoblast of a human blastocyst at the time of implantation and that of the inner
cell mass immediately after implantation.
Ans: The Trophoblast layer of the human blastocyst gets attached to the endometrium and the inner cell mass
gets differentiated into an embryo. After attachment, the uterine cells divide rapidly and cover the blastocyst.
As a result, the blastocyst becomes embedded in the endometrium of the uterus. It is termed Implantation.
4- Where are fimbriae present in the human female reproductive system? Give their function.
Ans: Fimbriae are fingerlike projections present at the end of the fallopian tubes.
Through fimbriae, eggs move from the ovaries to the uterus.
5- Name the muscular and glandular layers of the human uterus. Which one of these layers undergoes cyclic
changes during the menstrual cycle? Name the hormone essential for the maintenance of this layer.
Ans: Muscular layer – Myometrium, Glandular layer – Endometrium, Endometrium undergoes cyclic changes
during the menstrual cycle.
Progesterone hormone maintains the lining of the uterus.
6- What are the functions of the placenta other than its endocrine function?
Ans: The placenta helps in the supply of nutrients and oxygen to the embryo, the elimination of excretory
wastes and carbon dioxide produced by the embryo.
7- What is corpus luteum? How does it function as an endocrine gland?
Ans. During LH surge the Graafian follicle ruptures to release secondary oocyte. The remaining part of the
Graafian follicle forms the corpus luteum. Corpus luteum functions as endocrine glands as they secrete
progesterone.
8- Differentiate between Sertoli cells and Leydig cells concerning their location in the organ
and their function.
Ans: Sertoli cells are the nutritive cells present inside seminiferous tubules. They provide nourishment to the
male germ cells.
Leydig cells are present outside the seminiferous tubules in the interstitial spaces, they secrete androgens.
9- Draw a sectional view of the seminiferous tubules o humans females and label any four parts.
Ans: fig 3.4, page 47, NCERT
10- What are the various male accessory glands? Give their function.
Ans. The male accessory glands include paired seminal vesicles, a prostate gland and paired bulbourethral
glands.
These glands secrete seminal plasma rich in fructose, calcium and certain enzymes. Secretions of bulbourethral
glands help in the lubrication of the penis.

Long Answer Type Questions

1- Explain the organization of the mammary gland with the help of a diagram.
Ans: Mammary gland occurs in pairs. It contains glandular tissues which are organized into 15-20 mammary
lobes in each breast. Mammary lobes possess alveolar cells. These cells secrete milk. The alveoli open into the
mammary tubules which combine to form the mammary duct. Mammary ducts join to form a mammary
ampulla that is connected to the lactiferous ducts. Through these structures, milk is sucked.
Fig- 3.4, page 46, NCERT
2- Explain the menstrual cycle in human females by giving a suitable diagram.
Ans: Menstrual cycle has three phases: menstrual, proliferative, and secretory.
a) Menstrual Phase (3-5 days)- During the endometrial lining of the uterus is ruptured and is released out from
the vagina along with blood vessels and mucus.
b) Proliferative or Follicular Phase about 11 days): in this phase ovarian follicle matures into a Graffian follicle.
The endometrial layer again proliferates. In this phase, ovulation occurs.
c) Secretory Phase 9about 12 days)- Corpus Luteum is formed from a ruptured Graffian follicle. The
endometrium grows and thickens further.
Fig-3.9, page 50, NCERT
3- Explain the changes that take place during maturation of a follicle top Graffian follicle in the ovary.
Ans: Oogonia (gamete mother cells) are formed within each fetal ovary. No more Oogonia are formed after
birth. Oogonia enter into the prophase-I stage of meiotic division and form primary oocytes.
The primary oocytes get surrounded by granulosa layers forming the primary follicle and then secondary
follicles. The secondary follicle is transformed into a tertiary follicle characterized by an antrum (a fluid-filled
cavity). The tertiary follicle matures into the Graafian follicle.
4- Give a schematic representation of spermatogenesis and oogenesis.
Ans: Fig 3.8, page 49, NCERT
5- Describe the hormonal control of the male reproductive system.
Ans:
i- Hypothalamus: It releases GnRH (gonadotropin-releasing hormones) which stimulate the pituitary
glands to secrete gonadotropins.
ii- LH and FSH
iii- Follicle Stimulating Hormone (FSH): It stimulates spermatogenesis in seminiferous tubules of the
testis.
iv- LH stimulates testosterone production from the interstitial cells of the testes.
v- Sertoli cells secrete androgen binding protein (ABP) which concentrates testosterone in the
seminiferous tubules.

CASE BASED QUESTION


CBQ 1 -Read the following and answer the questions given below:
Oogenesis is the process of formation of ovum in ovaries. It consists of three phases:
multiplication,growth and maturation. Oogenesis is controlled by hormones GnRH,LH,FSH. GnRH secreted
by the hypothalamus stimulates the anterior lobe of pituitary gland to secreteLH and FSH.
i. What is the function of hormone FSH?
a. It inhibits the formation of estrogen. b .It induces the release of secondary oocyte.
c. It stimulates the growth of Graafian follicle. d. causes ovulation.
Ans: c
ii. Which hormone induces the rupture of the mature Graffian follicle?
a. Follicle stimulating hormone b. Gonadotropin releasing hormone
c. Progesterone d. Luteinising hormone
Ans: d

20
iii. Identify the function(s) of LH.
a. Release of secondary oocyte from Graafian follicle. b. corpus luteum to secrete progesterone.
c. Stimulates estrogen formation. d. Promotes development of egg to form secondary oocyte
a. and b only b. b and c only c. c and d only d. b only
Ans: a
CBQ -2
Spermatogenesis is the production of sperms from male germ cells (spermatogonia) inside the testes
(seminiferous tubule). This process begin at puberty. Observe the following flow diagram and answer the
questions that follows-

(i) This happens during spermatogenesis


(a) Meiosis
(b) Mitosis
(c) Meiosis and mitosis
(d) None of these
Answer: (c)
(ii) The process of spermatogenesis is induced by
(a) TSH
(b) FSH
(c) MSH
(d Answer: (b)

(iii) The number of spermatozoa, a single primary spermatocyte finally produced in


spermatogenesis is
(a) 2
(b) 4
(c) 6
Answer: (b)
(iv) In spermatogenesis, the phases of maturation involve
(a) formation of spermatids from primary spermatocyte through meiosis
(b) growth of spermatogonia into primary spermatocytes
(c) formation of spermatogonia from gonocytes through mitosis
(d) formation of oogonia from spermatocyte through meiosis
Answer: (a)
(v) The correct sequence of cell stage in spermatogenesis is
(a) spermatocyte → spermatids → spermatogonia → spermatozoa
(b) spermatogonia → spermatids → spermatocyte → spermatozoa
20
(c) spermatocytes → spermatogonia → spermatid → spermatozoa
(d) spermatogonia → spermatocytes→ spermatids → spermatozoa
Answer: (d)
ASSERTION – REASON BASED QUESTIONS
Directions: In the following questions, statement of assertion is followed by a statement of reason.
Mark the correct choice as:
a) If both assertion and reason are true and reason is the correct explanation of assertion.
b) If both assertion and reason are true but reason is not the correct explanation of assertion.
c) If assertion is true but reason is false.
d) If both assertion and reason are false.
1. Assertion: A drop in temperature does not affect spermatogenesis.
Reason: During temperature drop the smooth muscles contract and bring the testes closer to the
pelvic cavity.
Ans: a)
2. Assertion: the regions outside the seminiferous tubule are called interstitial spaces which cantain
leydig cells.
Reason: Leydig cells synthesise and secrete testicular hormones called androgens.
Ans: b)
3. Assertion: Wall of seminiferous tubule is formed of double layered germinal epithelium.
Reason: Majority of cells of germinal epithelium are germ cells and some are sertoli cells.
Ans: d)
4. Assertion: Infundibulum is a funnel shaped part closer to ovary.
Reason: The edges of the infundibulum help in collection of ovum after ovulation.
Ans: b)
5. Assertion: The shape of the uterus is like an inverted pear.
Reason: The inner glandular layer that lines the uterine cavity is called as myometrium.
Ans: c)
6. Assertion: The endometrium undergoes cyclical changes during menstrual cycle.
Reason: The myometrium exhibits strong contraction during delivery of the baby.
Ans: b)
7. Assertion: The female external genital includes mons pubis, labia majora and labia minora.
Reason: The glandular tissue of each breast is divided into 5-10 mammary lobes.
Ans: c)
8. Assertion: The type B spermatogonia are called primary spermatocytes.
Reason: Primary spermatocytes complete the first meiotic division leading to secondary
spermatocytes.
Ans: b)
9. Assertion: The middle piece is called as power house of the sperm.
Reason: The numerous mitochondria coiling around axial filament produce energy for the movement
of the tail.
Ans: a)
10. Assertion: Human male ejaculates about 50-100 million sperms during a coitus.
Reason: Semen has an alkaline Ph to neutralise acidity of urethra.
Ans: d)
11. Assertion: All copulations do not lead to the fertilisation and pregnancy.
Reason: Fertilisation can occur only if the ovum and sperms are transported simultaneously to the
ampullary-isthmic junction.
Ans: a)
12. Assertion: The embryo with 8 to 16 blastomeres is called a morula.
Reason: The morula continues to divide and and transforms into trophoblast.
Ans: c)
13. Assertion: After implantation, finger like projections appear on the trophoblast called chorionic
villi.
Reason: Chorionic villi are surrounded by the uterine tissue and maternal blood.
20
Ans: b)
14. Assertion: During pregnancy the levels of hormone like estrogens and progestogens are
increased.
Reason: The increased production of these hormones is essential for foetal growth.
Ans: b)
15. Assertion: Vigorous contraction of the uterus at the end of pregnancy causes expulsion.
Reason: The stimulatory reflex between the uterine contraction and oxytocin secretion results in
weakening contractions.
Ans: c)
16. Assertion: In human male, testes are extra abdominal and lie in scrotal sacs.
Reason: Scrotum acts as thermoregulator and keeps testicular temperature lower by 2°C for normal
spermatogenesis.
Ans: a)
17. Assertion: Testicular lobules are the compartments present in testes.
Reason: These lobules are involved in the process of fertilization.
Ans: d)
18. Assertion : Interstitial cell is present in the region outside the seminiferous tubule called
interstitial spaces.
Reason: Interstitial cells provide nutrition to the sertoli cells.
Ans: c)
19. Assertion: The testes are situated outside the abdominal cavity within the scrotum.
Reason: Muscles in scrotum helps to maintain low temperature of testes, necessary for
spermatogenesis.
Ans: a)
20. Assertion: The bulbourethral gland is a male accessory gland.
Reason: Its secretion helps in the lubrication of the penis, thereby facilitating reproduction.
Ans: a)
21. Assertion: Each seminiferous tubule is lined on its inside by three type of cells.
Reason: These cells are male germ cells, Sertoli cells and Leydig cells.
Ans: d)
22. Assertion: In human male, there are perianal glands near the anus.
Reason: Perianal glands secrete sex-attractant pheromone which initiates sexual desire in human.
Ans: d)
23. Assertion: Testes are located in the scrotum, outside the coelom.
Reason: A vaginal coelom partly surrounds the testes in the scrotum.
Ans: c)
24. Assertion: Fimbriae are finger-like projections of the infundibulum part of oviduct which is closest
to ovary.
Reason: They are important for collection of ovum after ovulation from ovary.
Ans: b)
25. Assertion: Finger-like projections appear on the trophoblast called chorionic villi after
implantation.
Reason: Chorionic villi are surrounded by the uterine tissue and maternal blood.
Ans: b)
26. Assertion: Infundibulum is a funnel shaped part closer to ovary.
Reason: The edges of infundibulum help in collection of the ovum after ovulation.
Ans: b)
27. Assertion: The female external genitalia include mons pubis, labia majora and labia minora.
Reason: The glandular tissue of each breast is divided into 5-10 mammary lobes.
Ans: c)
28. Assertion: Vagina acts as copulation canal and fertilization canal.
Reason: Both insemination and fusion of gametes occur in the vagina of female.
Ans: d)

20
29. Assertion: In the testis, spermatogenesis occurs in the seminiferous tubules and testosterone
secretion takes place from the sertoli cells.
Reason: Testosterone brings growth and maturation of primary sex organs and also development of
accessory sex characters.
Ans: d)
30. Assertion: Spermatogenesis starts at the age of puberty.
Reason: There is a significant increase in level of gonadotropin releasing hormone at puberty.
Ans: a)

Important You tube link to understand contents--


https://youtube.com/playlist?list=PLMQhn0MTk6aLevo-Yw3wDG-rW6XtaIWMd

20
CHAPTER- 4
REPRODUCTIVE HEALTH

WHO: reproductive health means total Well-being in all aspects of reproduction i.e. physical, emotional, behavioral
and social.
REPRODUCTIVE HEALTH: PROBLEM AND STRATEGIES

Amniocentesis –
• Amniocentesis is a procedure used to take out a small sample of the amniotic fluid for testing of chromosomal
abnormalities in a developing embryo.
• Misuse- It is also misused to check fetal sex determination based on the chromosomal pattern in the amniotic
fluid surrounding the developing embryo.

POPULATION EXPLOSION AND BIRTH CONTROL

MMR- Maternal mortality rate IMR- Infant mortality rate


21
CONTRACEPTIVE METHODS

Natural methods work on the principle of avoiding chances of ovum and sperm meeting.
Periodic abstinence couples avoid or abstain from coitus from day 10 to 17 of the menstrual
cycle when ovulation could be expected
Withdrawal or coitus The male partner withdraws his penis from the vagina just before ejaculation to
interruptus avoid insemination
Lactational amenorrhea as long as the mother breast-feeds the child fully, chances of conception are almost
nil (up to 6 months)
Barrier methods Ovum and sperm are prevented from physically meeting with the help of barriers.
Condoms Also, protect from STDs
Other examples of Diaphragms, cervical caps, and vaults.
barrier methods
IUDs Inserted in the uterus through the vagina.
IUDs increase phagocytosis of sperms within the uterus and the Cu ions released
suppress sperm motility and the fertilizing capacity of sperms.
Non-medicated Lippes loop
Copper releasing CuT, Cu7, Multiload 375
Hormone releasing IUDs Progestasert, LNG-20
Pills
progestogens or taken daily for a period of 21 days starting preferably within the first five days of the
progestogen–estrogen menstrual cycle. After a gap of 7 days (menses), it has to be repeated in the same
combinations pattern.
Pills inhibit ovulation and implantation and alter the quality of cervical mucus to
prevent/ retard entry of sperms.
CDRI:- Central drug research institute (Lucknow)
It develops Saheli–a new oral contraceptive pill for females. Saheli –the new oral
contraceptive for females contains a non-steroidal preparation. It is a ‘once a week’
pill with very few side effects and high contraceptive value.
Injection
Progestogens alone or in combination with estrogen are also used.
If used within 72 hours of coitus is very effective.
Surgical Methods/ Sterilization
Vasectomy in males, a small part of the vas deferens is removed or tied up through a small
incision on the scrotum
Tubectomy a small part of the fallopian tube is removed or tied up through a small incision in
the abdomen or through the vagina.
An ideal contraceptive should be-

MEDICAL TERMINATION OF PREGNANCY (MTP)


• Voluntary termination of pregnancy before full term is called MTP or induced abortion.
• In India, MTP is legalized in 1971 with some strict conditions to avoid its misuse like female foeticide.
• MTP is used to get rides of unwanted pregnancy due to unprotected intercourse or failure of contraceptives
used during coitus or rapes.
• It is relatively safe during the first trimester or up to 12 weeks of pregnancy.

SEXUALLY TRANSMITTED DISEASES (STDS)


22
• Diseases or infections which are transmitted through sexual intercourse are collectively called sexually
transmitted diseases (STD) or venereal diseases (VD) or reproductive tract infections (RTI).
• Examples- HIV- AIDS, Genital warts, Hepatitis, Gonorrhea, syphilis, genital herpes, chlamydiosis,
trichomoniasis etc.
• Some infections like Hepatitis-B and HIV are also transmitted by sharing injection needles, surgical instruments
with the infected person, transfusion of blood, or from infected mother to foetus.
One could be free of these infections by following the simple principles given below:
(i) Avoid sex with unknown partners/multiple partners.
(ii) Always use condoms during coitus.
(iii) In case of doubt, one should go to a qualified doctor for early detection and get complete treatment if diagnosed
with the disease.
INFERTILITY
Couples which are unable to reproduce children in despite of protected sexual cohabitation are called infertile.
Reason of infertility
Physical, Congenital, Diseases, Drugs, Immunological, Psychological,
ART (assisted reproductive technologies)-
Infertile couples can be assisted to have children through certain special techniques commonly called (ART).
Examples of ARTs-

The ovum from the wife/donor Transfer of Specialized infertility cases in which the male
spermerms frthe om husband/ gametes procedure to partner is unable to produce
donor are collected and induced to collected from a form an embryo healthy sperm are treated by AI.
fertilthe ize in lab. The zygote or donor into the in the laboratory In this semen collected from a

23
early embryo (8 blastomeres) fallopian tube of in which sperm donor is artificially introduced
could be transferred into a another female is directly into the vagina or the uterus, IUI
fallopian tube called ZIFT (zygote who does not injected into the (intrauterine insemination) of
infra fallopian transfer) and an produce ovum. ovum. the female.
embryo with more than 8
Blastomere IUT (intra uterine
transfer) into the uterus to
complete the further development

IVF/ Test Tube Baby GIFT ICSI AI

IMPORTANT QUESTIONS
Very Short Answer Type Questions/ MCQ
1- Test tube baby implies which of the following techniques?
a- IUT
b- ICSI
c- ZIFT
d- All of these
Ans: C
2- Which of these can be used to cure infertility in couples where the male partner has a very low sperm count?
(a) IUD
(b) GIFT
(c) IUI
(d) None of these
Ans: C
3- Which part of the female is mainly involved in the tubectomy?
a- Fallopian tube
b- Endometrium
c- Myometrium
d- Vagina
Ans: A
4- Following statements are given regarding MTP.
(i) MTPs are generally advised during the first trimester
(ii) MTPs are used as a contraceptive method
(iii) MTPs are always surgical
(iv) MTPs require the assistance of qualified medical personnel
Choose the correct option.
(a) (ii) and (iii)
(b) (i) and (iii)
(c) (i) and (iv)
(d) (i) and (ii)
Ans: C
5- Sterilization techniques are generally very effective methods of contraception with the least side effects.
Yet, this is the last option for the couples because
(i) It is almost irreversible
(ii) the misconception that it will reduce sexual urge
(iii) it is a surgical procedure
24
(iv) of lack of sufficient facilities in many parts of the country
Choose the correct option.
(a) (i), (ii), (iii) and (iv)
(i) and (iii)
(b) (ii) and (iii)
(c) (ii) and (iv)
Ans: A
6- Define reproductive health according to WHO. Which society will be called a reproductively healthy society?
Ans: According to the World Health Organisation (WHO) - reproductive health means total Well-being in all
aspects of reproduction i.e. physical, emotional, behavioral and social. A reproductively healthy society
includes people having physically and functionally healthy reproductive organs. They have normal behavioral
and emotional interactions in sex-related matters.
7- Give the term for prenatal diagnostic technique aimed to know the sex of the developing foetus and to
detect congenital disorders.
Ans: Amniocentesis
8- Which research institute develops the contraceptive pill “Saheli”?
Ans: CDRI (Central Drug Research Institute) is located in Lucknow.
9- Why tubectomy is considered a contraceptive method?
Ans: In tubectomy, a small part of the fallopian tube is cut and tied up to block the entry of spetos to prevent
fertilization.
10- Which methods of contraception are also known as sterilization methods?
Ans: Tubectomy and Vasectomy
11- Expand ICSI.
Ans: Intra cytoplasmic sperm injection.
12- Categorize the following contraceptive methods as natural, IUDs, Barrier or hormonal-
Lippes loop, lactation amenorrhea, Vault, Saheli
Ans: Lippes loop- IUD, Lactational amenorrhea – Natural, Vault- Barrier, Saheli- Hormonal
13- Why has the Government imposed a statutory ban on amniocentesis?
Ans: The Government has banned amniocentesis to check on the incidences of female foeticides.
14- Name two STDs that can be transmitted through contaminated blood.
Ans: Hepatitis, HIV-AIDS
15- How can a possible pregnancy due to rape or casual unprotected intercourse be avoided?
Ans: Administration of progestogens or progestogens-estrogen combinations or IUDs within 72 hours of coitus
Short Answer Types Questions
1- Removal of gonads cannot be considered as a contraceptive option. Justify.
Ans: Removal of gonads leads to sterility of individuals therefore it is not considered as a contraceptive option.
2- What do oral pills contain and how do they act as effective contraceptives?
Ans: Oral contraceptives or pills are either progestogens or progestogen-estrogen combinations.
They function as contraceptives by
(i) Inhibiting ovulation. (ii) Inhibiting implantation. (iii) Altering the quality of cervical mucus to prevent or stop
the entry of sperm
3- Mention simple principles by which one could be free of STDs.
Ans- i- Avoid sex with unknown partners/ multiple partners. Ii-Always use a barrier method of
contraceptive Iii-Consult doctor in case of any symptom for early detection and treatment
4- Fill the a. b, c, d with the appropriate answer in the given table-
Method of Birth Control Device
1. Barrier A
2. IUD B
25
3. Surgical Technique C
4. Natural D
Ans: a- Condom, b- Cu- t, Vasectomy d- Coitus interruptus
5- Mention at least four reasons for sterility.
Ans: physical, congenital diseases, drugs, immunological or physiological
6- Expand ZIFT and IUT. How these are different from each other?
Ans: ZIFT (zygote intrafallopian transfer) – it is the transfer of the zygote or early embryos (with up to 8
blastomeres) into the fallopian tube.
IUT– intra uterine transfer- in this Embryo with more than 8 blastomeres is transferred into the uterus
7- Mention one positive and one negative application of amniocentesis.
Ans- Positive- helpful in the detection of congenital disorder
Negative- used in sex determination and female foeticides
8- An infertile couple is advised to adopt a test tube baby programme. Describe two principles and procedures
adapted to such technologies.
Ans: In-vitro fertilization (IVF): In this process, fertilization takes place outside the body (test tube baby). The
following techniques are included in IVF:
ZIFT - In this sperm from a male donor and ovum from a female donor are fused in the laboratory. The zygote
(8 blastomere stage) so formed is transferred into the fallopian tube.
GIFT (Gamete Intrafallopian Transfer)- In GIFT, females who cannot produce an ovum, but can provide suitable
conditions for the fertilization of the ovum, are provided with an ovum from a donor.
9- Describe the Lactational amenorrhea method of birth control.
Ans: Lactational Amenorrhea is a natural method of contraception. In lactating mothers there is a complete
absence of menstruation and the chances of conceiving are almost negligible.
10- How are non-medicated IUDs different from hormone-releasing IUDs? Give examples.
Ans. (a) Non-medicated IUDs - Lippes loop, Copper releasing IUDS (CuT, Multiload 375), these increase
phagocytosis of sperms within the uterus and release copper ions which suppress sperm motility and fertilizing
capacity of sperm.
(b) Hormone-releasing IUDs – Progestasert, LNG-20 -These make the uterus unsuitable for implantation and
the cervix hostile to sperm.
Long Answer Type Questions
1- What is MTP? What is its safety concern and in what cases MTPs are allowed in India?
Ans: MTP is Medical termination of pregnancy or induced abortion.
MTPs are considered relatively safe during the first trimester, i.e., up to 12 weeks of pregnancy. Second-
trimester abortions are much more risky
The government of India legalized MTP in 1971 with some strict conditions to avoid its misuse. It is to get rid
of unwanted
pregnancies either due to casual unprotected intercourse or failure of the contraceptive used during coitus or
rape. MTPs are also essential in cases where pregnancy could be harmful or even fatal to the mother/ foetus
or both.
2- Some contraceptives can be taken orally. Give examples of such contraceptives and explain the mechanism
of their action. In what dose it is advised to take by females?
Ans: Oral contraceptives (pills) contain progestogens or progestogen–estrogen combinations.
Pills have to be taken daily for a period of 21 days starting preferably within the first five days of the menstrual
cycle. After a gap of 7 days (during which menstruation occurs) it has to be repeated in the same pattern.
They inhibit ovulation and implantation as well as alter the quality of cervical mucus to prevent retard entry
of sperms.

26
Pills have to be taken daily for a period of 21 days starting preferably within the first five days of the menstrual
cycle. After a gap of 7 days (during which menstruation occurs) it has to be repeated in the same pattern.
Example- Saheli
3- Describe any three types of contraceptive methods widely used by couples to avoid pregnancy.
Ans:
Natural Method
Periodic abstinence couples avoid or abstain from coitus from day 10 to 17 of the menstrual
cycle when ovulation could be expected
Withdrawal or coitus The male partner withdraws his penis from the vagina just before ejaculation to
interruptus avoid insemination
Lactational as long as the mother breast-feeds the child fully, chances of conception are almost
amenorrhea nil (up to 6 months)
IUDs Fitted in the uterus. Increases sperm phagocytosis, Cu ions suppress sperm motility
and the fertilizing capacity of sperms. It also makes the uterus unsuitable for
implantation and the cervix hostile to the sperm.
Non-medicated -Lippes loop
Copper releasing - CuT, Cu7, Multiload 375
Hormone releasing - Progestasert, LNG-20
Injection Progestogens alone or in combination with an estrogen.

4- What are STDs? Give five examples of STDs and also suggest ways to prevent them.
Ans: Diseases or infections which are transmitted through sexual intercourse are collectively called sexually
transmitted diseases (STDs).
Examples- HIV-AIDS, Gonorrhoea, syphilis, genital herpes, chlamydiosis, genital warts, trichomoniasis, and
hepatitis B.
Ways to prevent- (i) Avoid sex with unknown partners/multiple partners. (ii) Always use condoms during
coitus. (iii) In case of doubt, one should go to a qualified doctor for early detection and get complete treatment
if diagnosed with the disease.
5- Reproductive and Child Healthcare (RCH) programs are currently in operation. One of the major tasks of
these programs is to create awareness amongst people about the wide range of reproduction-related
aspects. This is essential for building a reproductive health society.
“Providing sex education in schools is one of the ways to meet this goal.” Give four points in support of
your opinion regarding this statement. Also, List any two indicators that indicate a reproductively
healthy society.
Ans: Sex education is important in schools:
(a) To be aware of myths and misconceptions. (b) For awareness about reproduction.
(c) To be aware of STDs (d) Proper guidance about sex abuse, sex-related crimes, etc.
Indicators about a reproductively healthy society-
(a) Low infant mortality rate (IMR) (b) Low maternal mortality rate (MMR)

27
CBQ
Study the diagram of the female reproductive system given below. Answer the questions
based on the diagram.

(i) What does the diagram depict?


(ii) (ii) At what stage zygote can be introduced in the fallopian tube in Zygote Intra
Fallopian
Transfer (ZIFT)?
(iii) Mention any two events that are inhibited by the intake of oral contraceptive pills to
prevent pregnancy in humans.
Ans.
(i) The diagram depicts the process of vasectomy and tubectomy.
(ii) 8-celled stage
(iii) Two events that are inhibited by the intake of oral contraceptive pills to prevent
pregnancy in humans are ovulation and implantation.
ASSERTION – REASON BASED QUESTIONS
Directions: In the following questions, statement of assertion is followed by a statement of reason.
Mark the correct choice as:
a) If both assertion and reason are true and reason is the correct explanation of assertion.
b) If both assertion and reason are true but reason is not the correct explanation of assertion.
c) If assertion is true but reason is false.
d) If both assertion and reason are false.
1. Assertion: Introduction of sex education in schools should be encouraged.
Reason: Sex education in schools will encourage children to believe in myths about sex related
aspects.
Ans: c)
2. Assertion: Periodic abstinence is a method in which couples avoid coitus from day 17 to 27 of
menstrual cycle.
Reason: Periodic abstinence has limited effectiveness because menstrual cycles are not regular
always.
Ans: d)
3. Assertion: As long as the mother breast- feeds the child fully, chances of conception are almost
nil.
Reason: Lactational amenorrhea method is based on the fact that ovulation does not occur during
the period of intense lactation.
Ans: a)
4. Assertion: In barrier methods ovum and sperms are prevented from physical meetings.
Reason: Barrier methods are used during coitus to prevent the entry of ejaculated semen into the
28
female reproductive tract.
Ans: a)
5. Assertion: Diaphragms, cervical caps and vaults are barriers made of rubber.
Reason: Diaphragms, cervical caps and vaults are inserted into the male reproductive tract during
coitus.
Ans: c)
6. Assertion: Pills are very effective contraceptives with few side effects.
Reason: Pills inhibit ovulation and implantation.
Ans: a)
7. Assertion: Saheli, the new oral contraceptive for the females contains a steroidal preparation.
Reason: Saheli is taken once in a week after initial intake of twice a week dose for 3 months.
Ans: d)
8. Assertion: Sterilisation is a terminal method used only for males.
Reason: Sterilisation is highly effective and its reversibility is very poor.
Ans: d)
9. Assertion: In vasectomy, a small part of the vas deferens is removed or tied up.
Reason: In tubectomy, a small part of the fallopian tube is removed or tied up.
Ans: b)
10. Assertion: Second trimester abortions are much more complicated.
Reason: After 12 weeks, the foetus becomes intimately associated with the maternal tissues.
Ans: a)
11. Assertion: Syphilis, gonorrhoea and AIDS are STDs.
Reason: Syphilis, gonorrhoea and AIDS are transmitted through sexual intercourse.
Ans: a)
12. Assertion: Infertility is the inability to produce children inspite of unprotected sexual cohabitation.
Reason: Infertile couples could have children using assisted reproductive technologies (ART).
Ans: b)
13. Assertion: IVF is fertilisation outside the body of woman.
Reason: The zygote upto 8 blastomeres could be transferred into the fallopian tube.
Ans: b)
14. Assertion: IUT is transfer of embryo with more than 8 blastomeres into the fallopian tubes.
Reason: IUT is a very popular method of forming embryos in vitro.
Ans: d)
15. Assertion: Intra cytoplasmic sperm injection ( ICSI) is a procedure to form an embryo in vitro.
Reason: in ICSI sperm is directly injected into the ovum.
Ans: b)
16. Assertion: A person should be considered reproductively healthy if they have healthy
reproductive organs but are emotionally imbalanced.
Reason: This statement about reproductive health was given by WHO.
Ans: d)
17. Assertion: Family planning is an action plan to attain reproductive health among people.
Reason: Improved programmes covering reproduction related areas were propagated by RCH to
create awareness among people.
Ans: b)
18. Assertion: Reproductive and Child Healthcare Programmes is for reproduction related areas.
Reason: It deals with creating awareness among various reproduction related aspects.
Ans: a)
19. Assertion: A wide range of contraceptive methods are available for family planning.
Reason: Natural method includes condoms, diaphragms, etc., while barrier methods use of included
method like periodic abstinence, actational amenorrhea, etc.
Ans: c)
20. Assertion: Introduction of sex education in schools should be encouraged.
Reason: This will encourage children to believe in myths about sex related aspects.
Ans: c)
29
21. Assertion: Amniocentesis is often misemployed.
Reason: Amniocentesis is meant for determining the genetic disorders in the foetus, but it is being
used to determine the sex of the foetus, leading to death of the normal female foetus.
Ans: a)
22. Assertion: Natality increases both population density and population size.
Reason: Natality increases the number of individuals in an area by births.
Ans: a)
23. Assertion: Saheli, the new oral contraceptive for the females, contains a steroidal preparation.
Reason: It is “once in a day” pill with very few side effects.
Ans: d)
24. Assertion: Rapid decline in death rate, MMR and IMR have lead to a staggering rise in
population.
Reason: Such an alarming growth rate has lead to an absolute scarcity of even the most basic
requirements, i.e. food and shelter.
Ans: b)
25. Assertion: Zero population growth should be achieved as early as possible to control human
population.
Reason: This as requires not two children per couple but a little more.
Ans: a)
26. Assertion: Periodic abstinence is a method in which couples avoid from coitus from day 17 to 27
of menstrual cycle.
Reason: It is a very effective method and 100% sure of birth control.
Ans: d)
27. Assertion: There is chance of fertilisation during 10 -17 days of menstrual cycle.
Reason: Ovulation occurs during these days.
Ans: a)
28. Assertion: Contraceptives are methods to prevent unwanted pregnancies.
Reason: Unwanted pregnancies can only be prevented by using oral contraceptives.
Ans: c)
29. Assertion: Barrier methods prevent physical meeting of sperms and ova.
Reason: This prevents conception.
Ans: a)
30. Assertion: Reusable contraceptives are not full proof method of contraceptions.
Reason: Diaphragms, cervical caps and vaults are barrier methods which prevent conceptions by
blocking entry of sperms through cervix. They are reusable.
Ans: b)

Important You tube link to understand contents--


https://youtube.com/playlist?list=PLMQhn0MTk6aLevo-Yw3wDG-rW6XtaIWMd

2
10
CHAPTER – 5
PRINCIPLES OF INHERITANCE AND VARIATION
Keywords
Genetics Study of inheritance, heredity and variation of characters or Study of genes and
chromosomes.
Inheritance Transmission of characters from parents to progeny.
Variation Difference between offspring and parents
Gene It is a short fragment of DNA that encodes for a certain trait
Allele An allele is a variant form of a gene
Homozygous Individuals carrying two identical alleles (RR or rr) are known as homozygous.
Heterozygous individual organisms bearing different alleles (Rr) are known as heterozygous
Hybrid An individual produced by the mating of genetically unlike parents
Mendelian factors Genes
Dominance It can express in both homozygous and heterozygous condition
Recessive It can express only in homozygous
F Filial generation
Test cross Cross between offspring and recessive parent
Back cross It is the crossing of a hybrid with one of its parents or an individual genetically similar to its
Parent
Codominance Codominance is a heterozygous condition in which both alleles at a gene locus are fully
expressed in the phenotype
Incomplete Both alleles of a gene at a locus are partially expressed, often resulting in an intermediate or
dominance different phenotype.
Multiple allelism occurrence of three or more three alleles for a particular gene
Linkage Tendency of closely situated genes to inherit together
Crossing over It is an exchange of genetic material during sexual reproduction
Genotype The specific combination of alleles for a given gene
Phenotype An individual's observable traits
Punnet square Square diagram is used to predict the genotypes of a particular cross
Monohybrid Offspring with one particular character
Dihybrid Offspring with two different character
Polygenic The inheritance of a trait governed by more than one gene
inheritance
Pleiotropy In which a single gene has multiple phenotypic expressions
Mutation A change in the DNA sequence of an organism
Sex-linked disorder Inherited through sex chromosomes ( X or Y chromosomes)
Autosome-linked It is the pattern of inheritance in which the transmission of traits depends on the genes in
disorder the autosome
Chromosomal A disorder due to a change in the chromosome number
Disorders
Aneuploidy Aneuploidy is a type of chromosomal aberration, where there is one extra chromosome or
one missing chromosome
Polyploidy cells of an organism have more than one pair of chromosomes.
Syndrome A syndrome is a set of medical signs and symptoms that are correlated with each other
and often associated with a particular disease or disorder.

2
11
MENDEL’S PRINCIPLES OR LAWS OF INHERITANCE

Dihybrid Ratio

• Genotype ratio: -9:3:3:1


• Phenotypic ratio:- 9 Round yellow: 3 Round green: 3 Wrinkled yellow: 1 Wrinkled green

INCOMPLETE DOMINANCE/ CODOMINANCE/ MULTIPLE ALLELISM.

2
12
Thomas Hunt Morgan
 Worked on fruit Reason of working on Drosophila
flies Drosophila  They could be grown on simple synthetic medium
melanogaster  They complete their life cycle in about two weeks
 He proposed  Single mating could produce a large number of progeny
the linkage  Very clear differentiation of male and female
theory

LINKAGE and CROSSING OVER

2
13
Alfred Sturtevant used the frequency of recombination between gene pairs on the same chromosome as
a measure of the distance between genes and ‘mapped’ their position on the chromosome.
POLYGENIC INHERITANCE and PLEIOTROPY

SEX DETERMINATION

MENDELIAN DISORDER

• Mendelian Disorders can be defined as a type of genetic disorder that arises due to alterations in one gene
or as a result of abnormalities in the genome.

30
• Examples: Colour Blindness, Haemophilia, Thalassemia

CHROMOSOMAL DISORDERS

IMPORTANT QUESTIONS
VERY SHORT ANSWER QUESTIONS/ MCQ
1- Identify the name/ names of the scientists who rediscovered Mendel’s work.
a- Hugo DeVries
b- Carl Correns
c- Tschermak
d- All of these
Ans: D
2- What is the karyotype of Klinefelter’s Syndrome and Turner’s syndrome?
a- 45 X0
b- 47 XYY
c- 47 XXY
d- 45 XXY
Ans: 47, XXY and 45 with X0
3- Which of the following represents the XO type of sex determination?
a- Grasshopper
b- Lizard
c- Huma
d- Earthworm
Ans: Grasshopper
4- Write the Dihybrid cross ratio when self-cross is carried out between two heterozygous gametes.
a- 1:2:1
b- 9:3:3:1

3
1
c- 1:1:1:1
d- None of these
Ans: A
5- For which types of reproduction Mendel's laws of inheritance are applicable?
a- Asexual Reproduction
b- Sexual Reproduction
c- Both asexual and sexual reproduction
d- None of these
Ans: B
6- Who observed that during spermatogenesis only 50 % of insect sperm received a specific structure, whereas
50 % of sperm did not receive it? A,lso mention the name of that specific structure.
Ans: Henking, X-body
7- What is the cross known as when the F1 progeny is crossed with a homozygous recessive parent? Write one
advantage of suca h cross.
Ans: The cross is a test cross.
It is advantageous to determine the genotype of the parent plant.
8- Name any one plant that shows the phenomenon of incomplete dominance during the inheritance of its
flower colour.
Ans: Dog flower (Snapdragon or Antirrhinum sp.)
9- What term used in which there are extra set of chromosomes is presen?.
Ans: aneuploidy
10- A haemophilic man marries a normal homozygous woman. What is the probability that their daughter will
be haemophilic?
Ans: no chance (0%), she will be carrier only
SHORT ANSWER TYPE QUESTIONS

1- In a plant 2n= 12 and another plant= 20. What will be the ploidy of plants obtained by crossing these
plants?
Ans- 2n = 12 then gamete (n)= 6
2n = 20 then gamete (n)= 10
Progeny = n+n = 6+ 10 = 16
2- What observations we can get from the law of dominance.
Ans: The law of inheritance provides vide knowledge The law of dominance is used to explain the expression
of only one of the parental characters in a monohybrid cross in the F1 and the expression of both in the F2. It
also explains the proportion of 3:1 obtained at the F2
3- The human male never passes on the gene for haemophilia to his son. Why is it so?
Ans: haemophilia is sex (X) linked recessive disorder. The male can transfer only ‘Y’ chromosome to his son.
4- The child has a blood group of O. If the father has blood group A and the mother has blood group B, work
out the genotypes of the parents and the possible genotypes of the Offspring.
Ans: Genotype of father IAIA
Genotype of mother IB IO
Genotype of child IOIO

3
2
5- In Snapdragon, a cross between true-breeding red-flowered (RR) plants and true-breeding white-flowered
(RR) plants showed a progeny of plants with all pink flowers.
(i) The appearance of pink flowers is not known as blending. Why?
(ii) What is this phenomenon known as?

Ans: i- This is not blending as neither of genes can express in heterozygous condition but can express when
they are in homozygous condition.
Ii-Incomplete dominance.
6- What are the possible effects of modified alleles?
Ans: (i) the normal/less efficient enzyme, or
ii- a non-functional enzyme, or
iii- no enzyme at al
7- Differentiate between co-dominance and incomplete dominance.
Ans: Co-dominance –condition when phenotypes of all the alleles are expressed.
Incomplete dominance- When both alleles of a gene at a locus are partially expressed and produce an
intermediate phenotype, such phenomenon is called incomplete dominance
8- Describe multiple allelism by citing one example.
Ans: when a single trait is controlled by more than 2 alleles, it is called a multiple allelism.
Example: Human blood group is controlled by three alleles IA,IB and i.
9- Explain the cause of Klinefelter’s syndrome. Give any three symptoms shown by the sufferer of this
syndrome.
Ans. Caused due to the presence of an additional copy of X chromosome resulting in a karyotype of 47, XXY.
Features- individual has overall masculine development, however, the feminine development (development
of breast, i.e., Gynaecomastia) is also expressed. Such individuals are sterile.
10- How does distance between genes affect linkage and crossing over?
Ans: (a) closely situated genes show more linkage and less crossing over
(b) Distantly located gene shows more crossing over and less linkage.

Very Long Answer Type Questions

1- Discuss in detail the contributions of Morgan and Sturtevant in the area of genetics.
Ans:
i- Morgan worked on fruit fly (Drosophila melanogaster).
ii- He stated and established that genes are located on the chromosome.
iii- He established the principle of linkage.
iv- Established the technique of chromosome mapping.
Sturtevant was a student of Morgn .His contributions are-
i- He constructed the first genetic map of a chromosome while working on the Drosophila genome.
ii- Established concept of the linkage group.
2- Diagrammatically represent results of a dihybrid cross where the two parents differed in two pairs of
contrasting traits: seed colour and seed shape of the pea plant.
Ans: 5.7, page 79, NCERT
3
3
3- What is recombination? Mention its applications with reference to genetic engineering.
Ans: Recombination is the process of producing a new combination of genes by crossing over during meiosis.
It is the characteristic feature of sexually reproducing organisms.
Applications:
i- It causes variation in a population.
ii- Variability leads to better adaptation and survival
iii- With the help of linkage groups, chromosome map can be prepared.
iv- The desired recombinants can be produced.
4- A cross between a red flower-bearing plant and a white flower-bearing plant of Antirrhinum produced all
plants having pink flowers. Work out across, to explain how is this possible?
Ans: It is due to incomplete dominance. In this both alleles of a gene at a locus are partially expressed and
produce an intermediate phenotype

5- A dihybrid heterozygous round, yellow seeded garden pea (Pisum sativum) was crossed with a double
recessive plant.
a- What type of cross is this?
b- Workout on the genotype and phenotype of the progeny.
c- What principle of Mendel is illustrated through the result of this cross?

Ans:
a- It is a dihybrid test cross
b- It illustrates the Principle of independent assortment.

Assertion and Reasoning


Read the following Assertion and Reason based questions and select the most appropriate answer
for the questions:-
➢ Assertion and reason both are correct, and reason is correct explanation of the assertion.
➢ Assertion and reason both are correct, and reason is not correct explanation of the assertion.
➢ Assertion is correct and reason is incorrect.
➢ Assertion is incorrect and reason is correct.
1. Assertion: - The point mutation is the substitution or replacement of a single nucleotide from
DNA.
Reason: - Sickle cell anaemia caused due to point mutation.
2. Assertion: - Colour Blindness caused due to a recessive gene, which is found in X
chromosome.
Reason: - Colour blindness is an example of X linked Recessive disease.
3. Assertion: - there are three pairs of alleles, which responsible to control the human skin colour.
3
4
Reason: - The inheritance of human skin colour called as Polygenic Inheritance.
4. Assertion: - The non-disjunction of the homologous chromosome, is resulting as non-proper
distribution of the chromosomes.
Reason: - Down‘s Syndrome disease is caused due to the non-disjunction of the chromosomes.
5. Assertion: - There are triple alleles, IA, IB, I responsible to control the blood group of human being.
Reason: - The controlling of one trait by number of alleles is called as multiple allelism.
6. Assertion :-when a pure red flowered and pure white flowered , dog flower plants are crossed
together, pink flowered plants are produced in f-1 generation.
Reason: - this is the incomplete dominance, which create the pink colour of the flowers.
7. Assertion: - if the genotype is ―Tt‖, the phenotype of the pea plant is tall.
Reason: - Mendel‘s first law ― law of Dominance ― work to create phenotype in the
heterozygous genotype.
8. Assertion: - The life span of Drosophila, is about 2 weeks.
Reason: - for the linkage, T. H. Morgan selected, Drosophila as an experimental insect.
9. Assertion: - Alfred Sturtevant, used the frequency of recombination, to measure the distance
between genes.
Reason:- more frequency of recombination means , genes are located farther , low frequency of
recombination means genes are located nearer.
10. Assertion:-In fowls, the female has ZW and male has ZZ sex chromosome.
Reason: - the sex determination in the fowls is done by the female, not by the male parent.
11. Assertion: - The male honey bees or the drones produced by parthenogenesis.
Reason: - Male honey bees perform mitosis during Gametogenesis.
12. Assertion: - In human female, XX is the sex chromosomal configuration.
Reason: - The determination of the sex is done by both the parents.
13. Assertion: - in phenylketonuria, phenyl alanine is excreted by help of urine.
Reason: - Phenyl alanine has poor absorption, by the kidney.
14. Assertion: - The possibility of a female becoming a haemophilic is extremely rare.
Reason: - mother must be at least carrier and father must be affected by the disease.
15. Assertion: - beta thalassemia, production of beta chain affected.
Reason: - it caused due to mutation in one or both genes on chromosome no. 16.
16. Assertion: - Chromosomal disorders can be classified into aneuploidy or polyploidy.
Reason: - Chromosomal disorders can be caused either gaining of extra copy number of
chromosomes or an increase in a whole set of chromosome.
17. Assertion: - A male child cannot be affected by colour-blindness.
Reason:-mother is a carrier for colour blindness.
18. Assertion: - a female individual has rudimentary or non-functional ovaries.
Reason:-Sterility of the female caused due to the Turner‘s Syndrome.
19. Assertion: - Genes and chromosomes have parallel behaviour.
Reason: - Sutton &Boveri introduced chromosomal theory of inheritance to prove it.
20. Assertion: - The work of Mendel,remain, unrecognized till 1900.
Reason: - Expression of the traits, did not blend with each other, was not accepted by his
contemporaries

3
5
Cased based questions: Study the flowchart given below and answer the questions that
follows

(i) What is a mutagen? Name a physical factor that can be mutagen.


(ii) What is point mutation? Give one example.
(iii) Mention two causes of frame-shift mutation.
Answer :
(i) All the physical and chemical factors that induce mutation are called mutagens. UV radiation and
X-rays are physical mutagens.
(ii) Mutation arising due to change in a single base pair of DNA is called point mutation.
(iii) Insertion and deletion of three bases or multiples of three bases cause frame-shift mutation
because the reading frame remains unaltered from that point onwards
Case Study 2:
Non-disjunction is the failure of homologous chromosomes to disjoin correctly during meiosis. It leads
to the formation of a new cell with an abnormal amount of genetic material. A number of clinical
conditions are the result of this type of chromosomal mutation. This results in the production of
gametes containing a greater or lesser chromosomal amount than normal ones. Consequently, the
individual may develop a trisomy or monosomal syndrome. Nondisjunction can occur in both Meiosis -
I and Meiosis- II of the cellular division. It is also the main cause of many genetic disorders; however,
its origin and process remain vague. Although it results in the majority of cases from errors in
maternal meiosis II, both paternal and maternal meiosis I do influence it. Maternal age is considered a
risk factor for trisomy, as well as recombination alterations and many others that can affect
chromosomal segregation.
Q1. Which of the following conclusions can be true regarding aneuploidy?
I. It is the presence of an extra chromosome in a diploid cell.
II. An aneuploid cell differs from other cells only in size.
III. It can be less number of chromosomes in a diploid cell.
IV. Aneuploidy always affects female individuals.
a) i only b) both i and iii c) both ii and iii d), iii and iv
Ans: b) both i and ii

3
6
By interpreting the graph of Down syndrome frequency and mothers’ age, select the best
conclusion(s) from the following options.
I. Aneuploidy is not influenced by the mother’s age.
II. Delivery before 30 years of age can decrease the incidence of aneuploidy in
a. most cases
III. The chance of aneuploidy increases up to 22 years of age.
IV. There is a dramatic increase in aneuploidy if the maternal age exceeds 30

a. I only
b. Both I and iii
c. both ii and iv
d. I, iii and iv
Answer: C
Case Study 3:
Two blood samples of suspects ‘A’ and ‘B’ were sent to the Forensic Department along with sample ‘C’
from the crime scene. The Forensic Department was assigned the responsibility of running the
samples and matching the samples of the suspects with that of the sample from the scene of the
crime and thereby identifying the culprit.

3
7
Q1: In genetic fingerprinting, the ‘probe’ refers to –
a. A radioactively labelled double stranded RNA molecule.
b. A radioactively labelled double stranded DNA molecule.
c. A radioactively labelled single stranded DNA molecule.
d. A radioactively labelled single stranded RNA molecule.
Answer: C
Important You tube link to understand contents--
https://youtube.com/playlist?list=PLMQhn0MTk6aLevo-Yw3wDG-rW6XtaIWMd

3
8
CHAPTER- 6

MOLECULAR BASIS OF INHERITANCE

Keywords

DNA Deoxyribonucleic acid


RNA Ribonucleic acid
Phosphodiester bond The bond between two nucleotide
Ribose sugar Pentose sugar
Histone Positively charged proteins
Nucleosome Histone octamer + DNA
Chromatin Chromatin is a complex of DNA and protein found in eukaryotic cells
Euchromatin Loosely packed chromatin, transcriptionally active
Heterochromatin Densely packed chromatin, transcriptionally inactive
Transformation Transfer of DNA into bacteria
Replication Synthesis of new DNA from parental DNA
Transcription Synthesis of RNA from DNA
Translation Protein synthesis
Ribosome Organelles for protein synthesis
Genetic code Set of codons
Operon Set of genes that regulate gene expression
VNTR Variable number of tandem repeats
Cistron As a segment of DNA coding for a polypeptide
Exons Coding sequence on mRNA
Introns Intervening sequences do not appear in mature or processed RNA
Splicing Mechanism of removal of introns and ligation of exons.
POLYNUCLEOTIDE CHAIN
✓ DNA / RNA both are made of polynucleotide chains.
✓ In polynucleotides the nucleotides are joined with a
Phosphodiester bond.

DNA

✓ Friedrich Miescher in 1869 identify DNA as an acidic substance present in the nucleus and named it ‘Nuclein’

Organism Length of DNA


Bacteriophage φ ×174 5386 nucleotides
Bacteriophage lambda 48502 bp
Escherichia coli 4.6 × 106 bp
Human (haploid content) 3.3 × 109 bp

3
9
Double Helical Model of DNA

Erwin Chargaff
He found that in DNA, the ratios of adenine (A) to thymine (T) and guanine (G) to cytosine (C) are equal.
A= T and G=C
[A] + [G] = [T] + [C] or [A] + [G] / [T] + [C] =1

CENTRAL DOGMA

✓ Proposed by Francis Crick


✓ Genetic information flows from DNA to RNA to Protein
✓ The process of making protein from DNA is known as the “central dogma”

PACKAGING OF DNA HELIX

THE SEARCH FOR GENETIC MATERIAL

▪ Transforming principle- Proposed by Frederick Griffith in 1928


▪ Biochemical characterization of transforming principle- proposed by Oswald Avery, Colin Macleod &
Maclyn McCarty
▪ Hershey-chase experiment- proposed by Alfred Hershey and Martha Chase (1952)

3
10
PROPERTIES OF GENETIC MATERIAL (DNA VERSUS RNA)

(i) It should be able to generate its replica (Replication).


(ii) It should be stable chemically and structurally.
(iii) It should provide the scope for slow changes (mutation) that are required for evolution.
(iv) It should be able to express itself in the form of 'Mendelian Characters’.
DNA is more stable than RNA because –

DNA is double-stranded The mutation rate in RNA is more than in DNA


Thymine in DNA is less reactive than The absence of 2’-OH in sugar in DNA makes it less
Uracil in RNA Reactive

RNA WORLD

▪ RNA was the first genetic material.


▪ Essential life processes (such as metabolism, translation, splicing, etc.), evolved around RNA.
▪ RNA used to act as a genetic material as well as a catalyst.
▪ RNA can directly code for the protein synthesis, hence can easily express the characters. DNA is dependent
on RNA for protein synthesis

TYPES OF RNA
mRNA It contains messages of DNA in the form of codons.
Eukaryotic mRNA undergoes splicing before entering the translation process.
Prokaryotic mRNA does not need to be processed.
rRNA rRNA forms ribosomes along with protein.
The ribosomes site to bind aminoacyl-tRNAs and link amino acids together to create
polypeptides.
tRNA It is also known as adapter RNA.
They have a cloverleaf structure that consists of a 3’ acceptor site, 5’ terminal phosphate, D
arm, T arm, and anticodon arm.
The primary function of a tRNA is to carry amino acids to the ribosome.

3
11
REPLICATION, TRANSCRIPTION AND TRANSLATION

Complexities in Eukaryotic Transcription

i- Three different RNA Polymerases

RNA polymerase I Transcribes rRNAs (28S, 18S, and 5.8S),


RNA polymerase II Transcribes mRNA, heterogeneous nuclear RNA (hnRNA)
RNA polymerase III Transcribes of tRNA, 5srRNA, and snRNAs
ii- Splicing Mechanism
Primary transcripts contain both the exons and the introns. Introns are removed by splicing process and
exons are joined in a defined order.
iii- Capping and Tailing- In capping an unusual nucleotide (methyl guanosine triphosphate) is added to the
5'-end of hnRNA. In tailing, adenylate residues (200-300) are added at 3'-end. It is the fully processed
hnRNA, now called mRNA that is transported out of the nucleus for translation

GENETIC CODE

Genetic code is the sequence of codons on m RNA that contain information about amino acids.
The salient features of the genetic code are as follows:
• The codon is a triplet. 61 codons code for amino acids and 3 codons do not code for any amino acids, hence
they function as stop codons.
• Some amino acids are coded by more than one codon, hence the code is degenerate.
• The codon is read in mRNA in a contiguous fashion. There are no punctuations.
3
12
• The code is nearly universal e.g. from bacteria to humans UUU would code for Phenylalanine. Some
exceptions to this rule have been found in mitochondrial and protozoan codons.
• AUG has dual functions. It codes for Methionine (met) , and it also acts as an initiator codon.
• UAA, UAG, UGA are stop terminator codons.

REGULATION OF GENE EXPRESSION


In eukaryotes, the regulation is controlled at-
o Transcriptional level (formation of primary transcript),
▪ Processing level (regulation of splicing)
• Transport of mRNA from nucleus to the cytoplasm,
o Translational level.
THE LAC OPERON

• Francois Jacob and Jacque Monod proposed Lac operon for lactose metabolism in E. coli
• Genes on Lac Operon
Regulator gene (i) Codes for the repressor of the lac operon.
Promotor (p) It is binding site for RNA Polymerase
operator gene (o) It is the binding site for the repressor
structural genes (z, y, a) The z gene (beta-galactosidase (β-gal) –breakdown lactose into
galactose and glucose.
The ‘y’ gene (permease)- increases the permeability of the cell to β-
galactosidase.
The ‘a’ gene encodes a transacetylase.

Lactose is the substrate for the enzyme beta-


galactosidase and it regulates switching on and
off of the operon. Hence, it is termed as an
inducer.

When lactose is absent the regulator gene


synthesizes repressor protein. The repressor
protein binds on operator and prevents RNA
polymerase from transcribing the operon.

When lactose is present the repressor is


inactivated by interaction with the inducer. This
allows RNA polymerase access to the promoter
and transcription proceeds.

3
13
HUMAN GENOME PROJECT
It is an international scientific project with the goal of determining the sequence of nucleotide base pairs human
genome and its mapping.
Goals of HGP Methodologies

SALIENT FEATURES OF THE HUMAN GENOME PROJECT


DNA FINGERPRINTING
• DNA fingerprinting is a technique of determining nucleotide sequences of certain areas (VNTRs) of DNA that
are different in different individuals. DNA fingerprinting involves identifying differences in repetitive DNA.
• Alec Jeffreys develops the technique of DNA Fingerprinting. He used satellite DNA as a probe that shows a
very high degree of polymorphism called as VNTRs (as Variable Number of Tandem Repeats)
• These sequences with a high degree of polymorphism form the basis of DNA fingerprinting.
• The repetitive DNA is separated from bulk genomic DNA as different peaks during density gradient
centrifugation. The bulk DNA forms a major peak and the other small peaks are referred to as satellite DNA
(such as micro-satellites, mini-satellites, etc.)
• DNA Polymorphism (variation at the genetic level) arises due to mutations.
• It is widely used in forensic science, paternity disputes, etc.

Steps of DNA Fingerprinting

➢ Isolation of DNA
➢ Digestion of DNA by restriction endonucleases
➢ Separation of DNA fragments by electrophoresis
➢ Transferring (blotting) separated DNA fragments to synthetic membranes, such as nitrocellulose or nylon
➢ Hybridization using labeled VNTR probe
➢ Detection of hybridized DNA fragments by autoradiography

IMPORTANT QUESTIONS

Very Short Answer Type Questions/ MCQ

1- By which method detection of hybridized DNA fragments is possible-


a- Electrophoresis
b- Southern blotting
c- Northern blotting
d- Autoradiography
Ans: D
2- Who invented methodologies of automatic DNA sequencing and Lac operon?
a- Sanger
b- Jeffry
40
c- Nicholson
d- Both a and b
Ans: D
3- Give the name of commonly used vectors in the human genome project.
a- BAC
b- YAC
c- Both a and b
d- None of these
Ans: C
4- What is the basic approach adopted as a methodology in HGP?
a- ESTs
b- Splicing
c- Tagging
d- Emasculation
Ans: A
5- met tRNA met is an example of-
a- AUG
b- GUG
c- Activated tRNA
d- Activated mRNA
Ans: c
6- What are inducers in lac operon, give two examples of inducers.
Ans: inducer binds with repressor protein so that they can’t bind on operator gene.
Example- Lactose and Allolactose
7- What are untranslated regions (UTR)? Give their importance.
Ans: The UTRs are present at both 5' -end (before start codon) and at 3' -end (after stop codon). They
are required for an efficient translation process.
8- What are initiator tRNA? How many tRNAs are present in a cell for 20 different amino acids?
Ans: Initiator tRNA is for initiation of the translation process. It always carries methionine amino acid.
There are 20 different tRNAs for 20 different amino acids.
9- What is the dual nature of codon AUG?
Ans: It codes for the amino acid methionine.
This is also known as the initiation codon responsible for the initiation of the translation process.
10- What is the role of the sigma factor and DNA ligase?
Ans: Sigma factor initiates the process of transcription
DNA ligase is to join adjacent nucleotides.
11- Name the bacteria used by Fredrik Griffith in his experiment. Also, mention the types of strain he used to
prove the nature of genetic material.
Ans: Bacteria- Streptococcus pneumonia
Strain- S strain (virulent) R strain (non-virulent)

Short Answer Type Questions

1- The sequence of the coding strand of DNA in a transcription unit is mentioned below
3′ TATAGCATCTATTAGG 5′
Write the sequence of RNA formed on this DNA.
Ans: 3′ UAUAGCAUCTAUUAGG 5′
2- Three codons on mRNA are not recognized by tRNA during the translation process. Mention these codons?
Write the importance of these in protein synthesis.
Ans: Stop codons- UAA, UAG and UGA
Help in termination of the translation process and release of polypeptide into cytoplasm
3- Label the parts ‘A’ and ‘B’ of the transcription unit given below

Ans: A- Promoter B- Coding strand

4- Describe any three steps required for the maturation of nascent mRNA into the cell.
Ans: Splicing- Primary transcripts contain both the exons and the introns. Introns are removed by splicing
process and exons are joined in a defined order.
Capping- addition of unusual nucleotide (methyl guanosine triphosphate) at 5’ end (capping)
Tailing- adding polyadenylate residue at 3’ end.
5- List the features which must be present in good genetic material.
Ans: (i) It should be able to generate its replica (Replication). (ii) It should be stable chemically and structurally.
(iii) It should provide the scope for slow changes (mutation) that are required for evolution. (iv) It should be
able to express itself in the form of 'Mendelian Characters’.
6- Describe the role of the following-
Sigma factor, Rho factor, Release factor, Histone protein
Ans:
▪ Sigma factor- initiate transcription process
▪ Rho factor- termination of the transcription process
▪ Release factor- helps in the termination of translation and dissociation of ribosome subunits,
the release of the polypeptide
▪ Histone Protein- help in the formation of nucleosome
7- Give two reasons why both the strands of DNA are not copied during the transcription process.
Ans: 1- If the strands code for RNA, two different RNA molecules & two different proteins are formed.
2- The two RNA molecules if produced simultaneously would be complementary to each other, hence would
form a double-stranded RNA. This would prevent RNA from being translated into protein.
8- RNA was the first genetic material, DNA evolved later on. Explain.
Ans: RNA can directly code for the synthesis of proteins, and hence can easily express the characters.
RNA is used to act as a genetic material as well as a catalyst.
Essential life processes like metabolism, translation, splicing, etc. evolved around RNA.
9- Briefly describe the termination of a polypeptide chain.
Ans: When the ribosome reaches the Stop codons (UAA, UAG, and UGA) on mRNA, RF Protein (Release factor)
binds on it and dissociates subunits of the ribosome, and the polypeptide is released in the cytoplasm.
10- List two essential roles of ribosomes during translation.
Ans: i- provide an attachment site for mRNA
ii) Provide the site of polypeptide synthesis

Long Answer Type Questions

1- a- What is DNA fingerprinting?


b- What are the steps of DNA fingerprinting?
C-Mention its application
Ans: a- DNA fingerprinting is a technique of determining nucleotide sequences of certain areas (VNTRs) of DNA
which are different in different individual
b- 1- Isolation of DNA 2- Digestion of DNA by restriction endonucleases 3- Separation of DNA
fragments by electrophoresis 4- Transferring (blotting) of separated DNA fragments to synthetic
membranes, such as nitrocellulose or nylon 5- Hybridization using labeled VNTR probe 6- Detection
of hybridized DNA fragments by autoradiography
c- Paternity disputes can be solved by DNA
fingerprinting It is useful in the detection of
crime and legal pursuits.
2- a- Differentiate between the genetic codes given
below:
(i) Unambiguous
(ii) Degenerate
b- What is the significance of the Severo Ochoa enzyme?
Ans: a- i- Unambiguous: one Condon code for only one amino acid.
Ii-Degenerate: When an amino acid is coded by more than one codon
b- Severo Ochoa enzyme (polynucleotide phosphorylase) is useful in polymerizing RNA with
defined sequences in a template-independent manner (enzymatic synthesis of RNA)
3- i- Write the stages at which the Regulation of gene expression can be achieved in eukaryotes.
ii-What is the meaning of I, p, and o in operon?
Ans: i- transcriptional level (formation of primary transcript), processing level (regulation of splicing),
transport of mRNA from the nucleus to the cytoplasm, translational level.
ii-Regulator gene (i) Codes for the repressor of the lac operon, promoter (p) is the binding site
for RNA Polymerase and operator gene (o) It is the binding site for the repressor.
4- i- Why is DNA molecule a more stable genetic material than
RNA? Explain ii-Draw a double helical diagram of DNA showing
phosphodiester bonds. Ans: the hydrogen between purine and
pyrimidine provides stability to DNA. The DNA lacks a 2’OH group
which makes it less reactive than RNA.
The thymine in DNA is less reactive than Uracil
in RNA. RNA mutates easily.
ii-Fig6.2, page 98 NCERT
5- Provide any five silent features of human genome.
Ans: The human genome contains 3164.7 million bp., The total number of genes is estimated at
30,000, Almost all (99.9 percent) nucleotide bases are exactly the same in all people, The functions
are unknown for over 50 percent of the discovered genes, and Less than 2 percent of the genome
codes for proteins, Chromosome 1 has most genes (2968), and the Y has the fewest (231), there are
about 1.4 million locations where single base DNA differences (SNPs – single nucleotide
polymorphism : ‘snips’) occur in humans.

MULTIPLE CHOICE QUESTIONS


1 Nucleic acids were first isolated by
a)Kornberg
b) Meischer
c)Altman
d)Mendel
2 How many base pairs of DNA helix are contained in 10 nucleosomes?
a)4000
b)3
000
c)20
00
d)2
00
3 Ribozymes are
a)23s r RNA
b) 18s r RNA
c) 5.8s r RNA
d) 5s r RNA
4 RNA polymerase binds to the -----gene
a)Regulator
b)Operator
c)Structural
d)Promotor
5 Methionine and Tryptophan has ------ triplet codon
a) Single
b)Double
c)Tripple
d)Quadrat
e
6 In some viruses, DNA is synthesised by using RNA as template. Such a DNA is called
(a) A – DNA
(b) B – DNA
(c) cDNA
(d) rDNA.
7 If a double stranded DNA has 18% of cytosine, what will be the percentage of adenine in it ?
(a) 18%
(b) 40%
(c) 32%
(d) 64%
8 Find out the number of base pairs in E. coli DNA,if its DNA is
1.36mm long
a)400000
0bp
b)200000
0bp
c)3000000
bp
d)600000
0bp
9 Human genome has 3x109bp.99.9% of base sequence among the humans are same. How many base
sequence would be the different in two individuals.
a)3x105b)3x10
6

c)3x107d)3x10
8

10 In DNA if adenine is Apple, Guanine is grape, Cytosine is Citrus and Thymine is Tamarind.
Which of the following option is true ?
a)Apple +Citrus= Citrus +Grapes
b) Apple+ Tamarind =Grapes+ Apple
c)Apple +Grapes= Tamarind +Citrus
d)Apple+Tamarind =Grapes+Citrus
11 Shape of t-RNA is ---shaped
I.Y II.L III.Z IV. Clover leaf
a)
I,II
b)II,
IV
c)II,I
II
d)I,I
V
12 Raj is studying certain organisms and found its base compositionto be A=22%,T=28% ,G=20%and
C=30%. What will be his conclusion regarding it’s genetic material ?
a) single stranded RNA
b) single stranded DNA
c) double stranded RNA
d) double stranded DNA
13 If a length of a DNA has 45,000 base pairs,how many complete turns will the DNA molecule take ?
a)45
b)450
c)4500
d)45,0
00
14 t-RNA molecule Anticodon pairing
I UCA
II AUG
III AUU
IV AUC
Which of the following t RNA molecule does not exist ?
a)I and II
b) II and IV
c) II and III
d) III and IV
15 VNTR varies in size from
a)0.1-20kb
b)0.2-
10kb
c)0.3-
30kb
d)0.4-
40kb
16 Nucleic acids are polymer of
(a) Nucleosides
(b) Proteins
(c) Glycoproteins
(d) Nucleotides
17 Synthesis of any protein in a cell is determined by
(a) Types of ribosomes
(b) Mitochondria
(c) Sequence of nucleotides in DNA
(d) Sugar and phosphate of DNA
18 In DNA replication, the primer is
(a) A small deoxyribonucleotide polymer
(b) Small ribonucleotide polymer
(c) Helix destabilizing protein
(d) Enzyme taking part in joining nucleotides to their complementary template bases

19 Which of the following combination is a correct observation for the transformation experiment
performed by Griffith?
a) Type IIIS (living) + mouse = dead
b) Type IIIS (heat killed) + mouse = dead
c) Type IIR (living) + mouse = dead
d) Type IIIS (heat killed) + type IIR (living) + mouse = living
20 The central dogma of molecular biology (genetic information flow) was modified by the discovery of
(a) RNA polymerase
(b) DNA ligase
(c) Reverse transcriptase
(d) DNA polymerase
21 The human chromosome with the highest and least number of genes in them are respectively [NCERT
Exemplar]
(a) Chromosome 21 and Y
(b) Chromosome 1 and X
(c) Chromosome 1 and Y
(d) Chromosome X and Y
22 The net electric charge on DNA and histones is
(a) both positive
(b) both negative
(c) negative and positive, respectively
(d) zero.
23 The DNA site where DNA-dependent RNA- polymerase binds for transcription, is called
(a) operator
(b) promotor
(c) regulator
(d) receptor
24 Arrange the following events in the order of synthesis of a protein
i) A peptide bond forms
ii) A tRNA matches its anticodon to the codon in the A- site
iii) The movement of second tRNA complex from A-site to P-site
iv) The large subunit attaches to the small subunit and the initiator tRNA fits in the P-site
v) A small subunit binds to the mRNA
vi) The activated amino acid tRNA complex attaches the initiation codon on mRNA
a) iv, v, iii, ii, i, vi
b) iv, vi, v, ii, I, iii
c) v, iv, iii, ii, vi, I
d) v, vi, iv, ii, i, iii
25 Match the names of scientists in column I with their achievements in column II and choose the correct
answer given below
Column I Column II
A) Watson and Crick P) DNA fingerprinting
B) R. W. Holley Q) Decipher genetic code
C) Marshal Nirenberg R) Double helix of DNA
D) Jacob and Monod S) Clover model of tRNA
E) Alec Jeffrey T) Lac operon concept
(A) (B) (C) (D) (E)
a) R S P T Q
b) R S Q T P
c) R Q P T S
d) R T S P Q
26 The sequence of nitrogen bases in a ‘segment of a coding strand of DNA is ’ AATGCTTAGGCA.
What will be the
sequence of nitrogen bases in the mRNA transcribed by it?
(a) UUA CGA AUC CGU
(b) AAU GCU AAC CGA
(c) AAU GCA AUC CGU
(d) AAU GCU UAG GCA
27 DNA finger printing is a technique in molecular biology. Arrange the following steps in sequence.
1) Blotting of DNA fragment to nitro cellulose.
2) Digestion of DNA by restriction endonuclease.
3) Deletion of DNA by restriction endonuclease.
4)Isolation of DNA,
5) separation of DNA fragments by
electrophoresis. a) 4 2 1 5 3
b) 3 1 4 5 2
c) 4 3 5 1 2
d) 2 4 1 5 2
e) 4 2 5 1 3
28 In E. coli, the lac operon gets switched on when
(a) lactose is present and it binds to the repressor.
(b) repressor binds to operator.
(c) RNA polymerase binds to the operator.
(d) lactose is present and it binds to RNA polymerase.
29 Which of the following are the functions of RNA?
(a) It is a carrier of genetic information from DNA to ribosomes synthesizing polypeptides.
(b) It carries amino acids to ribosomes
(c) It is a constituent component of ribosomes
(d) All of the above.
30 Discontinuous synthesis of DNA occurs in one strand, because
(a) DNA molecule being synthesized is very long.
(b) DNA dependent DNA polymearsecatalyzes polymerization only in one direction (5′ → 3′).
(c) it is a more efficient process.
(d) DNA ligase has to have a role.
31 Bacteriophage lamda has _ base pairs
a) 45802
b) 48520
c) 48025
d) 48502

32 Escheria coli has _ bp.


a) 4.9 x 106
b) 3.6 x 106
c) 4.6 x 106
d) 3.9 x 106
33 Which of the following is a purine ?
a) Adenine
b) Uracil
c) Cytosine
d) Thymine

34 A free _ moiety at 5’ end of sugar is referred to as 5’ end of polynucleotide chain.


a) Calcium
b) Glucose
c) OH
d) Phosphate
35 DNA as an acidic substance present in nucleus was first identified by _ in 1869.
a) Erwin Chargaff
b) Rosalind Franklin
c) Maurice Wilkins
d) Friedrich Meischer

36 Who proposed the Central Dogma ?


a) Erwin Chargaff
b) Friedrich Meischer
c) Maurice Wilkins
d) Francis Crick

37 The chemical used in Meselson and Stahl’s experiment for buoyant density centrifugation was __ _ .
a) Water
b) 14NH4Cl
c) 15NH4Cl
d) CsCl
38 The RNA polymerase I transcribes _ _.
a) 28S, 18S, and 5.8S rRNA.
b) tRNA, 5s rRNA, and snRNAs
c) hnRNA
d) mRNA and snRNA
39 The process by which the introns are removed and exons are joined in a definite order in eukaryotic
transcription is called _ .
a) Tailing
b) Capping
c) Splicing
d) Ligation

40 The contribution of Dr Har Gobind Khorana in study of Genetic code was .


a) Synthesising RNA molecules with defined combinations of bases.
b) Cell-free system for protein synthesis.
c) Discovering the role of polynucleotide phosphorylase.
d) Discovering the role of DNA dependent DNA Polymerase.
41 Which is the wrong feature of Genetic code dictionary ?
a) Commaless
b) Non degenerate
c) Unambigious
d) Specific
42 Which of the following is not a stop codon?
a) AUG
b) UAA
c) UAG
d) UGA

43 The average gene consists of _ bases.


a) 2700 base
b) 1800 bases
c) 2200 bases
d) 3000 bases

44 Polymorphism in genome occurs due to _ .


a) Replication
b) Mutation
c) Transcription
a) Translation

45 The full form of VNTR is _ .


b) Vast Number of Tandem Repeats
c) Variable Number of Tandem Repeats
d) Vast Number of Transfer RNA
e) Variable Number of Terminal Repeats

46 Which enzyme is used in transcription?


a) Amino acid synthetase
b) DNA polymerase III
c) RNA polymerase
d) DNA ligase

47 According to Chargaff’s rule, which one is correct?


a) [A] + [T] = [G] + [C]
b) [A] + [C] = [G] + [T]
c) [A] + [G] = [T] + [C]
d) Both (a) and (b)

48 Out of 64 codons only 61 codes for the 20 different amino acids. This character of genetic code is
called:
a) Degeneracy
b) Non ambiguous nature
c) Overlapping
d) Universal

49 If percentage of cytosine is 20%, then percentage of adenine will be:


a) 20%
b) 30%
c) 40%
d) 60%

50 What will be the correct gene expression pathway?


a) Gene- mRNA- transcription- translation- protein
b) Transcription- gene- translation- mRNA- protein
c) Gene- transcription- mRNA- translation- protein
d) Gene- translation- mRNA- transcription- protein

51 In genetic fingerprinting, the ‘probe’ refers to......


a) A radioactivity labelled single stranded DNA molecule
b) A radioactivity labelled single stranded RNA molecule
c) A radioactivity labelled double stranded DNA molecule
d) A radioactivity labelled double stranded RNA molecule

52 During Meselson and Stahl’s experiments, heavy DNA was distinguished from normal DNA by
centrifugation in
a) CsCl gradient
b) 14NH4Cl
15
c) NH4Cl
d) 35SO2

53 A typical nucleosome contains:


a) 100 bp of DNA helix
b) 200 bp of DNA helix
c) 300 bp of DNA helix
d) 400 bp of DNA helix.

54 A mixture containing DNA fragments, A, B, C and D with molecular weights of A + C = D, A> C and B> D,
was subjected to agarose gel electrophoresis. The position of these fragments from cathode to anode
sides of the gel would be:
a) A, B, C, D
b) A, C, B, D
c) A, C, D, B
d) B, D, A, C

55 In Hershy and Chase experiments, radioactive 32P was used to culture bacteriophages which resulted
in radioactive:
a) Viral DNA
b) Bacterial capsule
c) Viral proteins
d) Plasma membrane of bacteria

56 In lac operon, the genes a, i, y and z code respectively for:


a) Repressor protein, permease, β- galactosidase, transacetylase.
b) Transacetylase, permease, β- galactosidase, repressor protein.
c) Permease, transacetylase, repressor protein, β- galactosidase.
d) Transacetylase, repressor protein, permease, β- galactosidase.

57 To which of the following factors, RNA polymerase binds transiently to initiate transcription?
a) Rho
b) Beta
c) Gamma
d) Sigma

58 The synthesis of DNA is discontinuous on one strand of the replication fork because:
a) DNA molecule being synthesised is very long.
b) DNA-dependent DNA polymerase catalyse polymerisation only in one direction ( 5’ → 3’)
c) It is more efficient process.
d) It helps to use DNA ligase.

59 Which of the following sequences will be produced as a result of transcription of DNA sequence
AAGCAGGGCA?
a) UUCCGCCUUG
b) UUCGUCCCGU
c) UUCGUGGGU
d) UUAAAGCCC

60 The double helical model of the DNA was proposed by Watson and Crick based on what data
produced by Wilkins and Franklin?
Hybridisation
DNA
sequencing X-
ray diffraction
Southern
blotting

61 Question:Pick the right difference between a DNA and RNA


(a) Sugar and phosphate
(b) sugar and purines
(c) purines and phosphate
(d) sugar and pyrimidines
62 Question:-A bacterial colony containing DNA made up of 100% N15
nitrogen bases is allowed to replicate in a medium containing N14
bases. After one round of replication the result would be.
(a) All individuals will be identical to parents
(b) All individuals will be hybrids
(c) Only 50% individuals would be hybrids
(d) All individuals would have DNA made up of 100% N14
63 Question:-In a transcription unit, the promoter is located towards
(a) 5’end of the structural gene
(b) 3’end of the structural gene
(c) 5’end of the template strand
(d) 3’end of the coding strand

64 Question:- What will the amount of guanine in a DNA if the total amount of adenine and thiamine in
the DNA is 45%?
(a) 45%
(b) 65%
(c)
27.5%
(d)
22.3%
65 Question:- Methyl guanosine triphosphate is added to the 5’ end of hnRNA in a process of
(a) splicing
(b) capping
(c) tailing
(d) none of these
66 Question:- Select the incorrectly matched pairs
(a) Purines – Nitrogenous bases cytosine, thymine and uracil
(b) Recombinant DNA – DNA formed by joining the DNA segments from two different sources
(c) rRNA – RNA found in ribosomes
(d) ATP – The energy-carrying compound in the cell
67 Question:- Which enzyme is not produced during lactose catabolism by E.coli?
(a) Q-galactosidase
(b) Lactose Permease
(c) Thiogalactoside transacetylase
(d) Lactose dehydrogenase
68 Question:- A molecule that acts as a genetic material must fulfil the following traits, except
(a) It should be structurally and chemically unstable
(b) It should have the ability to generate its replica
(c) It should facilitate slow changes necessary for evolution.
(d) It should be able to express itself in the form of Mendelian characters.
69 Question:- Which mRNA will be translated to a polypeptide chain containing 8 amino acids?
(a) AUGUUAAUAGACGAGUAGCGACGAUGU
(b) AUGAGACGGACUGCAUUCCCAACCUGA
(c) AUGCCCAACCGUUAUUCAUGCUAG
(d) AUGUCGACAGUCUAAAACAGCGGG
70 Question:-Hershey and Chase’s experiment was based on the principle
(a) Transformation(b) Translation
(c) Transduction(d) Transcription
71 Question:- One of the following is true with respect to AUG
(a) It codes for methionine only
(b) It is also an initiation codon
(c) It codes for methionine in both prokaryotes and eukaryotes
(d) All of the above
72 Question:-Arrange the following events in the order of synthesis of a protein
i) A peptide bond forms
ii) A tRNA matches its anticodon to the codon in the A- site
iii) The movement of second tRNA complex from A-site to P-site
iv) The large subunit attaches to the small subunit and the initiator tRNA fits in the P-site
v) A small subunit binds to the mRNA
vi) The activated amino acid tRNA complex attaches the initiation codon on mRNA
(a) iv, v, iii, ii, i, vi
(b) iv, vi, v, ii, I, iii
(c) v, iv, iii, ii, vi, I
(d)v, vi, iv, ii, i, iii

73 Question:-Which of the following statements is the most appropriate for sickle cellanaemia?
(a) It cannot be treated with iron
supplements (b)It is a molecular disease
(c) It confers resistance to acquiring malaria
(d) All of the above
74 Question:-Match the entries in column I with those of column II and choose the correct answer.
Column I Column II
A) Alkali treatment M) separation of DNA fragments on gel slab
B) Southern blotting N) split DNA fragments into single strands
C) Electrophoresis O) DNA transferred to nitrocellulose sheet
D) PCR P) X-ray photography
E) Autoradiography Q) produce fragments of different sizes
F) DNA treated with REN R) DNA amplification
(a) A - N, B- Q, C- P, D- R, E- M, F - O
(b) A - P, B - R, C - M, D -O, E - N, F – Q
(c) A - Q, B - O, C - M, D - R, E - P, F - N
(d) A - N, B - O, C - M, D - R, E - P, F - Q
75 Question:-Match the following
Column I Column II
(A) Helicase (M) activation of amino acid
(B) Peptidyl transferase (N) joins DNA fragments
(C) DNA polymerase (O) unwinds DNA helix
(D) DNA ligase (P) peptide bonds between amino acids
(E) Aminoacyl synthetase enzyme (Q) DNA synthesis
(F) RNA primase (R) synthesis of RNA primer
(a) A-O, B-P, C-Q, D-N, E-M, F-R
(b) A-R, B-M, C-N, D-Q, E-P, F-O
(c) A-M, B-R, C-P, D-Q, E-N, F-O
(d) A-R, B- Q, C- A, D- M, E-P, F-N
76 A bacterium grown over medium having radioactive 35S incorporated radioactivity in-
a) Carbohydrates
b) Protein
c) DNA
d) RNA
77 Leading strand during DNA replication is formed-
a) Continuously
b) Okazaki fragments
c) First
d) Ahead of replication
78 Segments of mRNA removed during splicing are called-
a) Introns
b) Exons
c) Promoter region
d) Regulator region
79 If the sequence of nitrogen bases of the coding strand of DNA in a transcription unit is:
5’-ATGAATG-3’
The sequence of bases in its RNA transcript would be-
a) 5’- AUGAAUG-3’
b) 5’-UACUUAC-3’
c) 5’-CAUUCAU-3’
d) 5’-GUAAGUA-3’
80 Haemophilia is more common in males because it is a-
a) Recessive character carried by Y-chromosomes
b) Dominant character carried by Y- chromosomes
c) Dominant trait carried by X- chromosomes
d) Recessive trait carried by X-chromosomes
81 In DNA strand, the nucleotides are linked together by-
a) Glycosidic bonds
b) Phosphodiester bonds
c) Peptide bond
d) Hydrogen bond
82 If a double stranded DNA has 20% of cytosine, what will be the percentage of adenine i
a) 20%
b) 40%
c) 30%
d) 60%
83 The three codons which result in the termination of polypeptide chain synthesis are-
a) UAA,UAG,UGA
b) UAA,UAG,GUA
c) UAA,UGA,UUA
d) UGU,UAG,UGA
84 Which of the following statement is incorrect-
a) Genetic code is ambiguous
b) Genetic code is degenerate
c) Genetic code is universal
d) Genetic code is non-overlapping
85 Amino acid acceptor end of tRNA lies at-
a) 5’ end
b) 3’ end
c) TVC loop
d) DHU loop
86 RNA is the genetic material in- [
a) Prokaryotes
b) Eukaryotes
c) Tobacco Mosaic Virus (TMV)
d) E. coli
87 DNA replication takes place at which phase of the cell?-
a) G1
b) S
c) G2
d) M
88 In transcription in eukaryotes, heterogeneous nuclear RNA is transcribed by-
a) RNA polymerase I
b) RNA polymerase II
c) RNA polymerase III
d) All of these
89 In the lac operon of E. coli , the I - gene code for-
a) Inducer
b) Repressor
c) Lactase
d) B- galactosidase
90 The promoter site and the terminator site for transcription are located at-
a) 3’ (downstream) end and 5’ (upstream) end respectively of the transcription unit
b) 5’(upstream) end and 3’ (downstream) end respectively of the transcription unit
c) The 5’ (upstream) end
d) The 3’ ( downstream) end
91 Bacteriophage lamda has _ base pairs
a) 45802
b) 48520
c) 48025
d) 48502
92 Escheria coli has _ bp.
6
a) 4.9 x 10
b) 3.6 x 106
c) 4.6 x 106
d) 3.9 x 106
93 Which of the following is a purine ?
a) Adenine
b) Uracil
c) Cytosine
d) Thymine
94 A free _ moiety at 5’ end of sugar is referred to as 5’ end of polynucleotide chain.
a) Calcium
b) Glucose
c) OH
d) Phosphate
95 DNA as an acidic substance present in nucleus was first identified by _ in 1869.
a) Erwin Chargaff
b) Rosalind Franklin
c) Maurice Wilkins
d) Friedrich Meischer
96 Who proposed the Central Dogma ?
a) Erwin Chargaff
b) Friedrich Meischer
c) Maurice Wilkins
d) Francis Crick
97 The chemical used in Meselson and Stahl’s experiment for buoyant density centrifugation was
_ .
a) Water
b) 14NH4Cl
15
c) NH4Cl
d) CsCl
98 The RNA polymerase I transcribes .
a) 28S, 18S, and 5.8S rRNA.
b) tRNA, 5s rRNA, and snRNAs
c) hnRNA
d) mRNA and snRNA
99 The process by which the introns are removed and exons are joined in a definite order in
eukaryotic transcription is called _ .
a) Tailing
b) Capping
c) Splicing
d) Ligation
100 The contribution of Dr Har Gobind Khorana in study of Genetic code was .
a) Synthesising RNA molecules with defined combinations of bases.
b) Cell-free system for protein synthesis.
c) Discovering the role of polynucleotide phosphorylase.
d) Discovering the role of DNA dependent DNA Polymerase .
101 Which is the wrong feature of Genetic code dictionary ?
a) Commaless
b) Non degenerate
c) Unambigious
d) Specific
102 Which of the following is not a stop codon?
a) AUG b) UAA
c) UAG d) UGA
103 The average gene consists of _ bases.
a) 2700 base
b) 1800 bases
c) 2200 bases
d) 3000 bases
104 Polymorphism in genome occurs due to _.
a) Replication
b) Mutation
c) Transcription
a) Translation
105 The full form of VNTR is _ .
b) Vast Number of Tandem Repeats
c) Variable Number of Tandem Repeats
d) Vast Number of Transfer RNA
e) Variable Number of Terminal Repeats

106 RNA Which functions as adapter molecule for amino acid is


a) mRNA
b) tRNAA
c) snRNA
d) tRNA
107 RNA codon for DNA codon ATG will be
a) AUG
b) UTG
c) UAC
d) TAC
108 Segments of DNA which can shift positions are
a) Cistrons
b) Transposons
c) Introns
d) Exons
109 Termination codon is
a) UGG
b) GUG
c) UAG
d) AUG
110 Anticodon occurs over
a) tRNA
b) DNA
c) mRNA
d) Rrna
111 Gene of operon which forms the repressor protein is
a) Operator
b) Promoter
c) Regulator
d) Structural
112 Central dogma of molecular biology was proposed by
a) Khorana
b) Beadle and tatum
c) Watson and crick
d) Crick
113 Nucleotides base present in RNA but not in DNA is
a) C
b) U
c) A
d) G
114 Pick the right difference between a DNA and RNA
A. Sugar and phosphate
B. sugar and purines
C. purines and phosphate
D. sugar and pyrimidines
115 Hershey and Chase experiment proving DNA as the genetic material was based on the principle
A. Transduction
B. transformation
C. transcription
D translation
116 Cistron is
A. The coding sequence of DNA
B. The functional unit of DNA molecule that codes for a particular gene product
C. Intervening non coding sequence of DNA
D. The sequences which are removed during RNA splicing.
117 The coding sequences found in split genes are called
A. Operons
B. introns
C. exons
D. cistrons
118 The removal of which enzyme affects the synthesis of hnRNA in eukaryotes
A. RNA polymerase II
B. RNA primase
C. RNA polymerase III
D. RNA polymerase I
119 Sickle cell anemia is caused
A. When valine is replaced by glutamic acid in beta polypeptide chain
B. When glutamic acid is replaced by valine in beta polypeptide chain
C. When glutamic acid is replaced by valine in alpha polypeptide chain
D. When valine is replaced by glutamic acid in alpha polypeptide chain

120 Wobble position means


A. Base paring
B. altered base on code
C. third altered base on codon
D. none of the above
121 Peptidyl transferase
A. Is a 23s rRNA
B. forms peptide bonds
C. component of ribosome
D. all the three
122 Which mRNA will be translated to a polypeptide chain containing 8 amino acids?
A. AUGUUAAUAGACGAGUAGCGACGAUGU
B. AUGAGACGGACUGCAUUCCCAACCUGA
C. AUGCCCAACCGUUAUUCAUGCUAG
D. AUGUCGACAGUCUAAAACAGCGGG

123 Arrange the following events in the order of synthesis of a protein


i) A peptide bond forms
ii) A tRNA matches its anticodon to the codon in the A- site
iii) The movement of second tRNA complex from A-site to P-site
iv) The large subunit attaches to the small subunit and the initiator tRNA fits in the P-site
v) A small subunit binds to the mRNA
vi) The activated amino acid tRNA complex attaches the initiation codon on mRNA
A. iv, v, iii, ii, i, vi
B. iv, vi, v, ii, I, iii
C. v, iv, iii, ii, vi, i
D. v, vi, iv, ii, i, iii
124 Select the incorrect statement out of the five given below about lac operon when Lactose is
present in the medium.
A . Gene – A gets transcribed into mRNA which produces β-galactoside permease
B. Inducer-Repressor complex is formed
C. Lactose inactivates repressor protein
D. RNA polymerase transcribe Z-gene, Y-gene and A-gene
125 The percentage of human genome which encodes proteins is approximately
a) Less than 2%
b) 5%
c) 25%
d) 99%
126 Enzyme which can break and seal the DNA strand
A. Topoisomerase II
B. Helicase
C. Primase
D. Restriction endonuclease
127 Which of the statements give below is correct with respect to frameshift mutation
A. a single nucleotide base change, insertion, or deletion of the genetic material
B. Glutamine is replaced by valine
C. Sickle cell anemia is an example
D. insertions or deletions of a number of nucleotides in a DNA sequence that is not divisible by
three
128 The transcription initiation factor associated with the RNA polymerase holoenzyme in
prokaryotes is
a) β
b) ω
c) σ
d) Α
129 The stretch of codons between AUG and a stop codon is called
a) open reading frame
b) TATA box
c) colinearity
d) Degenerate
130 The structural genes of lac operon transcribe mRNA which is
a) polycistronic
b) replicative
c) monokaryotic
d) Monocistronic
131 The first genetic material could be
a) protein
b) Carbohydrate
c) DNA
d) RNA

132 Which of the following statement is appropriate for sickle cell anaemia ?
A . it can not be treated with iron supplement
B. it is a molecular disease
C. it confers resistance to acquiring malaria
D. all of the above
133 When two hybrids are crossed, the percentage of recessive is
a) 25%
b) 100%
c) 50%
d) 75%
134 In A DNA strand the nucleotides are linked together by
A. Glycodidic bond
B. phosphodiester bond
C. peptide bond
D. hydrogen bond
135 Which of the following is not a part of transcription unit
A. Inducer
B. A terminator
C. A promoter
D. The structural gene
136 Identify the none-sense codon of the following
a) AUG
b) GUG
c) UAA
d) UGC
137 Nucleotide arrangement in DNA can be seen by
(a) X- ray crystallography
(b) Electron microscope
(c)Ultra centrifuge
(d) Light microscope
138 The distance between two nucleotide in a strand of DNA molecule is
(a) 3.4Å
(b) 4.3Å
(c) 20 Å
(d) 36 Å
139 Among the RNA polymerases, RNA polymerase II transcribes
(a) r- RNA
(b) t-RNA
(c) Sn-RNA
(d) hn-RNA
140 The unequivocal proof of DNA as the genetic material came from the studies on a
(a) Viroid
(b) Bacterial virus
(c) Bacterium
(d) Fungus
141 A single strand of nucleic acid tagged with a radioactive molecule is called
(a) Vector
(b) Selectable marker
(c) Plasmid
(d) Probe
142 If one strand of DNA has the nitrogenous base sequence as ATCTC, what would be the
complementary strand sequence?
(a) TTAGU
(b) UAGAG
(c) AACTG
(d)ATCGU
143 What would happen if in a gene encoding a polypeptide of 50 amino acids,25th codon (UAU) is
mutated to UAA
(a) A polypeptide of 25 amino acid will be formed
(b) A polypeptide of 24 amino acid will be formed
(c) Two polypeptides of 25 and 24 amino acids will be formed
(d) A polypeptide of 49 amino acid will be formed
144 Satellite DNA is useful tool in
(a) Forensic science
(b) Genetic engineering
(c) Organ transplantation
(d) Sex determination
145 The 3I---5I phosphodiester linkages inside a polynucleotide chain serve to join
(a) One nitrogen base with pentose sugar
(b) One DNA strand with other DNA strand
(c) One nucleoside with the other nucleoside
(d) One nucleotide with the other nucleotide
146 In some viruses, DNA is synthesised by using RNA as template, such a DNA is called
(a) A- DNA
(b) B-DNA
(c) c DNA
(d) r-DNA
147 Total amount of adenine and thymine in a double stranded DNA is 45%. The amount of guanine
in this DNA would be
(a) 22.5%
(b) 27.5%
(c) 45%
(d) 55%
148 "What is it that forms the basis of DNA fingerprinting?
(a) The relative proportions of purines and pyrimidines in DNA.
(b) The relative difference in the DNA occurrence in blood, skin and saliva.
(c) The relative amount of DNA in the ridge and grooves of the fingerprints.
(d) Satellite DNA occuring as highly repeated short DNA segments

149 Which enzymes will be produced in a cell in which there is a nonsense mutation the Lac y gene?
(a) Lactose permease and transacetylae
(b) galactosidase
(c) Lactose permease
(d) Transacetylabe

150 Which one of the following is not applicable to RNA?


(a) 5' phosphonyl and 3' hydroxyl ends
(b) Heterocylicnitrogenous bases
(c) Chargaff's rule
(d) Complementary base pairing

151 Which one of the following is wrongly matched?


(a) Repressor protein-Binds to operator to stop enzyme synthesis
(b) Operon -structural genes, operator and promoter.
(c) Transcription- writing information DNA to t-RNA.
(d) Translation - using information in m-RNA to make protein.

152 Which among the following statements are true about DNA?
i) DNA is long polymer of deoxyribonucleotides
ii) The length of DNA is defined as no. of nucleotides
iii) DNA consist of 2 parallel strands opposite in direction
iv) Haploid content of human DNA is0.33x109 bp.
A) (i), ii , iii
B) (i), ii , iv
C) (i), ii ,
D) (i), iv , iii
153 Figure A shows the structure of polynucleotide chain, identify the types of bond shown by X, Y, Z
and choose correct option from the table below.

X Y Z
A Hydrogen Bond Glycosidic linkage Phosphodiester
linkage
B Hydrogen Bond Phosphoester linkage Glycosidic linkage
C Hydrogen Bond Glycosidic linkage Phosphoester linkage
D Glycosidic linkage Hydrogen Bond Phosphoester linkage
154 Which is not true about the DNA model proposed by Watson and crick.
A) The bases in two strands are paired through Hydrogen bonds
B) Two chains coiled in left handed fashion.
C) There are roughly 10 bp in each turn
D) Pitch of helix is 3.4nm
155 Find the correct sequence of gene expression. Where
X --- DNA, Y---- RNA, Z ------ Protein
A) X—Translation→ Y ------- Transcription→ Z
B) X ------Transcription→ Y—Translation→ Z
C) Y—Translation→ Z ------ Transcription→X
D) Z ----- Transcription→ Y—Translation→ X
156 Histone is rich in which amino acids.
A) Lysin
B) Lysin and arginine
C) Lysin, arginine and histidine
D) Only arginine
157 Chose the correct sequence of DNA packaging.
A) DNA, Nucleosome, Chromatin, Chromosomes.
B) DNA, Chromatin, Nucleosome, Chromosome
C) Chromosome, chromatin, nucleosome, DNA
D) Chromatin, DNA , Nucleosome, Chromosome.
158 What is not true about transforming principle.
A) Transforming principle was given by Frederick Griffith
B) Mice infected with S strain die due to pneumonia
C) Mice infected with R strain do not die.
D) Mice infected with heat killed S Strain die due to pneumonia.
159 Viruses where grown in medium containing Radioactive sulfur and phosphorus it will
respectively label.
A) DNA and protein
B) Protein and DNA
C) mRNA and DNA
D) mRNA and Protein
160 Hershey and chase used Bacteriophage and E.coli for the experiment choose the correct
sequence of the steps.
i) Infection ii) labelling iii) centrifugation iv) blending
A) (i), iv, iii, ii
B) (ii), i ,iv, iii
C) (i), ii, iii, iv
D) (ii), iii, i, iv
161 Which should not be the property of genetic material.
A) It should chemically and structurally be stable.
B) It should not provide the scope for sow changes.
C) It should be able to generate its replica.
D) It should be able to express itself in the form of mendelian characters.
162 DNA replication requires various enzymes, table represent enzyme and function, choose the
function of respective enzyme.
X Y Z W
(i) Helicase Breakdown of H Joining DNA Formation of Joining DNA
bond fragments RNA primer fragments
(ii) DNA Polymerization of Polymerization Polymerization Polymerization
polymerase nucleotides of nucleotides of nucleotides of nucleotides
(iii) Ligase Joining DNA Formation of Joining DNA Breakdown of
fragments RNA primer fragments H bond
(iv) Primase Formation of RNA Breakdown of Breakdown of Formation of
primer H bond H bond RNA primer
A) X
B) Y
C) Z
D) W
163 Which among the following is not the part of transcription unit
A) Terminator
B) Promotor
C) Repressor
D) Structural gene
164 A nucleotide consist of?
A) RNA and five carbon sugar
B) 5 carbon sugar, phosphate, nitrogenous base
C) Phosphate, nitrogenous base
D) 5 carbon sugar and nitrogenous base
165 Match the following:
(a) RNA Pol I (i) DNA
(b) RNA Pol II (ii) rRNA
© RNA pol III (iii) hnRNA
(d) DNA pol I (iv) tRNA

(a) (b) © (d)


A (iii) (iv) (ii) (i)
B (ii) (iii) (iv) (i)
C (iii) (ii) (iv) (i)
D (ii) (iv) (iii) (i)
166 Q 30 Which among the following steps are not true about the post transcription modification of
hnRNA.
(i) 7 Methyl guanosine added to 3’ end of mRNA.
(ii) Exons are removed by spliceosomes
(iii) Mature mRNA consist of only Expressed sequences.
(iv) Capping and tailing done to prevent degradation of mRNA
A) (ii), iv
B) (i), ii
C) (iii), iv, i
D) (ii),iii
167 Reverse transcription will occur in which of the following virus?
I. Adeno virus
II. Corona virus
III. Hepatitis B virus
IV. HIV
(A) Option I & II
(B) Option I & III
(C) Option II& IV
(D) Option II, III & IV
168 RNA polymerase is a
(A) RNA dependent DNA polymerase
(B) DNA dependent RNA polymerase
(C) DNA dependent DNA polymerase
(D) RNA dependent DNA polymerase
169 Which of the following observations of Hershey-Chase experiment proved that DNA is genetic
material?
(A) The bacteriophage labelled with radioactive sulphur made the bacterial DNA radioactive
(B) The bacteriophage labelled with radioactive phosphorus did not make bacteria radioactive
(C) Bacteriophage labeled with radioactive sulphur made only the bacterial proteins
radioactive
(D) Bacteriophage labored with radioactive phosphorus made the bacterial DNA radioactive.
170 Find out the correct matching of column I and column II to from the quotes given below.
Column IColumn II
a. Helicase (i) Joining of deoxyribonucleotides
b. Topoisomerase (ii)Working within Y-fork
c. DNA directed RNA polymerase (iii) Unlinking of DNA
d. DNA polymerase III (iv) RNA primer
(A) a-(ii), b-(iii), c-(iv), d-(i)
(B) a-(i), b-(ii), c-(iii), d-(iv)
(C) a-(iv), b-(iii), c-(i), d-(ii)
(D) a-(ii), b-(iv), c-(iii), d-(i)
171 Which one of the following is wrongly matched?
(A) Transcription - Formation of mRNA from DNA
(B) Operon - Structural genes, operator, promoter and intron
(C) Repressor protein - Binds to operator to stop enzyme synthesis
(D) Translation - Formation of protein from mRNA
172 Identify ‘a’, ‘b’ and ‘c’ from the given figure.

{{
(A) a-Replication, b-Translation, c-Transcription
(B) a-Reverse Transcription, b-Replication, c-Translation
(C) a-Replication, b-Transcription, c-Translation
(D) a- Translation, b-Replcation, c-Transcription
173

Identify ‘A’, ‘B’ and ‘C’ from the given figure.


(A) A-Discontinuous strand, B-Continuous strand, C-Template strand
(B) A- Template strand, B- Discontinuous strand, C-Lagging strand
(C) A-Parental strand, B-Leading strand, C-Lagging strand
(D) All of these.

174 Which of the following statements is not correct regarding genetic code?
(A) The codon is triplet. 61 codons code for amino acids and 3 codons do not code for any
amino acids.
(B) The genetic code has punctuations as codon is read in mRNA in a discontiguous fashion.
(C) The code is degenerate because some amino acids are coded by more than one codon.
(D) AUG has dual functions. It codes for Methionine (met) , and it also act as initiator codon.
175 Identify ‘a’, ‘b’, ‘c’and‘d’ from the given figure.
176 Read the given statements and select the correct option.
Statement 1: RNA polymerase II transcribes heterogeneous nuclear RNA (hnRNA).
Statement 2: Complete processing of hnRNA involves only splicing.
(A) Both statements 1 and 2 are correct and statement 2 is the correct explanation of
statement 1
(B) Both statements 1 and 2 are correct but statement 2 is not the correct explanation of
statement 1
(C) Statement 1 is correct but statement 2 is incorrect
(D) Both statements 1 and 2 are incorrect.
177 In a DNA strand, two adjacent nucleotides are linked together by
(A) 3’ – 5’ phospho diester bond
(B) Van der Waals forces
(C) Disulphide linkage
(D) Hydrogen bonds.
178 Fill in the blanks with suitable terms in the statements given below:
I. In eukaryotes, organisation of genetic material occurs with a set of positively
charged, basic proteins called (a)_ _.
II. Histones are organised to form a unit of eight molecules called (b)_ _.
III. The negatively charged DNA is wrapped around the positively charged histone
octamer to form a structure called (c)_ _.
a b c
(A) Histone octomer, histones, nucleosome
(B) Histones, histone octamer, nucleosome
(C) Histones, nucleosome, histone octamer
(D) Nucleosome, histones, histone octamer.
179 ‘Base pair rise’ in a typical DNA molecule is
(A) 3.4 nm
(B) 3.4 μm
(C) 0.34 nm
(D) 0.34 Å
180 Which of the following is an initiation codon in prokaryotes?
(A) AUG
(B) GUC
(C) GUA
(D) GUU.
181 Which of the following helps in bringing a specific amino acid at its proper position during
translation?
(A) Codon
(B) mRNA
(C) RNA
(D) Anticodon.
182 During transformation experiment a combination of rough and smooth bacteria killed mice
because
(A) Live rough type bacteria transformed to live smooth type bacteria
(B) Dead smooth type bacteria transformed into live rough type bacteria
(C) Dead rough type bacteria transformed to live live rough type bacteria
(D) Live smooth type bacteria transformed to live rough type bacteria.
MCQ ANSWERS (MOLECULAR BASIS OF
INHERITANCE)
1 A 27 E 53 B 79 A 105 C 131 D 157 A
2 D 28 A 54 D 80 D 106 D 132 D 158 D
3 A 29 D 55 A 81 B 107 C 133 A 159 B
4 D 30 B 56 D 82 C 108 B 134 B 160 B
5 A 31 D 57 D 83 A 109 C 135 A 161 B
6 C 32 C 58 B 84 A 110 A 136 C 162 A

7 C 33 A 59 B 85 B 111 C 137 A 163 C


8 A 34 D 60 C 86 C 112 D 138 A 164 B
9 A 35 D 61 D 87 B 113 B 139 D 165 B
10 C 36 D 62 B 88 D 114 D 140 B 166 B

11 B 37 D 63 B 89 B 115 A 141 D 167 C


12 B 38 A 64 C 90 B 116 B 142 B 168 D
13 C 39 C 65 B 91 D 117 C 143 B 169 D
14 D 40 A 66 A 92 C 118 A 144 A 170 A

15 A 41 B 67 D 93 A 119 B 145 C 171 B


16 D 42 A 68 A 94 D 120 B 146 C 172 C
17 C 43 D 69 B 95 D 121 D 147 B 173 C
18 B 44 B 70 C 96 D 122 B 148 D 174 C
19 A 45 C 71 D 97 D 123 D 149 B 175 D
20 C 46 C 72 C 98 A 124 A 150 C 176 C
21 C 47 A 73 D 99 C 125 A 151 C 177 A
22 C 48 A 74 D 100 A 126 A 152 C 178 B
23 B 49 B 75 A 101 B 127 D 153 C 179 C

24 D 50 C 76 B 102 A 128 C 154 B 180 A


25 B 51 A 77 A 103 D 129 A 155 B 181 D
26 D 52 A 78 A 104 B 130 A 156 C 182 A

ASSERTION REASONING MCQ

Sl Questions No
Assertion (A):……… Reason (R):
………..
(A) Both Assertion and Reason are true and reason is the correct explanation of assertion.
(B) Both Assertion and Reason are true and reason is not the correct explanation of assertion.
(C) Assertion is true and reason is false.
(D) Assertion is false and reason is true.
1 Assertion (A):hn RNA is larger than mRNA..
Reason (R): hn RNA has non-translating introns which are not Required for translation.
2 Assertion(A): Genetic code is degenerate.
Reason (R): Most amino acids are coded by more than one codon.
3 Assertion (A): Split gene concept is applicable only to the prokaryotes.
Reason (R):Eukaryotic genome is divided into exons and introns.
4 Assertion (A): The genetic code are commaless.
Reason (R): Genetic codes are overlapping.
5. Assertion (A): The operon is a unit of gene expression.
Reason (R): Lac operon in E. coli is an inducible control.
6 Assertion: The two strands of DNA are antiparallel
Reason: Only antiparallel polynucleotides form a stable double helix
(A) Both Assertion and reason are true and reason is correct explanation of assertion.
(B) Assertion and reason both are true but reason is not the correct explanation of assertion.
(C) Assertion is true, reason is false.
(D) Assertion is false, reason is true.
7 Assertion (A): DNA replication occurs in small replication forks and not in its entire length. Reason(R):
Replication of DNA does not initiate randomly and DNA polymerases on their own cannot initiate
replication.
(A) Both Assertion and reason are true and reason is correct explanation of assertion.
(B) Assertion and reason both are true but reason is not the correct explanation of assertion.
(C) Assertion is true, reason is false.
(D) Assertion is false, reason is true.
8 Assertion: The anticodon loop of the tRNA contains bases that are complementary
to the codes.
Reason(R): The stop codons are UAA, UAG and UGA.
(A) Both Assertion and reason are true and reason is correct explanation of assertion.
(B) Assertion and reason both are true but reason is not the correct explanation of assertion.
(C) Assertion is true, reason is false.
(D) Assertion is false, reason is true.
9 Assertion: Primary transcripts in eukaryotes are nonfunctional.
Reason: Methyl guanosine triphosphate is attached to 5’ – end of hnRNA (A) Both Assertion and
reason are true and reason is correct explanation of assertion.
(B) Assertion and reason both are true but reason is not the correct explanation of assertion.
(C) Assertion is true, reason is false.
(D) Assertion is false, reason is true.
10 Assertion: tRNA is called an ‘adapter’
Reason: tRNA on one hand bind to a specific amino acid and on the other hand reads the codon of
the amino acid bound to it through its anticodon
(A) Both Assertion and reason are true and reason is correct explanation of assertion.
(B) Assertion and reason both are true but reason is not the correct explanation of assertion.
(C) Assertion is true, reason is false.
(D) Assertion is false, reason is true.

(A) Both Assertion and reason are true and reason is correct explanation of assertion.
(B) Assertion and reason both are true but reason is not the correct explanation of assertion.
(C) Assertion is true, reason is false.
(D) Assertion is false, reason is true.
11 Assertion: DNA is better genetic material than RNA.
Reason: DNA is more heat stable than RNA.
12 Assertion: In eukaryotes, the process of transcription and translation are discontinuously occurring
events.
Reason: In eukaryotes, transcription occurs in nucleus while translation in cytoplasm.
13 Assertion: During transcription only one of the strands functions as template strand.
Reason: Transcription forms single stranded RNA.
14 Assertion: HGP is closely associated with the rapiddevelopment of a new area in biology called
Bioinformatics.
Reason: Bioinformatics uses high speed computational devices for data storage, retrieval and
analysis of enormous amount of data generated from HGP.
15 Assertion: Regulation of lac operon can also be visualised as regulation of substrate synthesis by its
enzyme.
Reason: Lactose regulates switching on and off of the operon.
DIRECTION- Choose correct option from given below:
A) Both assertion and reason are true and reason is a correct explanation of the assertion.
B) Both assertion and reason are true but the reason is not the correct explanation of
assertion.
C) If the assertion is true but the reason is false.
D) If both assertion and reason are false.
16 ASSERTION- DNA is considered to be better genetic material than RNA for most organisms.
REASON-2’-OH group present in DNA makes it labile and less reactive
17 ASSERTION-Histones are basic in nature.
REASON- Histones are rich in amino acid lysine and arginine.
18 ASSERTION- DNA acts as a genetic material in all organisms.
REASON- It is a single -stranded biomolecule.
19 ASSERTION-Split genes concept is applicable only to the prokaryotes.
REASON- Prokaryotic genome is divided into exons and introns.
20 ASSERTION- Ribosomal RNA is synthesized in the cytoplasm of the cell. REASON-
It is translated with the enzyme RNA polymerase III.
21 Assertion : The RNA molecules are essential for cell function in both prokaryotes and eukaryotes.
Reason : They play an important role in protein synthesis.
(a) Both the Assertion and the Reason are true and the Reason is a correct explanation of the
Assertion.
(b) Both the Assertion and Reason are true but the Reason is not a correct explanation of the
Assertion.
(c) The Assertion is true but the Reason is false.
(d) Both the Assertion and Reason are false

22 Assertion: The genetic codes are commaless.


Reason: Genetic codes are overlapping.
(a) Both the Assertion and the Reason are true and the Reason is a correct explanation of the
Assertion.
(b) Both the Assertion and Reason are true but the Reason is not a correct explanation of the
Assertion.
(c) The Assertion is true but the Reason is false.
(d) Both the Assertion and Reason are false
23 Assertion :ThehnRNA in humans has exons and introns.
Reason : The primary transcript produced in eukaryotes is translated without undergoing any
modification
or processing.
(a) Both the Assertion and the Reason are true and the Reason is a correct explanation of the
Assertion.
(b) Both the Assertion and Reason are true but the Reason is not a correct explanation of the
Assertion.
(c) The Assertion is true but the Reason is false.
(d) Both the Assertion and Reason are false
24 Assertion :Ribosomal RNA is synthesized in the cytoplasm of the cell.
Reason : It is translated with the enzyme RNA polymerase III.
(a) Both the Assertion and the Reason are true and the Reason is a correct explanation of the
Assertion.
(b) Both the Assertion and Reason are true but the Reason is not a correct explanation of the
Assertion.
(c) The Assertion is true but the Reason is false.
(d) Both the Assertion and Reason are false
25 Assertion : The operon is a unit of gene expression.
Reason : Lac operon in E. coli is an inducible control.
(a) Both the Assertion and the Reason are true and the Reason is a correct explanation of the
Assertion.
(b) Both the Assertion and Reason are true but the Reason is not a correct explanation of the
Assertion.
(c) The Assertion is true but the Reason is false.
(d) Both the Assertion and Reason are false
26 Assertion: DNA is macromolecule of high molecular weight
Reason: Amino acids are monomeric units of DNA.
(A) Both Assertion and reason are true and reason is correct explanation of assertion.
(B) Assertion and reason both are true but reason is not the correct explanation of assertion.
(C) Assertion is true, reason is false.
(D) Assertion is false, reason is true.
27 Assertion:A. In a DNA molecule A -Trichproteinsmelt before G-C rich proteins.
Reason: There are 2H bonds between A and T, whereas 3H bonds are present between G and C
(A) Both Assertion and reason are true and reason is correct explanation of assertion.
(B) Assertion and reason both are true but reason is not the correct explanation of assertion.
(C) Assertion is true, reason is false.
(D) Assertion is false, reason is true.
28 Assertion: A. In prokaryotes initiating amino acid methionine is formylated
Reason. RNA carries methionine to the initiator codon on mRNA
(A) Both Assertion and reason are true and reason is correct explanation of assertion.
(B) Assertion and reason both are true but reason is not the correct explanation of assertion.
(C) Assertion is true, reason is false.
(D) Assertion is false, reason is true.
29 Assertion: A. Lac operon given by Jacob and Monod is repressible operon.
Reason: The system of regulation in lac operon is always a negative control
(A) Both Assertion and reason are true and reason is correct explanation of assertion.
(B) Assertion and reason both are true but reason is not the correct explanation of assertion.
(C) Assertion is true, reason is false.
(D) Assertion is false, reason is true.
30 Assertion: A. HnmRNA is larger than mRNA.
Reason:Hn RNA has non-translating introns and more exons than required for translation
(A) Both Assertion and reason are true and reason is correct explanation of assertion.
(B) Assertion and reason both are true but reason is not the correct explanation of assertion.
(C) Assertion is true, reason is false.
(D) Assertion is false, reason is true.
Directions: In the following questions, a statement of assertion is followed by a statement of reason.
Mark the correct choice as:
(A) both Assertion and Reason are true and Reason is the correct explanation of Assertion.
(B) both Assertion and Reason are true but Reason is not the correct explanation of Assertion.
(C) Assertion is true but Reason is false.
(D) Assertion is false and Reason are true

31 Assertion : In a DNA molecule, A–T rich parts melt before G–C rich parts.
Reason : In between A and T there are three H–bond, whereas in between G and C there are two H-
bonds.
32 Assertion: The two chains of DNA have anti-polarity.
Reason: In one chain of DNA, ribose sugar at 5′ end consists of a free phosphate moiety while at the
other end the ribose has a free 3′ OH group.
33 Assertion: Adenine cannot pair with cytosine.
Reason: Adenine and cytosine do not have a perfect match between hydrogen donor and hydrogen acceptor
sites. Hence, they cannot pair.
34 Assertion: The sugar phosphate backbone of two chains in DNA doutble helix show anti-parallel
polarity.
Reason: The phosphodiester bonds in one strand go from a 3′ carbon of one nucleotide to a 5′ carbon of
adjacent nucleotide, whereas those in complementary strand go vice versa.
35 Assertion: DNA is considered to be better genetic material than RNA for most organisms.
Reason: 2′-OH group present in DNA makes it labile and less reactive.
Directions: In each of the following questions, a statement of Assertion is given and a corresponding
statement of Reason is given just below it. Of the statements, given below, mark the correct answer as:
(a) Both assertion and reason are true and reason is the correct explanation of assertion.
(b) Both assertion and reason are true but reason is not the correct explanation of assertion.
(c) Assertion is true but reason is false.
(d) Both Assertion and Reason are false.
36 Assertion: Prokaryotic DNA is not scattered throughout the cell.
Reason: DNA held with protein in region termed as nucleoids.
37 Assertion: Heterochromatin is said to be transcriptionally inactive.
Reason: Heterochromatin is loosely packed.
38 Assertion: DNA replication is semiconservative. Reason:
Replication takes place from 5’ to 3’ direction.
39 Assertion: Transcription occurs from 5’ to 3’ direction.
Reason: RNA polymerase binds to promoter region located at 5’ end of the template strand.
40 Assertion: Lactose is termed as inducer.
Reason: Lactose regulates switching o and off of the operon.
41 Assertion (A): In Griffith’s experiment mice were injected by a mixture of heat killed Smooth type bacteria
and live Rough type bacteria. Some mice died of pneumonia and their blood contained both live Rough
type bacteria and live Smooth type bacteria.
Reason (R): The dead Smooth type bacteria became alive and caused pneumonia. Griffith named it as
transforming principle
(A) Both Assertion and reason are true and reason is correct explanation of assertion.
(B) Assertion and reason both are true but reason is not the correct explanation of assertion.
(C) Assertion is true, reason is false.
(D) Assertion is false, reason is true.
42 Assertion (A): Histones are basic in nature
Reason (R): Histones are rich in amino acids lysine and arginine
(A) Both Assertion and reason are true and reason is correct explanation of assertion.
(B) Assertion and reason both are true but reason is not the correct explanation of assertion.
(C) Assertion is true, reason is false.
(D) Assertion is false, reason is true.
43 Assertion (A): Replication on one strand of DNA is continuous and on another is discontinuous
Reason (R): The DNA polymerase works in 3’-5’direction.
(A) Both Assertion and reason are true and reason is correct explanation of assertion.
(B) Assertion and reason both are true but reason is not the correct explanation of assertion.
(C) Assertion is true, reason is false.
(D) Assertion is false, reason is true.
44 Assertion (A): Replication and transcription occur in the nucleus but translation takes place in the
cytoplasm
Reason (R): mRNA is transferred from the nucleus into cytoplasm where ribosomes and amino acids are
available for protein synthesis.
(A) Both Assertion and reason are true and reason is correct explanation of assertion.
(B) Assertion and reason both are true but reason is not the correct explanation of assertion.
(C) Assertion is true, reason is false.
(D) Assertion is false, reason is true.
45 Assertion (A): The genetic code is degenerate.
Reason (R): Most amino acids are coded by more than one codon
(A) Both Assertion and reason are true and reason is correct explanation of assertion.
(B) Assertion and reason both are true but reason is not the correct explanation of assertion.
(C) Assertion is true, reason is false.
(D) Assertion is false, reason is true.
The following questions consist of two statements each, printed as Assertion and Reason. While
answering these questions you are required to choose any one of the following four responses.
A. If both Assertion and Reason are true and Reason is a correct explanation of the Assertion.
B. If both Assertion and Reason are true and Reason is not a correct explanation of the Assertion.
C. If Assertion is true but the Reason is false.
D. If both Assertion and Reason are false
46 Assertion: DNA molecule acts as a template for synthesis of both RNA and DNA.
Reason: DNA duplex has the feature of right handed coiling and antiparallel.
47 Assertion: Guanine cannot pair with cytosine.
Reason: Guanine and cytosine do not have perfect matching.
48 Assertion: Amber codon is a termination codon.
Reason: if in a mRNA, a termination codon is present, the protein synthesis stops abruptly whether the
protein synthesis is complete or not.
49 Assertion: Sickle cell anemia is a genetic disorder due to point mutation.
Reason: Phenotypic character of beta- globin is changed.
50 Assertion: In bacteria, replication, transcription and translation take place in cytoplasm of the cell.
Reason: In bacteria, there is no well defined nucleus.

ANSWERS (ASSERTION REASONING MCQ)


1 A 11 A 21 A 31 C 41 A
2 A 12 A 22 C 32 A 42 A
3 A 13 B 23 C 33 A 43 C
4 D 14 B 24 D 34 A 44 A

5 B 15 B 25 B 35 C 45 A
6 A 16 C 26 C 36 A 46 A
7 B 17 A 27 A 37 C 47 D
8 B 18 D 28 B 38 B 48 A
9 B 19 D 29 D 39 C 49 A
10 A 20 D 30 D 40 A 50 A
CASE BASED QUESTIONS

A J.D. Watson, an American Biologist and F.H.C. Crick, an English chemist designed the model of DNA to
explain its structure. They did not perform any new experiment, but they synthesized the data
provided by Wilkin and Franklin.DNA consists of two long parallel chains.
1. Scientists who got Nobel prize in medicine and Physiology for the structure of DNA
1. Watson
2. Crick
3. Franklin
4. Wilkin
a)1,2
b)1,2,3
c)1,2,3,4
d)1,2,4
2. Width of DNA is ---angstroms
a)20
b)30
c)34
d)2

3. In the living system which one is the largest biomolecule


a)Glycogen
b)Protein
c)RNA d)DNA
B Questions
The uniqueness of every cell or species lies in the uniqueness of its protein. The cells can synthesize a
specific type of protein. The relationship between the sequence of amino acids in a polypeptide and
nucleotide sequence of m-RNA is called genetic code.
4. Triplet codon refers to sequence of three bases on ----- RNA
a) m
b) t
c) r
d) all of the above
5. In the genetic code dictionary, how many codons are used to code for 20 amino acids ?
a)20
b)61
c)60
d)64
6. Initiation of polypeptide chain is through
a)Arginine
b)Methionine
c)Lysine
d)Leucine
C Read the passage given below and answer the following questions……..
DNA: More than just a super hard drive Bill Gates, one of the founders of Microsoft, declared, ‘DNA is
like a computer program but far, far more advanced than any software ever created. Actually, DNA is
far more complicated than simply coding for proteins, as we are discovering all the time. 1 For
example, because the DNA letters are read in groups of three, it makes a huge difference which letter
we start from. E. g. the sequence GTTCAACGCTGAA … can be read from the first letter, GTT CAA CGC
TGA A … but a totally different protein will result from starting from the second letter, TTC AAC GCT
GAA … This means that DNA can be an even more compact information storage system. This partly
explains the surprising finding of The Human Genome Project that there are ‘only’ about 35,000 genes,
when humans can manufacture over 100,000 proteins. : Bacteria and yeast are the most commonly
used hosts for the process of cloning in Human Genome Project. Not all types of fungi can be used for
this process. But yeast and bacterium can be employed. Both BAC (Bacterial Artificial Chromosomes) and
YAC (Yeast Artificial Chromosomes) act as a suitable vector for the process of cloning in HGP whereas
bacteria and yeast act as the host for cloning in HGP. The methodologies for the HGP are involved in
two major processes. One among them is ESTs (Expressed Sequence Tags). It is used to identify all the
genes that are expressed as RNA in HGP. When one base pair is stacked over the other in a helical
fashion, the DNA will be stable. A right-handed curving fashion is seen in the DNA. When repeating
structures are present, the DNA will not be stable. The sum of Purines, A and G, is equal to the sum of
Pyrimidines, C and T (i. e. , A+G = C+T). Adenine and Thymine form a double hydrogen bond. Likewise,
Guanine and Cytosine form a triple hydrogen bond. The resistance showed by thymine towards all the
photochemical mutations is what makes the DNA more stable.
7 Which is the correct complementary strand for AGAATTCGC?
a) CTCCGGATA
b) GAGGCCTAT
c) TCTTAAGCG
d) GTGGCCATA
8 Which of the following methodology is used to identify all the genes that are expressed as RNA in
Human Genome Project (HGP)?
a) Sequence Annotation
b) Expressed Sequence Tags
c) Karyotyping
d) Ammonification
9 Which of the following is a suitable host for the process of cloning in Human Genome Project (HGP)?
a) Virus
b) All types of fungi
c) Bacteria
d) Protozoan
10 Which of the following ensures the stability of the helical structure of a DNA?
a) Presence of repetitive structures of a DNA code
b) Stacking of one base pair over the other
c) Presence of aneuploidy
d) Occurrence of chromosomal rearrangements
D Read the passage given below and answer the following question:
DNA fingerprinting is a laboratory technique Used to establish a link between biological evidence and
a suspect. In a Criminal investigation. A DNA sample taken from a Crime scene is compared with a DNA
sample from a suspect. If two DNA profiles are a match then the evidence came from that suspect. It
works on the principle of polymorphism in DNA sequences. DNA is extracted from the crime scene via
WBC in blood sample or spermatozoa in semen sample and in amplified by using PCR. Easy digested
by a restriction enzyme which caught DNA into fragments at specific sites. These fragments are
separated by gel electrophoresis. The separated fragments are visualized by staining them with die
that fluorescence under UV radiations. Double Stranded DNA is then split into single. Separated DNA
sequences are transferred to a nylon or nitrocellulose sheet placed over gel. The sheet is then
immersed in a bathand probes or markers that are radioactive, synthetic DNA segments of known
sequence are added., and hybridize them.it. Degree of variation is to be high that every
individual with exceptions of identical twins, produces a unique band pattern, as every individual has a
unique set of ordinary fingerprinting.
11.Polymorphism in DNA sequences is used for.
a) DNA fingerprinting.
b) Mapping of genes.
c) Crossing over
d) None of these.
12.Profiling is used to identify
a) criminal,
b) determine paternity,
c) study migration pattern,
d) all the above.
13. Each individual has Unique fingerprinting as they differ in
a) No of satellites
b) Location of satellites.
c) Size of satellites
d) All the above
14. During DNA Fingerprinting, the digestion of DNA is done by…
a) Restriction enzyme
b) RNA polymerase enzyme.
c) Peptidyl transferase enzyme.
d) None of the above.

E The Intelligent Design


In September 2013, intelligence agencies raided a house in Zephyr Heights in Mangaluru in search of
the alleged bombers of Hyderabad Twin blasts, but found it empty. A forensic team picked up DNA
samples from the house, and when the accused were finally caught, the samples matched with theirs. It
helped NIA secure convictions for all five accused.
15 Identify the phenomenon mentioned above:
(A) NIA raid
(B) Escape of suspects
(C) DNA fingerprinting
(D) None of these.
16 DNA profiling was initially developed by __ _.
(A) Alec Jeffreys
(B) James Watson
(C) Francis Crick
(D) Matthew Meselson.
17 Select the correct sequence of the process used to identify DNA samples:
(A) DNA isolation□ hybridisation using labelled VNTR probe□Gel electrophoresis□Southern
blotting□Digestion of DNA□Autoradiography.
(B) DNA isolation□Digestion of DNA□Southern blotting□Gel electrophoresis□ hybridisation
using labelled VNTR probe□Autoradiography.
(C) DNA isolation□Autoradiography□Digestion of DNA□Gel electrophoresis□Southern
blotting□ hybridisation using labelled VNTR probe.
(D) DNA isolation□Digestion of DNA□Gel electrophoresis□Southern blotting□ hybridisation
using labelled VNTR probe□autoradiography.
18 Assertion: Polymorphism in DNA sequence is the basis of DNA fingerprinting technology. Reason:
If more than one variant (allele) at a locus occurs in human population with a frequency greater
than 0.01 is called DNA polymorphism.
(A) Both Assertion and reason are true and reason is correct explanation of assertion.
(B) Assertion and reason both are true but reason is not the correct explanation of assertion.
(C) Assertion is true, reason is false.
(D) Assertion is false, reason is true.
19 All mutations occurring in the living organisms are lethal or harmful. (true/false)

Genome Sequencing Projects


When the sequencing of the human genome was announced two decades ago by the Human Genome
Project and biotech firm Celera Genomics, the sequence was not truly complete. About 15% was
missing: technological limitations left researchers unable to work out how certain stretches of DNA
fitted together. Researchers in the Telomere-to-Telomere (T2T) Consortium, an international
collaboration that comprises around 30 institutions, have filled in those gaps. Genomics researcher
Karen Miga at the University of California, Santa Cruz, and her colleagues report to have added 200
million DNA base pairs and 115 protein-coding genes — but they’ve yet to entirely sequence the Y
F chromosome.
20 What does the term genome mean?
(A) All the genetic material in the chromosomes of a particular population of organisms
(B) All the genetic material in the chromosomes of a particular organism
(C) The total metabolic content of a cell/organism
(D) All of these.
21 Select a correct statement about sequence annotation method.
(A) It’s a blind approach
(B) It’s a target specific approach
(C) It is cumbersome and time consuming approach
(D) Both A & C.
22 Read the following statements.
(i) Identify all the approximately 20,000-25,000 genes in human DNA
(ii) Determine the sequences of the 3 billion chemical base pairs thatmake up human DNA
(iiii) Store this information in huge volumes of printed books
(iv) Improve tools for data analysis
Select the statements which are correct about goals of HGP.
(A) Only (i)
(B) Both (i) & (ii)
(C) (i), (ii) and (iii)
(D)(i), (ii) and (iv)
23 Assertion: Polymorphism in DNA is the basis for genetic mapping of human genome as well as DNA
fingerprinting.
Reason: Polymorphism in DNA arises due to mutations.
(A) Both Assertion and reason are true and reason is correct explanation of assertion.
(B) Assertion and reason both are true but reason is not the correct explanation of assertion.
(C) Assertion is true, reason is false.
(D) Assertion is false, reason is true.
G Double Helix Model Given for the DNA Structure
● In 1953, Watson and Crick suggested the DNA double-helix structure.
● The ratio of adenine and thymine to that of guanine and cytosine is one and tends to
remain constant. This was stated by Ervin Chargaff.
● DNA is formed by two polynucleotide chains, where the backbone is sugar-phosphate and bases
are present within.
● There exists opposite polarity between the two chains, i.e. and the one with 3'-5' polarity and the
other with 5'-3' polarity.
● The base pair (bp) is created by the formation of hydrogen bonding amongst the nitrogen bases
that present on two of the polypeptide chains.
● To make a base pair, a purine base of one nucleotide chain is often connected to a pyrimidine
base of another nucleotide chain or vice versa.
● Two hydrogen bonds (A=T) pair Adenine with Thymine (or Uracil in RNA) while three
hydrogen bonds pair Guanine with Cytosine (G).

24 The ratio of purine and pyrimidine should be-


a) One :one
b) Two :one
c) Three :one
d) None of the above
25 How many Hydrogen bonds between cytosine and guanine-
a) One
b) Two
c) Three
d) Four

H Read the following and answer any four questions from21(i) to 21(v) given below:

DNA is the genetic material of almost all the organisms. One of the active functions of DNA is to make its
copies which are transmitted to the daughter cells. Replication is the process by which DNA makes exact
copies of itself. Replication is the basis of life and takes place during the interphase stage. Watson and
Crick suggested the semiconservative method of replication of DNA. This has been proved by Messelson
and Stahl’s in their experiments on Escherichia coli using radioactive isotopes. The replication of
chromosome in E. coli is completed in 40 minutes. During replication of DNA, the two complementary
strand of DNA uncoil and separate from one end in a zipper like fashion. The enzyme helicase unwinds
the two strands and as a result replication fork is formed. As the DNA unwinds, the part of the DNA that
is found above the replication fork becomes supercoils. These are called positive supercoils. An enzyme
called topoisomerase releases these supercoils. Based on separated DNA strands, new strands grow
by the addition of nucleotides. DNA polymerase I, II and III are involved in this elongation. However,
these enzymes are not capable of initiating DNA synthesis. For the synthesis of new DNA, two things are
required. One is RNA primer and the enzyme primase. The DNA polymerase moves along the newly
formed RNA primer nucleotides, which leads to the elongation of DNA. In the other strand, DNA is
synthesized in small fragments called Okazaki fragments. These fragments are linked by the enzyme
called ligase. In the resulting DNA, one of the strand is parental and the other is the newer strand
which is formed discontinuously. Hence, it is called semidiscontinuous replication.
26 During semiconservative replication of DNA the enzyme responsible for replication is _ .

a) RNA Polymerase
b) DNA dependent RNA Polymerase
c) RNA Dependent DNA Polymerase
d) DNA Dependent DNA polymerase
27 Which statement is untrue for semi conservative replication of DNA

a) On 5’□3’ prime of DNA multiple primers are formed


b) On 3’□5’ prime of DNA Okajakifragments are formed
c) The enzyme Primase is responsible for formation of Primers
d) Okajaki fragments are formed on 5’□3’ prime of DNA
28 The small fragments that is formed on the 5’□3’ prime of DNA is called as .

a) Lagging strand
b) Leading strand
c) Okajaki fragments
d) Primer
29 The enzyme that is responsible for and uncoiling of DNA and synthesis of new strand of DNA on DNA
templet are

a) Topoisomerase and DNA Dependent DNA polymerase


b) Topoisomerase and DNA Dependent RNA polymerase
c) Primase and DNA Dependent DNA polymerase
d) Primase and DNA Dependent DNA polymerase
30 The scientists who proved semi conservative replication of DNA are

a) Harshey and Chase


b) Watson and Crick
c) Meselson and Stahl
d) Har Gobind Khorana and Nirenberg
I Read the following and answer ay four questions from 31(i) to 35(v) given below:

DNA fingerprinting involves identifying differences in some specific regions in DNA sequence called as
repetitive DNA, because in these sequences, a small stretch of DNA is repeated many times. These
repetitive DNA are separated from bulk genomic DNA as different peaks during density gradient
centrifugation. The bulk DNA forms a major peak and the other small peaks are referred to as satellite
DNA. Depending on base composition (A : T rich or G:C rich), length of segment, and number of
repetitive units, the satellite DNA is classified into many categories, such as micro-satellites, mini-
satellites etc. These sequences normally do not code for any proteins, but they form a large portion of human
genome. These sequence show high degree of polymorphism and form the basis of DNA fingerprinting. Since
DNA from every tissue (such as blood, hair-follicle, skin, bone, saliva, sperm etc.), from an individual show
the same degree of polymorphism, they become very useful identification tool in forensic applications.
Further, as the polymorphisms are inheritable from parents to children, DNA fingerprinting is the basis of
paternity testing, in case of disputes. The technique of DNA Fingerprinting was initially developed by Alec
Jeffreys. He used a satellite DNA as probe that shows very high degree of polymorphism. It was called as
Variable Number of Tandem Repeats (VNTR). The technique, as used earlier, involved Southern blot
hybridisation using radiolabelled VNTR as a probe.

31 DNA fingerprinting cannot be done from the following sample

a) Fragment of Nail
b) Blood
c) Follicle of nail
d) Skin

32 Which statement is not true for DNA fingerprinting as developed by Alec Jefreysis

a) Finding the VNTR sequences


b) Isolation and staining of DNA
c) 33Use of density gradient centrifugation
d) De34tection of Polymorphism in DNA sequences

33 The process involved in increasing the number of DNA for analysis is _ .

a) Southern blotting technique


b) Polymerase Chain reaction
c) Hybridization using labeled VNTR probe
d) Use of autoradiography
34 Match the following
1 VNTR 5 transferring (blotting) of separated
DNA fragments to synthetic
membranes, such as nitrocellulose or
Nylon
2 Southern Blotting 6 Separation of DNA fragments
3 Restriction endonucleases 7 Size from 0.1 to 20 kb.
4 Electrophorosis 8 Digestion of DNA fragments
a) 1 and 7, 2 and 5, 3 and 8 , 4 and 6
b) 1 and 8, 2 and 5, 3 and 7 , 4 and 6
c) 1 and 7, 2 and 6, 3 and 8 , 4 and 5
d) 1 and 5, 2 and 7, 3 and 8 , 4 and 6

35 Which of the statement is incorrect for repetitive DNA?


a) They are a small stretch of DNA which is repeated many times.
b) They are separated from bulk genomic DNA as different peaks during density
gradient centrifugation.
c) These sequences normally do not code for any proteins
d) These sequence do not show high degree of polymorphism

J Read the passage given below and answer the following questions based on it.
III. The DNA replication is semi-conservative is proved by an experiment conducted by Meselson and Stahl
in 1958. To perform their experiment they use heavy nitrogen (¹5N) in E. coli. The process of replication in
living cells requires a set of enzymes. The main enzyme is DNA dependent DNA polymerase. The DNA-A
dependent DNA polymerase catalyse polymerization only in one direction, that is 5-3'. In eukaryotes, the
replication of DNA takes place at the S-phase of the cell cycle.
36 Viruses grown in the presence of radioactive phosphorus contained radioactive ------------------ but not --

(a) DNA, Protein (b)Protein ,


DNA
(c) RNA, Nucleoside (d)mRNA
, Protein
37 During DNA replication, the breaking of H-bonds is performed by
(a) Topoisomerase
(b) Gyrase
(c) Helicases
(d) None.
38 How many types of DNA polymerases are associated with eukaryotic cell ?
(a) Three
(b) Six
(c) Five
(d) One.
39 DNA replication is :
(a) Semi-conservative, continuous
(b) Semi-continuous, conservative
(c) Semi-conservative, semi-discontinuous
(d) Conservative.
40 Directions: In the following questions a statement of
assertion (A) is followed by a statement of reason (R), Mark
the correct choice as :
(A) Both assertion (A) and reason (R) are true and reason (R) is the correct explanation of assertion(A).
(B) Both assertion (A) and reason (R) are true but reason (R) is not the correct explanation ofassertion (A).
(C) Assertion (A) is true but reason (R) is false.
(D) Assertion (A) is false but reason (R) is true.
Assertion (A): Teminism is bidirectional flow of information Reason (R):
It requires DNA dependent RNA polymerase
K Read the following and answer the questions given below:

THE LAC OPERON


The elucidation of the lac operon was also a result of a close association between a geneticist, Francois
Jacob and a biochemist, Jacque Monod. They were the first to elucidate a transcriptionally regulated
system. In lac operon, a polycistronic structural gene is regulated by a common promoter and regulatory
genes. Such arrangement is very common in bacteria and is referred to as operon, e.g., lac operon, trp
operon, ara operon, his operon, val operon, etc. The lac operon consists of one regulatory gene (the i gene
– here the term i does not refer to inducer, rather it is derived from the word inhibitor) and three structural
genes (z, y, and a). The i gene codes for the repressor protein of the lac operon. The “z” gene codes for beta-
galactosidase (β -gal), which is primarily responsible for the hydrolysis of the disaccharide, lactose into its
monomeric units, galactose and glucose. The “y” gene codes for permease, which increases permeability of
the cell to β -galactosides. The “a” gene encodes a transacetylase to transfer the acetyl group.

41 In ‘Lac Operon’ repressor protein attaches to


(a) Operator
(b) Promoter
(c) Regulator
(d) β -galactosidase
42 In the Lac operon system, β-galactosidase is coded by
(a) a-gene
(b) i-gene
(c) 1-gene
(d) y- gene

43 The lac operon consists of


(a) four regulatory genes only
(b) one regulatory gene and three structural genes
(c) two regulatory genes and two structural genes
(d) three regulatory genes and three structural genes
44 In lac operon, i gene codes for :
(a) Inducer of lac operon
(b) Repressor of lac operon
(c) Hydrolysis of disaccharide
(d) Permease
45 Select the two correct statements out of the four (1 – 4), given below about lac operon.
(1) Glucose or galactose may bind with the repressor and inactivate it
(2) In the absence of lactose the repressor binds with the operator region
(3) The z-gene codes for permease
(4) This was elucidated by Francois Jacob and Jacque Monod The
correct statements are
(a) 1 and 4
(b) 2 and 3
(c) 1 and 3
(d) 2 and 4
L -“DNA a long polymer of deoxyribonucleotide . Altman and these substance s to be acidic hence he
named nucleic acid . The basic need of DNA is a nucleotide which has three components – a

nitrogenous base , a pentose sugar and a phosphate group. There are two types of nitrogen bases in DNA ,
Purine and Pyrimidine . J. Watson and F crick proposed a double helix model for the structure of DNA.
There are four types of DNA i.e. A,B,C,Z.
46 Which DNA form has maximum number of base pairs per turn. A)A-
DNA
b)B-DNA C)C-
DNA D)Z-DNA
47 Which among the following doesnot confer stability to the helical structure of DNA.
A)Phosphodiester Bond b)H-
Bond
C) N-Glycosidic Linkage D)All
of these
48 Cytidine is a
A)Nucleoside
B)Nytrogen base
C)Nucleotide
D) Common dinucleotide In DNA and RNA
49 Heaviest molecule of protoplasm is
A)Lipids
B)Proteins C)DNA
D)RNA
M CASE
Have you ever written a secret message to one of your friends? If so, you may have used some kind of code
to keep the message hidden. Similarly decoding messages is also a key step in gene expression, . In this
process DNA is transcribed and translated into amino acids they represent.
To crack the genetic code, researchers needed to figure out how sequences of nucleotides in a DNA or RNA
molecule could encode the sequence of amino acids in a polypeptide. In the mid-1950s, the physicist
George Gamow extended this line of thinking to deduce that the genetic code was likely composed of three
of nucleotides.
The cracking of the genetic code began in 1961, with work from the American biochemist Marshall
Nirenberg. First, Nirenberg synthesized an mRNA molecule consisting only of the nucleotide uracil (called
poly-U). When he added poly-U mRNA to the cell-free system, he found that the polypeptides made
consisted exclusively of the amino acid phenylalanine. Other researchers, such as the biochemist Har
Gobind Khorana ,extended Nirenberg's experiment by synthesizing artificial mRNAs with more complex
sequences. By 1965, using the cell-free system and other techniques, Nirenberg, Khorana, and their
colleagues had deciphered the entire genetic code. That is, they had identified the amino acid or "stop"
signal corresponding to each one of the 646464 nucleotide codons. For their contributions, Nirenberg and
Khorana (along with another genetic code researcher, Robert Holley) received the Nobel Prize in 1968.

50 Genetic code is?


A) Singlet
B) Duplet
C) Triplet
D) Quadruplet
51 Which among the following is the function of AUG.
A) Initiator codon
B) Codes for methionine
C) Stop codon
D) A and B both
52 Genetic code is degenerate means?
A) One code codes for only one amino acid.
B) Some amino acids are coded by more than one codon
C) Codons can ne exchanged for amino acids
D) Both C and B
53 Which among the following is not a stop codon.
A) UAA
B) UAG
C) Both A and B
D) UUA
54 CUU always codes for leucin, which property of genetic code is this?
A) Degenerate
B) Ambiguous
C) Universal
D) Both A and B
N FINDING THE STRUCTURE OF DNA
In 1951, James Watson, an American biologist, began an internship at the University of Francis Crick , a
British physicist, who was interested in molecular structures. Together they set out to determine the
structure of DNA and to build a model that would explain how DNA, the genetic material.
The bits and pieces of data available to Watson and Crick were like puzzle pieces they had to fit
together. This is what they knew from the research of others.
1. DNA is a polymer nucleotides, each one having a phosphate group, the sugar deoxyribose, and a nitrogen
containing base. There are four types of nucleotides because there are different base: (A) and guanine (G)
and purines, while cytosine(C) and thymine (T) are pyrimidines.
2. A chemist, Erwin Chargaff, had determines in the late 1940s that regardless of the species under
consideration the number of purines in DNA always equals the number of pyrimidines. Further, the amount
of adenine equals the amount of Thymine (A=T), and the amount of guanine equals the amount of cytosine
(G=C). These findings came to be known as Chargaff’s rules.
3. Rosalind, London , had just prepared an X-ray diffraction Photograph of DNA. It showed that DNA is a
double helix of constant diameter and that the bases are regularly stacked on top of one another.
Using these data Watson and Crick deducted that DNA has a twisted, ladder-type structure; the sugar-
phosphate molecules make up the slides of the ladder, and the bases make up the rungs. Further, they
determined that if A is normally hydrogen bonded with T , and G is normally hydrogen bonded with C(in
keeping with Chargaff’s rules), then the rungs always have a constant width (as required by the X- ray
photograph).
Watson and Crick built an actual model of DNA out of wire and tin. This double helix model does indeed
allow for differences in DNA structure between species because the bad pairs can be in any order. Also the
model suggest that complementary base pairing plays a role in the replication of DNA. As Watson and Crick
pointed out in their paper.

55 Which scientist determine the structure of DNA.


A. Francis Crick B
Watson
C Watson and crick
D. Rosalind Franklin and Mourice Wilkins
56 Name the scientist proposed double helical model of DNA. A
Francis Crick
B Watson
C Watson and crick
D Rosalind Franklin and Mourice Wilkins
57 complementary base pairing is proposed by . A
Francis Crick
B Watson
C. Erwin Chargaff
D Rosalind Franklin and Mourice Wilkins
58 Strands of B DNA is twisted in which fashion.
A. Right-handed
B. Left-handed
C. zig-zag
D. opposite
59 what is true about DNA.
A) It is a Polynucleotide chain
B) It is Polynucleoside chain
C) It is a Polymer of sugar and phosphate
D) It is polymer of phosphate and nitrogenous base

ANSWERS CASE STUDY


1 D 11 A 21 D 31 A 41 A 51 D
2 A 12 D 22 D 32 B 42 A 52 B

3 D 13 D 23 B 33 B 43 D 53 D

4 A 14 A 24 A 34 A 44 A 54 C
5 B 15 C 25 C 35 D 45 C 55 D
6 B 16 A 26 C 36 A 46 A 56 C
7 C 17 B 27 B 37 C 47 C 57 C
8 B 18 B 28 C 38 C 48 C 58 A
9 C 19 F 29 C 39 C 49 C 59 A
10 20 30 40 50
B B C C C
CHAPTER 7: EVOLUTION

MASTER CARD
GIST CONTENT
Origin of Universe • Big Bang Theory

Theories of Origin of Life • Theory of special creation


• Theory of Panspermia/Cosmozoic
Theory.
• Theory of Spontaneous generation
• Theory of Chemical Evolution.
Experimental Evidence of Chemical Evolution. • Miller’s Experiment

Evidences for Evolution • Paleontological Evidence


• Morphological and Comparative
anatomical Evidence
• Biochemical Evidence
• Bio geographical Evidence
Adaptive Radiation • Darwin Finches
• Australian mammals
• Australian Marsupials
Theories of Evolution • Lamark’s Theory
• Darwin’s Theory
• Mutation Theory

Hardey-Weinberg Principle • Hardey-Weinberg Equilibrium


• Factors affecting hardey-Weinberg
Equilibrium.

Brief account of Evolution • Evolution of plants


• Evolution of animals
• Evolution of man.
Evolution Evolution is a process of gradual change that takes place over
many generations, during which species of animals, plants, or
insects slowly change some of their physical characteristics
Big bang theory The single huge explosion resulted in the origin of the universe
about 20 billion years ago. The earth was formed about 4.5
billion years ago. There was no atmosphere on early earth.
Water vapor, CH4, CO2 & NH3 released from molten mass
covered the surface. The UV rays from the sun broke up water
into Hydrogen and Oxygen and the lighter H2 escaped. Oxygen
is combined with ammonia and methane to form water, CO2,
etc. The ozone layer was formed. As it cooled, the water vapor
fell as rain and form oceans. Life appeared four billion years
back.
Theories on Origin of Life
Theory of spontaneous It states that life came out of decaying and rotting matter like
generation (Abiogenesis) straw, mud, etc.
Theory of biogenesis Proposed by Louis Pasteur. He demonstrated that life comes
only from pre-existing life
Theory of Panspermia It states that the units of life (spores) were transferred to
different planets including earth.
Theory of special creation It states that living things were created by some supernatural
power
Theory of chemical Proposed by Oparin& Haldane. It states that the first form of
evolution life originated from non-living inorganic & organic molecules
Harold Urey & Stanley Miller They experimentally proved the theory of chemical evolution.
Experiment They created a condition like that of primitive earth (i.e. high
temperature, volcanic storms, reducing atmosphere with CH4,
NH3, H2O, H2, etc). When an electric discharge is produced in
a closed flask containing CH4, NH3, H2, and water vapor, at
800o C biomolecules (amino acids) like present-day were
formed
Evidences for Evolution
Paleontological Evidence - Fossils: Fossils are remnants of life forms found in rocks
(earth crust). The study of fossils is known as Palaeontology.
• Fossils provide evidence for phylogeny (evolutionary
history or race history). E.g., Horse evolution, Used to
study the connecting link between two groups of
organisms (E.g. Archaeopteryx), to study extinct animals
(E.g. Dinosaurs), to study geological periods by analysing
fossils in different sedimentary rock layers.
Morphological & Anatomical Homologous organs
evidences • These organs have similar structures and origins but
different functions. The origin of homologous organs is
due to Divergent evolution.
Analogous organs
• These organs have similar functions but different
structures & origins. The origin of analogous organs is
due to Convergent evolution.

Adaptive radiation This is the evolution of different species from an ancestor in a


geographical area starting from a point. It is a type of
divergent evolution. E.g., Darwin’s finches in Galapagos
Islands, Australian marsupials (Marsupial radiation), Placental
mammals in Australia.

Embryological evidences • He observed that all vertebrate embryos have some common
Proposed by Ernst Haeckel features that are absent in adults.
Biochemical evidence Organisms show similarities in proteins, genes, other
biomolecules & metabolism. It indicates common ancestry
Theories on Evolution

Lamarckism (Theory of It states that the evolution of life forms occurred by the
Inheritance of Acquired inheritance of acquired characters. Acquired characters are
characters) developed by the use & disuse of organs.

Darwinism (Theory of • Proposed by Charles Darwin.


Natural selection) • It was based on observations during a sea voyage isailingsail
ship called H.M.S. Beagle.
• Alfred Wallace (who worked in the Malay Archipelago) had
also come to similar conclusions.
• Work of Thomas Malthus on populations influenced Darwin.
• Darwinism is based on 2 key concepts: Branching descent
and Natural selection Natural selection is based on the
following facts: • Heritable minor variations
• Limited natural resources
• Struggle for existence
• Survival of the fittest
Mutation Theory • Hugo de Vries proposed the Mutation Theory of evolution.
• He conducted experiments evening primrose

KEY WORDS
Evolution This is a process of gradual change that takes place over many generations, during which
species of animals, plants, or insects slowly change some of their physical characteristics
Big bang theory Explains the origin of the earth
Homologous organs organs having the same basic structure but different functions.
Analogous organs organs having different basic structures but having similar appearances and performing
similar functions.
Adaptive radiation This is an evolutionary process of organisms that are grouped into a wide variety of types
adapted to specialized modes of life.
branching descent The process of developing a new species from a single common descendant is known as
branching descent
Natural selection The process through which populations of living organisms adapt and change
Saltation Single-step large mutation
Mya Million years ago
.

THEORIES OF THE ORIGIN OF LIFE

Natural selection is based on the following facts:

Heritable minor Limited natural resources Struggle for existence Survival of the fittest
variations

Darwin Finches- Darwin observed many varieties of finches on the same island. All varieties of finches had evolved
from original seed-eating finches. There was an alternation in beaks enabling some to become insectivorous and some
vegetarian.

MUTATION
• Hugo de Vries proposed the Mutation Theory of evolution.
• He conducted experiments in evening primrose
HARDY-WEINBERG PRINCIPLE
• It states that allele frequencies in a population are stable and are constant from generation to generation in
the absence of disturbing factors.
• the allelic frequency in a population will remain constant from one generation to the next in the absence of
disturbing factors.
Equation:

• Sum total of all the allelic frequencies = 1


p2 + 2pq + q2 = 1
Factors affecting Hardy-Weinberg equilibrium

Gene Migration Genetic drift Mutation Genetic recombination Natural selection


Types of Natural selection:

Stabilizing
More individuals acquire mean
selection
character value.
Directional
Individuals of one extreme are
selection
more favoured.
Disruptive
It breaks a homogenous
selection
population into many different
forms. Individuals of both
extremes are more favoured
ORIGIN AND EVOLUTION OF MAN
Dryopithecus and About 15 mya, primates (Dryopithecus and Ramapithecus) were existing.
Ramapithecus They were hairy and walked like gorillas and chimpanzees.
Ramapithecus was more man-like while Dryopithecus was more ape-like.
Australopithecus About two mya, Australopithecines probably lived in East African grasslands.
They hunted with stone weapons and essentially ate the fruit.
Homo habilis Brain capacities - 650-800cc., They probably did not eat meat.
Homo erectus About 1.5 mya, Homo erectus arose.
Brain capacity- 900cc.Homo erectus probably ate meat.
Neanderthal man Brain size- 1400cc. They used hides to protect their body and buried their dead.
Homo sapiens or Arose in Africa. During the ice age between 75,000-10,000 years ago, Homo sapiens
modern man arose.
Prehistoric cave art developed about 18,000 years ago.
Evidence -Bhimbetka rock shelter in Raisen district of Madhya Pradesh.
Agriculture came around 10,000 years back and human settlements started.
KEY WORDS
Evolution This is a process of gradual change that takes place over many generations, during which
species of animals, plants, or insects slowly change some of their physical characteristics
Big bang theory Explains the origin of the earth
Homologous organs organs having the same basic structure but different functions.
Analogous organs organs having different basic structures but having similar appearances and performing
similar functions.
Adaptive radiation This is an evolutionary process of organisms that are grouped into a wide variety of types
adapted to specialized modes of life.
branching descent The process of developing a new species from a single common descendant is known as
branching descent
Natural selection The process through which populations of living organisms adapt and change
Saltation Single-step large mutation
mya Million years ago
IMPORTANT QUESTIONS

Very Short Answer Type Questions/ MCQ

1- Who used pre-sterilized flasks and yeast to disprove the theory of abiogenesis-

a- Pasteur
b- Darwin
c- Lamarck
d- Morgan
Ans: Louis Pasteur
2- What was the observation of Ernst Heckel regarding evidence of the origin of life?

a- Embryological support for evolution


b- Finches
c- Homology
d- Convergent evolution
Ans: A
3- What is the basis of the Darwinian theory of evolution?

a- Natural selection
b- Branching Descent
c- Mutation
d- Both a and b
Ans: D
4- (p+q)2 = p2 + 2pq + q2 = 1 represents an equation used in
a- Population genetics
b- Cytogenetics
c- bioinformatics
d- Anthropology
Answer: A
5- The abiogenesis theory of origin supports
a- spontaneous generation
b- origin of life from blue-green algae
c- origin of life is due to pre-existing organisms
d-0020 organic evolution is due to chemical reactions.
Ans: A
6- What postulate was given by Oparin and Haldane about the origin of life?
Ans: The first form of life could have come from pre-existing non-living organic molecules. evolution.
7- Which scientist come to the same conclusion as Darwin?
Ans: Alfred Wallace (worked in the Malay Archipelago)
8- Give two examples of adaptive radiation.
And: Darwin finches, Australian marsupials.
9- What is speciation? What is the main cause of speciation according to Hugo DeVries?
Ans: speciation is the formation of new species.
The mutation is the main cause of speciation.
10- Would you consider the wings of a butterfly and a bat as homologous or analogous?
Ans: It is an example of the analogous organ as the basic structure is different but the function is the same.
11- State the significance of biochemical similarities among diverse organisms in evolution.
Ans: Similarities in biochemicals like DNA, help in deriving the line of evolution. Organisms with more similar
DNA sequences show the same ancestor.
12- What do you mean by panspermia?
Ans: Early Greek thinkers thought units of life called spores were transferred to different planets including
Earth.
Short answer types of questions

1- What is the founder effect? Mention its importance,


Ans: When a population gets separated from the existing population it becomes the founder of a new
population. This is known as the founder effect. The founder effect affects the genetic diversity of a population.
2- Explain convergent evolution by giving a suitable example.
Ans: when two different species with different ancestors tend to develop similar traits during the course of
evolution Examples: Some of the marsupials of Australia resemble placental mammals that live in similar
habitats to other continents.
3- Illustrate contribution of the Lamarck to the understanding of evolution?
Ans: he proposed the theory of Inheritance of Acquired characters, Use, and disuse of organs.
4- identify the following pairs as homologous of analogous organs:
(i)Sweet potato and potato (ii) Eye of Octopus and eye of mammals.
(iii)Thorns of Bougainvillea and tendrils of Cucurbits (iv)Forelimbs of bat and whale.
Ans: (i) and (ii) Analogous organs, (iii) and (iv) Homologous organs.
5- Compare the brain capacity of Homo habilis and Homo erectus.
Ans: Homo habilis- 650-800cc, Homo erectus- 900cc
Long Answer Type Question

1- Describe the three different ways by which natural selection can affect the frequency of a heritable trait in
a population. Provide a diagram also in support of your answer.
Ans- The three different ways by which natural selection can affect the frequency of a heritable trait in a
population are
(i) Stabilisation It results in a greater number of individuals acquiring the mean character value, i.e., variation
is much reduced.
(ii) Directional change It results in more individuals acquiring value other than mean character value, i.e., the
peak shift towards one direction.
(iii) Disruption In this more individuals acquire peripheral character value at both ends of the distribution curve,
i.e., two peaks are formed at the periphery
Fig, 7.8 page 136, NCERT
2- Diagrammatic representation of Miller’s experiment and label any ten parts.
Ans: fig 7.1, page 128 NCERT
3- Which law states that the sum of allelic frequencies in a population is constant? Write its mathematical
formula used to derive allelic frequency. List the five factors that influence the law.
Ans: The law is Hardy-Weinberg equilibrium.
Mathematical equation- P2 + 2pq + q 2= 1 where p and q are allelic frequencies
Factors affecting are - Genetic drift, Mutation, Gene flow, Genetic Recombination, Natural Selection
4- Study the figures given below & answer the following question.
i) Under the influence of which type of natural
selection would graph (a) become like (b).
ii) What could be the likely reason for new
variations arising in a population.
iii) Who suggested natural selection as the
mechanism of evolution?
Ans: (i) Disruptive & elective.
(ii) Individuals at the extremes contribute more offspring compared to those in the centre & produce two peaks
in the distribution of a trait.
(iii) Charles Darwin
5- Explain the evolution of human beings in a sequential manner.
Ans: Dryopithecus and Ramapithecus - about 15 mya, primates called Dryopithecus and Ramapithecus were
existing. They were hairy and walked like gorillas and chimpanzees. Ramapithecus was more man-like while
Dryopithecus was more ape-like.
Australopithecus- about two mya, Australopithecines probably lived in East African grasslands, they hunted
with stone weapons, essentially ate the fruit.
Homo habilis- Brain capacities - 650-800cc, they probably did not eat meat.
Homo erectus- about 1.5 mya, brain capacity- 900cc., Homo erectus probably ate meat.
Neanderthal man- brain size- 1400cc, they used to bury their dead.
Homo sapiens or modern man- Arose in Africa, During the ice age between 75,000-10,000 years ago.

ADDITIONAL MULTIPLE CHOICE QUESTIONS:-1


1. Extra-terrestrial origin of life was proposed by theory of
(a) Catastrophism
(b) Spontaneous Generation
(c) Special Creation
(d) Panspermia
2. Miller’s experiment provided evidence for the theory of
(a) Special Creation
(b) Biogenesis
(c) Biogenesis followed by Abiogenesis
(d) Chemical Evolution
3. Who said that ‘Life arises from pre-existing life’?
(a) Aristotle
(b) Louis Pasteur
(c) Oparin
(d) Darwin
4. Hardy-Weinberg principle is mainly applicable to
(a) Small Population
(b) Isolated population
(c) Large population
(d) All of these
5. Which of the following is considered as a pollution indicator in light of industrial melanism?
(a) Frequency of melanic moth
(b) Lichen
(c) Predator birds
(d) Frequency of light coloured moth
6. Reproductive isolation is
(a) Inability to interbreed in a natural population
(b) Ability to interbreed in a natural population
(c) Breeding in isolation
(d) Intraspecific breeding
7. Which is most important for speciation?
(a) Ecological isolation
(b) Reproductive isolation
(c) Temporal isolation
(d) Behavioral isolation
8. Which of the following is a direct ancestor of Homosapiens?
(a) Homo erectus
(b) Australopithecus
(c) Homo sapiens neanderthalensis
(d) Ramapithecus
9. If nature selects more individuals with mean characteristics compared to variants, which kind of
selection is said to be operating/
(a) Stabilising
(b) Directional
(c) Disruptive
(d) Balancing
10. Which of the following is not an Australian Marsupial?
(a) Kangaroo
(b) Wombat
(c) Lemur
(d) Bandicoot
SOLUTION:
1.d 2.d 3.b 4.c 5.b 6.a 7.b 8.a 9.a 10.c
Case Study Questions-1
Ques 1

(a) State the hypothesis which S.L. Miller tried to prove in the laboratory with the help of the set
up given above.
(b) Name the organic compound observed by him in the liquid water after running the above
experiment.
(c) A scientist simulated a similar set up and added CH4, NH3 and water vapour at 800°C.
Which important component is missing in his experiment?
CASE BASED 2

The forelimbs of four vertebrates are shown in the diagram shown above.
(a) What type of evolution is exhibited by the similarity among these organs in those
organisms?
(b) What are such organs known as?
(c) What do they indicate?

CASE BASED 3
In 1950s, there were hardly any mosquitoes in Delhi. The use of the pesticide, DDT on
standing water
killed their larvae. But, now there are mosquitoes because they have evolved DDT-
resistance through the interaction of mutation and Natural selection. State in a sequences,
how that could have happened.When the reptiles came down, mammals took over the earth.
There' were mammals in South America, which resembled some of the modern day
mammals. But due to continental drift, they disappeared whereas the pouched mammals of
Australia flourished and evolved into the various forms of pouched mammals that we see
today.
(a) Mention two characteristic features that were the reasons for the successful existence of
mammals on earth.
(b) Why did the continental drift affect the mammals of South America and Australia,
differently.

CASE BASED 4
According to Hardy-Weinberg principle, the allele frequencies in a population are stable and remain
constant through generations. When the frequency differs from the expected values, the difference
indicates the extent (direction) of evolutionary change. Disturbance in the genetic equilibrium or
Hardy-Weinberg equilibrium in a population can be interpreted as resulting in evolution.
(a) Write the algebraic equation representing Hardy-Weinberg equilibrium.
(b) List the five factors that affect the genetic equilibrium.
ANSWERS
1. (a) S.L. Miller tried to prove the hypothesis of Oparin and Haldane; it is as follows:
(i) The first from of life could have come from the pre-existing non-living organic molecules like
RNA, proteins, etc.
(ii) Formation of life was preceded by chemical evolution that resulted in the formation of diverse
organic molecules from inorganic constituents.
(b) Amino acids.
(c) Hydrogen is missing.
2. (a) Divergent evolution.
(b) Homologous organs.
(c) Homology indicates common ancestry.

1.(i) There were some larvae with a muttant gene that conferred resistance to DDT.
(ii) The DDT-resistant larvae survived while the others died.
(iii) The DDT-resistant larvae reached adulthood and reproduced in large numbers.
(iv) The progeny also consisted mostly of DDT-resistant larvae.
(v) Natural selection operating over a number of generations, favoured the DDT-resistant
mosquitoes to reproduce in large numbers.
(vi) Hence, today there is a large number of mosquitoes that are resistant to DDT.
2. (a) (i) Most of the mammals were viviparous and protected their unborn young ones inside the
mother's body.
(ii) With increased brain size, they became intelligent in sensing and avoiding danger.
(b) (i) Due to continental drift, when South America joined North America, the South American
mammals were overridden by those of North America.
(ii) But Australia became separated and due to lack of competition from any other mammal, the
pouched mammals flourished and evolved.
1. (a) (p + q)2 or p2 + 2pq + q2 = I.
(b) The factors include:
(i) Gene migration/gene flow
(ii) Genetic drift
(iii) Mutation
(iv) Genetic recombination
(v) Natural selection .
ASSERTION AND REASON BASED QUESTIONS
Directions: In the following questions, a statement of assertion is followed by a
statement of reason.
Mark the correct choice as:
(a) If both Assertion and Reason are true and Reason is the correct explanation
of Assertion.
(b) If both Assertion and Reason are true but Reason is not the correct
explanation of Assertion.
(c) If Assertion is true but Reason is false.
(d) If both Assertion and Reason are false.
Q.1. Assertion: According to big-bang hypothesis about 20 billion years ago universe
was a big ball of only neutrons.
Reason: Movement of these particles is known to generate tremendous heat which
caused explosion due to temperature and pressure changes.
Q.2. Assertion: Big-bang theory is based on studies of Sir James Jeans.
Reason: He gave the theory of steady state.
Q.3. Assertion: Milky way is the galaxy in the universe.
Reason: Our Earth is part of milky way.
Q.4. Assertion: The primitive atmosphere was reducing once i.e., without oxygen.
Reason: In the primitive atmosphere, oxygen was involved in forming ozone. [AIIMS
2009]
Q.5. Assertion: Organic compounds first evolved in earth required for origin of life were
protein and nucleic acid.
Reason: All life forms were in water environment only. [AIIMS 2016]
Q.6. Assertion: Theory of chemical evolution proposed that life comes from pre-
existing non-living organic molecules.
Reason: The primitive earth conditions led to production of organic molecules.
Q.7. Assertion: Louis Pasteur showed that in flask open to air, new living organisms
appeared in the heat killed yeast culture.
Reason: Life arises from pre-existing life.
Q.8. Assertion: Primitive atmosphere was of reducing type.
Reason: First hydrogen atoms combined with all oxygen.
Q.9. Assertion: Stanley Miller could work on experimental evidence of origin of life
because of Harold Urey.
Reason: H. Urey was geochemist, cosmo chemist and teacher of S. Miller.
Q.10. Assertion: Darwin’s finches show a variety of beaks suited for eating large
seeds, flying insects and cactus seeds.
Reason: Ancestral seed-eating stock of Darwin’s finches radiated out from South
America main land to different geographical areas of the Galapagos Islands, where
they found competitor-free new habitats.
Q.11. Assertion: Coacervates are believed to be the precursors of life.
Reason: Coacervates were self-duplicating aggregates of proteins surrounded by lipid
molecules. [AIIMS 2004]
Q.12. Assertion: The first cells used RNA as their hereditary molecule.
Reason: DNA evolved from RNA.
Q.13. Assertion: The first molecules formed for replicating cells were most probably
RNA.
Reason: This was proved by origin of ribozyme T. Cech in Tetrahymena.
Q.14. Assertion: We have lost all the direct evidence of origin of life.
Reason: The persons responsible for protecting evidences were not skilled. [AIIMS
1998]
Q.15. Assertion: The earliest organisms that appeared on the earth were non-
green and presumably anaerobes.
Reason: The first autotrophic organisms were the chemoautotrophs that never
released oxygen.
Important You tube link to understand contents--
https://youtube.com/playlist?list=PLMQhn0MTk6aLevo-Yw3wDG-rW6XtaIWMd
Factors affecting are - Genetic drift, Mutation, Gene flow, Genetic Recombination, Natural Selection
6- Study the figures given below & answer the following question.
iv) Under the influence of which type of
natural selection would graph (a) become like (b).
v) What could be the likely reason for new
variations arising in a population.
vi) Who suggested natural selection as the
mechanism of evolution?
Ans: (i) Disruptive & elective.
(iv) Individuals at the extremes contribute more offspring compared to those in the centre & produce two
peaks in the distribution of a trait.
(v) Charles Darwin
7- Explain the evolution of human beings in a sequential manner.
Ans: Dryopithecus and Ramapithecus - about 15 mya, primates called Dryopithecus and Ramapithecus were
existing. They were hairy and walked like gorillas and chimpanzees. Ramapithecus was more man-like while
Dryopithecus was more ape-like.
Australopithecus- about two mya, Australopithecines probably lived in East African grasslands, they hunted
with stone weapons, essentially ate the fruit.
Homo habilis- Brain capacities - 650-800cc, they probably did not eat meat.
Homo erectus- about 1.5 mya, brain capacity- 900cc., Homo erectus probably ate meat.
Neanderthal man- brain size- 1400cc, they used to bury their dead.
Homo sapiens or modern man- Arose in Africa, During the ice age between 75,000-10,000 years ago.
Chapter-8
Human Health and Diseases
MASTER CARD
HEALTH a state of complete physical, mental and social well-being.
Pathogens Disease causing organisms ( bacteria, viruses, fungi, protozoans, helminths, etc.)
Congenital Diseases which are present since birth.
Diseases
Infectious Diseases which are easily transmitted from one person to another
diseases

Immunity The ability of the body (host) to fight against the disease causing agent

Active It is a type of acquired immunity in which the body produces its own antibodies against
immunity disease-causing antigens.
Passive It is a type of acquired immunity in which ready-made antibodies are transferred from one
immunity individual to another.
Immuno The chemical which suppress the immune response to antigen.
Suppressant`
Antigens These are substances which when enter the body, stimulate the production of antibodies
(Immunogens)
Antibodies These are immunoglobulins (class of proteins) produced in response to antigenic
stimulation. (IgA, IgM, IgE and IgG)
allergy The exaggerated response of the immune system to certain antigens
present in the environment
Lymphoid These are the organs where origin and/or maturation and proliferation of lymphocytes
organs occur.
1. Primary lymphoid organs -Where Lymphocytes originate and mature.
● Bone marrow- all blood cells are produced & B-lymphocytes mature
● Thymus- lobed organ, located beneath breast bone, maturation of T-lymphocytes.
2. Secondary lymphoid organs- Where Lymphocytes interact with the antigen
and proliferate to become effective cells
● Spleen -filters blood by trapping blood borne microbes, reservoir of
erythrocytes
● Lymph nodes -trap microbes or antigens, activate the lymphocytes &
cause response
● MALT-(Mucosal Associated Lymphoid Tissue)- 50% of lymphoid tissue
α-interferon biological response modifiers
drug abuse When drugs and alcohol taken for purpose other than medicinal that impairs
physical, physiological and psychological functions, it is called drug abuse.
● Opioids- latex of poppy plant Papaver somniferum eg. Heroin (smack), morphine
● Cannabinoids- inflorescences of the plant Cannabis sativa
● Coca alkaloid / cocaine/ coke or crack is obtained from coca plant Erythroxylum
coca
Addiction A psychological attachment to certain effects –such as euphoria and a temporary feeling of
well-being – associated with drugs and alcohol
PMNL Poly morpho-Nuclear Leukocytes
CMI Cell Mediated Immunity
ELISA Enzyme Linked Immuno sorbent Assay

MALT Mucosal Associated Lymphoid Tissue


NACO National AIDS Control Organisation
MRI Magnetic Resonance Imaging
CO Carbon monoxide
CT Computed tomography
HIV Human Immuno deficiency Virus
AIDS Acquired Immuno Deficiency Syndrome
Carcinogens Cancer causing agents. e.g., gamma rays, UV rays, dyes and lead
Interferon The glycoproteins produced by our body cells in response to a viralinfection
Incubation The time period between infection and the appearanceof symptoms.
Period
Metastasis The property in which the cancer cells spread to different sitesthrough blood and
develop secondary tumours.
Oncogene Gene which causes cancer.
Retrovirus A virus having RNA as genetic material and forms DNA by reverse transcription and then
replicate e.g., Human Immunodeficiency Virus (HIV).
Syndrome Group of symptoms.
Vaccination: Inoculation of a vaccine to stimulate production of antibodies andprovide immunity for
one or more disease.

FACTORS AFFECTING HEALTH


• genetic disorders – deficiencies with which a child is born and deficiencies/defects which the child inherits
from parents from birth;
• infections
• Lifestyle
INFECTIOUS AND NON-INFECTIOUS DISEASE
Infectious Non- Infectious

transmitted from one person to another Not transmitted from one person to
like AIDS another like Cancer
COMMON DISEASES IN HUMAN
Typhoid Salmonella typhi Test- Widal Test
Intestinal perforation Sustain high fever (39-40 0 C), weakness,
stomach pain, constipation, headache and loss of appetite
Pneumonia Streptococcus pneumoniae Infects the alveoli (air filled sacs) of the lungs.
Haemophilus influenzae Problems in respiration, fever, chills, cough, headache, lips and
fingernails may turn grey to bluish in colour
Common Rhino Virus Infection of the nose and respiratory passage but not the lungs.
cold Nasal congestion and discharge, sore throat, hoarseness,
cough, headache, tiredness.
Malaria Plasmodium Chill and high fever recurring every three to four days
Amoebiasis Entamoeba histolytica Infection of large intestine
(amoebic Constipation, abdominal pain and cramps, stools with excess
dysentery) mucous and blood clots.
Ascariasis Ascaris Infect intestine
internal bleeding, muscular pain, fever, anemia and blockage
of the intestinal passage
Elephantiasis Wuchereria (W. bancrofti Infect lymphatic vessels
or Filariasis and W. malayi) Chronic inflammation of the organs like lower limbs and
genitals.
Ringworms Microsporum, The appearance of dry, scaly lesions on various parts of the
body such as skin, nails and scalp

Trichophyton
Epidermophyton
AIDS HIV Syndrome
Dengue Dengue virus high fever, headache, joint pains, chill, rashes, abdominal pain,
Liver disorder, fall in platelets
Chikunguny Virus rashes on limbs and trunk, arthritis of multiple joints, high
a fever, conjunctivitis

MALARIA
• Pathogen- Plasmodium (P. vivax, P. malaria and P. falciparum). Malignant malaria caused by Plasmodium
falciparum is the most serious and can be fatal.
• Vector- female Anopheles mosquito
• Plasmodium enters the human body as sporozoites (infectious form) through the bite of an infected female
Anopheles mosquito.
• The parasites in the liver cells and then rupture the RBCs.
• The rupture of RBCs results in the formation of hemozoin, which is responsible for the chill and high fever
recurring every three to four days.
• When a female Anopheles mosquito bites an infected person, these parasites enter the mosquito’s body,
undergo further development, and again form sporozoite.
• The sporozoites are stored in their salivary glands.
AIDS (Acquired Immuno Deficiency Syndrome)

• Pathogen –HIV a retrovirus (RNA-containing virus)


• Transmission- occurs by (a) sexual contact with the infected
person, (b) by transfusion of contaminated blood (c) by
sharing infected needles (d) from the infected mother to her
child through the placenta.

• After infection, the virus enters macrophages (HIV factory). In


macrophages, viral RNA replicates to form viral DNA with the
help of the enzyme reverse transcriptase.
• This viral DNA gets incorporated into the host cell’s DNA and
directs the infected cells to produce virus particles.
• Simultaneously, HIV attacks helper T-lymphocytes (TH). Thus,
the number of TH cells decreases significantly. Due to this, the
infected person starts suffering from various infections like
IMMUNITY Mycobacterium, viruses, fungi, and Toxoplasma.
• Symptoms- fever, diarrhoea and weight loss, etc.
Ability of the host to fight the disease-causing organisms, conferred by the immune system is called immunity.
• Diagnostic test- ELISA (enzyme-linked immunosorbent assay)
• - (i)
Types Treatment- Anti-retroviral
Innate immunity and (ii) drugs
Acquired immunity.
Non-specific, present at the time of birth

Pathogen-specific, after birth

Barriers of Innate Immunity

Physical barriers Skin, Mucus coating of the It prevents entry of the micro-
epithelium lining organisms
Physiological barriers Acid in the stomach, saliva in the prevent microbial growth
mouth, tears from eyes
Cellular barriers WBC, polymorpho-nuclear Phagocytose and destroy microbes.
leukocytes (PMNL-neutrophils)
Cytokine barriers Virus-infected cells secrete protect non-infected cells from
proteins called interferons further viral infection
Acquired Immunity

• It is characterised by memory.
Primary response- When or body encounters a pathogen for the first time produces low intensity response.
Secondary response- Subsequent encounter with the same pathogen elicits a highly intensified secondary or
anamnestic response.
The primary and secondary immune responses are carried out with the help of two special types of lymphocytes
present in our blood- (i) B-lymphocytes and (ii) T lymphocytes.
B lymphocyte- they produce Antibodies (immunoglobin) in response to pathogens
T-cells - help B cells produce antibodies.
Antibody
• H2L2 structure – Antibodies are composed of four peptide chains, two small
(light chains) and two longer (heavy chains).
• Types of antibodies - IgA, IgM, IgE, IgG.
Humoral IMMUNITY

Cell mediated
(Antibody mediated) (No antibody formation)

Active Passive
Active Immunity – antibodies are produced in the host cell after infection with antigens.
Active immunity is slow and takes time to give its full effective response
Passive Immunity- When ready-made antibodies are directly given to protect the body against
foreign agents. e.g. baby receives a mother's antibodies through the placenta or breast milk.

Vaccination and Immunisation

Allergies

• The immune system’s exaggerated response to certain antigens (allergens) present in the environment is called
allergy. Common allergens are - pollen grains, mites in dust, animal dander etc.
• Allergic reactions are due to chemicals like histamine and serotonin released from the mast cells.
• Antibody IgE is produced in response to allergens.
• Symptoms of allergy - sneezing, watery eyes, running nose, difficulty in breathing.
• Treatment- Antihistamine, adrenaline and steroids
Auto Immunity

• It is an abnormality in which the immune response starts rejecting its own body cell or self-cells and molecules
e.g. Rheumatoid Arthritis. Such diseases are known as an auto-immune diseases.

CANCER

In this, some of the body's cells grow uncontrollably


It shows uncontrolled proliferation of cells without any differentiation.
Contact inhibition- Process that inhibits a normal cell's uncontrolled growth due
to its contact with the surrounding cells. Cancerous cells lost this property
Tumour- Large mass of cells formed due to the repeated division of cancerous
cells

Benign tumors- Remain Malignant tumors These cells grow


. confined to their original very rapidly, invading and damaging
location, do not spread to the surrounding normal tissues of
other body parts, and cause other organs
little damage.

Metastasis- Cancer cells move from tumour to new sites through blood for forming secondary tumours. This invasion
of cancer cells from one part to another parts by the body fluids is called metastasis.

Carcinogens (cancer-causing agents)


Chemicals nicotine
Biological These are cancer-causing viruses is called oncogenic viruses.
Oncogenic viruses They have genes called viral oncogenes.
ionizing radiation X-rays and y-rays
Non-ionizing radiations UV-rays

Diagnosis- Biopsy and histopathological studies of the tissue and blood and bone marrow tests for increased cell counts
in the case of leukaemia, Radiography by X-rays, CT scan, MRI, use of Antibodies against cancer-specific antigens

Treatment of cancer- (a) Surgery (b) Radiation therapy (c) Chemotherapy (d) Immunotherapy- use of biological
modifiers like a-interferon

DRUGS AND ALCOHOL ABUSE

Drug Common name and source Effect


Opoids Heroin (smack) Bind to specific opioid receptors present in our
Source - Latex of poppy CNS and gastrointestinal tract.
(Papaver somniferum) Heroin is a depressant and slows down body
functions

Cannabinoids Examples- marijuana, hashish, Interact with cannabinoid receptors present


charas and ganja. principally in the brain.
Obtained from Cannabis Effects on the cardiovascular system of the body.
sativa.

Coca alkaloid coke or crack interferes with the transport of the neuro-
Obtained from coca plant transmitter dopamine.
Erythroxylum coca. It acts on CNS producing a sense of euphoria and
may cause hallucinations.

Nicotine Tobacco Stimulates adrenal gland raises blood pressure


and increase heart rate
Barbiturates, Amphetamines, Benzodiazepines, and lysergic acid diethyl amides (LSD), are used as medicine to
help patients cope with mental illnesses like depression and insomnia.
ADOLESCENCE, ADDICTION AND DEPENDENCE

• Adolescence (12-18 years) is a bridge linking childhood and adulthood. It is accompanied by several biological
and behavioural changes. Adolescence thus is a very vulnerable phase of the mental and psychological
development of an individual.
• Addiction is a psychological attachment to certain effects –such as euphoria and a temporary feeling of well-
being – associated with drugs and alcohol.
• Dependence is the tendency of the body to manifest a characteristic and unpleasant withdrawal syndrome if
the regular dose of drugs/alcohol is abruptly discontinued.

IMPORTANT QUESTIONS
Very Short Answer Type Questions/ MCQ
1- Antibodies present in colostrum that protect the new-born from certain diseases iareof
a- IgG type
b- IgA type
c- IgD type
d- IgE type.
Ans: B
2- Malignant malaria is caused by which pathogen?
a- Plasmodium falciparum
b- Plasmodium vivax
c- Plasmodium malignancies
d- None of these
Ans: A
3- Chemically interferons are-
a- Carbohydrates
b- Glycosides
c- Peptides
d- Lipoproteins
Ans: C
4- Which of the following is not lymphoid tissue?
a- Spleen
b-Tonsils
c- Liver
d-Thymus
Ans: C
5- Ig E is released during-
a- Allergy
b- Colostrum secretion
c- Tear formation
d- None of these
Ans: A
6- Give any two molecular diagnosis techniques used to diagnose disease.
Ans: Widal test for typhoid
Biopsy- Cancer
7- Interferons are secreted by which type of cell. What is the chemical nature of interferon?
Ans: interferon is secreted by the virus-infected cell. Interferons are proteins.
8- Name the infective stage of the malaria parasite in humans. In which organ of the mosquito is situated?
Ans: Sporozoite. Situated in mosquito salivary gland
9- The malaria parasite requires two hosts to complete its life cycle. Identify the host where the following
events take place-
(i) Asexual reproduction and gametocyte formation (ii) Fertilization (fusion of gametocytes)
Ans: (i) Human (ii) Mosquito
10- Name any four types of immunoglobin present in the human immune system.
Ans: IgA, IgM, IgE, IgG
11- Provide two means of passive immunity through which the fetus and newly born baby get protected from
infection.
Ans: The yellowish fluid colostrum is secreted by the mother during the initial days of lactation. The fetus also
receives some antibodies from their mother, through the placenta during pregnancy.
12- Identify the type of immunoglobins in the following-
(i)In colostrum (ii) released during an allergic response
Ans: (I) Ig A (II) Ig E
13- Why it is not possible to treat autoimmune diseases. Give one example of such a disease.
Ans: there is no treatment for such diseases because the immune system of the body attacks self-cells. Example-
Rheumatoid arthritis
14- Mention any two drugs that are used to treat mental illnesses like depression and insomnia.
Ans: Barbiturates, Amphetamines
Short Answer Type Question
1- Give a detailed account of factors that affects disease.
Ans: (i) genetic disorders – deficiencies with which a child is born and deficiencies/defects which the child inherits
from parents from birth
(ii) Infections
(iii) Lifestyle including food and water we take
2- Explain the disease and its type with two examples of each.
Ans: Complete physical, mental and social well-being is known as health.
Infectious disease – cancer, arthritis
Non-infectious disease- flu, tuberculosis
3- Primary immune response is of slow intensity than secondary immune response. Justify the statement.
Ans: when our body pathogen for the first time it produces a primary response which is of low intensity.
Subsequent encounter with the same pathogen shows a quick and highly intensified secondary or anamnestic
response. This is because in primary response antibodies are formed which have the property of memory.
4- (i) Draw the structure of immunoglobin.
(ii) Why immunoglobins are called H2L2 molecules.
Ans: (i) Fig 8.4, page 151, NCERT
(ii) Because it contains two heavy chains (H2) and two light chains (L2) of polypeptide
5- (i) What is an allergy?
(ii) Name two factors that are responsible for allergy in our body.
(iii) List any two-medicine advised by doctors to reduce the effect of allergy.
Ans: (i) the exaggerated response of the immune system to certain antigens present in the environment is called
an allergy
(ii) Histamine and serotonin
(iii) Anti-histamine, adrenalin
6- Which cell is known as the factory of HIV and why?
Ans: Macrophages are called HIV factories. After entering the e host body, HIV moves into macrophages where its
RNA replicates to form viral DNA. This viral DNA gets incorporated into the host cells' DNA and directs the infected
cells to produce more viruses. Hence macrophages continue to produce viruses and act as HIV factories.
7- What are the different diagnosis techniques to detect cancer in a patient?
Ans: CT scan, MRI, Biopsy, Blood and bone marrow tests are done for increased cell counts in case of leukemia, X
ray, Monoclonal antibodies test
8- (i) Name two recent incidences of widespread diseases caused by Aedes mosquitoes.
(ii) Mention the name of two pathogens that are responsible for ringworm disease.
K.V.S.MUMBAI REGION
(iii) Which pathogen infects the alveoli (of the lungs that result in severe breathing problems?
Ans: (i) Dengue and Chikungunya
(ii) Microsporum, Trichophyton
(iii) Streptococcus pneumoniae or Haemophilus influenzae
9- Differentiate between two different types of tumors. Which one is lethal and why?
Ans: (i) Benign tumour - tumour remains confined to the place of origin or affected organ. The rate of growth of
tumors is low.
(ii) Malignant tumor- it invades surrounding tissue & spreads throughout the body. The rate of growth of the tumor
is rapid
Malignancy is lethal as it spreads all over the body through the process of metastasis.
10- A person undergoes ELISA testing and tested positive-
(i) ELISA is widely conducted to diagnose which disease.
(ii) Write the causative agent of that disease.
(iii) Which organization in India educates people about that disease?
Ans:
(i)AIDS, (ii) HIV, (iii) NACO (National AIDS Control Organisation)
Long Answer Type Question
1- Describe the different mechanisms by which innate immunity protects the human body since birth.
Ans: Physical barriers - Skin prevents entry of the micro-organisms. Mucus coating of the epithelium lining the
respiratory, gastrointestinal and urogenital tracts also helps in trapping microbes entering our body
Physiological barriers- Acid in the stomach, saliva in the mouth, tears from the eyes–all prevent microbial growth.
Cellular barriers- Leukocytes (WBC) like polymorpho-nuclear leukocytes (PMNL-neutrophils) and monocytes and
natural killers in the blood, macrophages in tissues can phagocytose and destroy microbes
Cytokine barriers- Virus-infected cells secrete proteins called interferons which protect non-infected cells from
further viral infection.
2- A disease has symptoms of high fever with chill. The causative agents depend on the red blood cells of the
human body for their life cycle. Based on the statement answer the following questions-
(i) Why do patients suffer from high fever with a chill?
(ii) Name the disease and its causative agent.
(iii) Represent the life cycle of the pathogen diagrammatically.
Ans. (i) due to the release of chemical hemozoin from ruptured RBC
(ii) Malaria, Plasmodium (P. vivax, P. Malariae and P. falciparum).
(iii) Life cycle of Plasmodium: Fig. 8.1 Page 148, NCERT
3- How do addiction and dependence differ from each other? What are the consequences of withdrawal
syndrome?
Ans: Addiction is a psychological attachment to certain effects –such as euphoria and a temporary feeling of well-
being – associated with drugs and alcohol.
Dependence is the tendency of the body to manifest a characteristic and unpleasant withdrawal syndrome if the
regular dose of drugs/alcohol is abruptly discontinued.
Withdrawal syndrome occurs if the regular dose of drugs/alcohol is abruptly discontinued. This is characterized by
anxiety, shakiness, nausea, and sweating, which may be relieved when use is resumed again. In some cases,
withdrawal symptoms can be severe and even life-threatening and the person may need medical supervision
4- How does the humoral immune system work when our body is infected?
Ans: Humoral immunity is also called antibody-mediated immunity.
In this Immunoglobins (antibodies) are developed from B- lymphocytes.
These antibodies (B-cell) have receptors on their surface that recognize a specific antigen.
The antibody binds to the antigen- forming an antigen-antibody complex which is later digested by phagocytic cells
like macrophages.
5- By observing the diagram answer the flowing questions-
(i) Mention the group of drugs this structure represents.
(ii) How these drugs are taken by drug abusers?
K.V.S.MUMBAI REGION
(iii) Name the source of the plant from which these are isolated.
(iv) Which part of the human body is affected by this drug?
(v) Provide any two common names for this drug.
Ans: (i) Cannabinoids (ii) Oral Ingestion or inhalation
(iii) Cannabis sativa
(iv) Cardiovascular system
(v) Charas, ganja
ASSERTION AND REASON QUESTIONS
Answer the questions selecting the appropriate option given below:
A. Both A and R are true and R is the correct explanation of A.
B. Both A and R are true and R is not the correct explanation of A.
C. A is true but R is false.
D. A is False but R is true.
1 Assertion: Inspite of exposure to large number of infectious agents, humans are resistive to
diseases.
Reason: Humans are able to defend against most of the foreign agents due to the ability to fight
disease- causing organisms.
ANSWER: A
2 Assertion : An antibody is represented by H2L2.
Reason : Each antibody is made of four peptide chains.
ANSWER: A
3 Assertion: Virus-infected cells secrete proteins known as interferons.
Reason: Interferons protect the non-infected cells from bacterial infection.
ANSWER: C
4 Assertion : Tobacco contains nicotine which stimulates the adrenal gland.
Reason: Nicotine increases the blood pressure and the heart rate. Assertion :
Answer B
5 Assertion: Histamine is related with allergic and inflammatory reactions.
Reason: Histamine is a vasodilator.
ANSWER: A
6 Assertion : Organ transplantation patients are given immunosuppressive drugs.
Reason : Transplanted tissue has antigens which stimulate the specific immune response
of the recipient.
ANSWER: A
7 Assertion : A person who has received a cut and is bleeding needs to be given anti-tetanus
treatment.
Reason : Anti-tetanus injection provides immunity by producing antibodies for tetanus.
ANSWER: C
8 Assertion : Mast cells in the human body release excessive amounts of inflammatory
chemicals, which cause allergic reactions.
Reason : Allergens in the environment on reaching human body stimulate mast cells in
certain individuals.
ANSWER: A
9 Assertion: HIV infected person are prone to opportunistic diseases.
Reason: Immune system weakens during HIV infection.
ANSWER: A
10 Assertion: A person experiences chills and high fever recurring every three to four days in
malaria.
Reason: This is caused by the release of haemozoin with rupture of liver cells.
ANSWER: C

K.V.S.MUMBAI REGION
COMPETENCY BASED QUESTIONS:

CCT QUESTIONS
1.

i. What does the above diagram illustrate?


ii. Name the parts labelled ‘a’ and ‘b’
iii. Name the type of cells that produce this molecule.
ANSWER
i. The diagram illustrate the structure of an antibody molecule
ii. a-antigen binding site b- Heavy chain
iii. B lymphocytes
2 Observe the given diagram and answer the given questions

2a Choose the correct statement(s) regarding the figure.


I. It is a depressant.
II. It is a stimulant.
III. It produces a sense of euphoria and increased energy
IV. It slows down body functions.
Codes
(a) I and III
(b) II and IV
(c) I and IV
(d) II and III
2b The diagram shown above represents the flowering branch of which plant?
(a) Cannabis sativa
(b) Datura
(c) Opium poppy
(d) Atropa belladona
2c Name the blank spaces a,b,c and d in the table given below
Name of the drug Plant source Organ system affected
i a Poppy plant b
ii Marijuana c d
Answer:
a-morphine b- central nervous system c- Cannabis
sativa d- cardiovascular system
K.V.S.MUMBAI REGION
SOURCE BASED QUESTIONS
1 Read the following passage and answer any four questions given below:
. Aravind went to his hometown along with his parents during his summer vacation. His grandparents’ house is
surrounded by farm land. He sought permission from his mother to play in farm land along with his friends. On
returning back he had sneezing, watery eyes, running nose.
Based on the above information, answer the following questions
I. State the functions of mast cells in allergic response.
II. In the metropolitan cities of India, many children are suffering from allergy/asthma. What are the main
causes of this problem?
III. Give some symptoms of allergic reactions.
OR
Name the type of antibody responsible for allergic response
Ans.
i. Mast cells release chemicals like histamine and serotonin in allergic response.
ii. Allergy is the exaggerated response of the immune system to certain antigens present in the
environment. In metropolitan cities lifestyle is responsible in lowering of immunity and sensitivity to
allergens. More polluted environment increases the chances of allergy in children.
iii. Some symptoms of allergic reactions are sneezing, watery eyes, running nose and difficulty in breathing.
OR
IgE
2 The data below shows the concentration of nicotine smoked by a smoker taking 10 puffs/ minute.
.

(a) With reference to the above graph explain the concentration of nicotine in blood at 10 minutes.
(b) How will this affect the concentration of carbon monoxide and haembound oxygen at 10 minutes?
(c) How does cigarette smoking result in high blood pressure and increase in heart rate?
OR
(c) How does cigarette smoking result in lung cancer and emphysema?
(from cbse SAMPLE QP 22-23)
ANSWER
(a) Concentration of nicotine is maximum at 10 minutes/ conc. of nicotine increases steadily in the blood to
reach 45mg/cm3
(b) The Concentration of CO will increase resulting in reduced concentration of haembound oxygen.
(c) Nicotine results in stimulating the adrenal gland which results in release of adrenaline / nor - adrenaline in
the blood resulting in increase of blood pressure and heart rate.
OR
(c) Chemical carcinogens present in tobacco smoke are the major cause of lung cancer.
The cigarette smoke irritates the air passages of the lungs causing them to produce mucus which causes cough
resulting in enlarging air spaces/ reduce surface area/lose their elasticity (any point can be mentioned) thus
difficulty in breathing causing emphysema.

QUESTIONS and ANSWERS


MCQ
1 Which of the following are the correct reasons for Rheumatoid arthritis?
K.V.S.MUMBAI REGION
i.) Lymphocytes become more active
ii.) Body attacks self cells
iii.) More antibodies are produced in the body
iv.) The ability to differentiate pathogens or foreign molecules from self cells is lost
Choose the correct answer from the options given below:
a.) i and ii b.) iii and iv c.) i and iii d.) ii and iv

2 Interferons are most effective in making non-infected cells resistant against the spread of which of the
following diseases in humans?
(a) ascariasis (b) ringworm (c) amoebiasis (d) AIDS
3 Transplantation of tissues/organs to save certain patients often fails due to rejection of such tissues/organs
in the patient’s body. Which type of immune response is responsible for such rejection?
a.) auto-immune response b.) humoral immune response c.) physiological immune response d.) cell-
mediated immune response

4 The following table shows certain diseases, their causative organisms and symptoms.
Diseases Causative Symptoms
organisms
i. Filariasis A Inflammation of lymphatic vessels
ii. Typhoid B High fever, stomach pain
iii. C Rhino viruses Nasal congestion and discharge
iv. Ascariasis Ascaris D

The correct option regarding A, B, C and D is


(a) A–Wuchereria, B–Salmonella typhi, C–Common cold, D–Internal bleeding, fever, anaemia
(b) A–Salmonella typhi, B–Ascaris, C–Typhoid,D–Stomach pain, headache
(c) A–Ascaris, B–Entamoeba histolytica, C–Pneumonia,D–Constipation, fever
(d) A–Entamoeba histolytica, B–Salmonella typhi, C–Common cold, D–Nasal discharge, high fever
5. Apis mellifera are killer bees possessing toxic bee venom. Identify the treatment and
the type of immunity developed from the given table to treat a person against the
venom of this bee.

ANSWER (c) Preformed Antibodies, Passive


VSA 1 mark
1. How does α-interferon help in the treatment of malignant tumours?
ANSWER
α-interferon is a biological response modifier and can target specific disease causing mechanisms.
2. Identify the compound chemical structure is shown below. State any three of its physical properties.

It is Morphine. Physically it appears as a white, odourless, crystalline compound.


3. Malaria, caused by Plasmodium is often characterised by high fever recurring every three to four days.
Explain why this happens.
ANSWER
K.V.S.MUMBAI REGION
The rupture of RBCs releases a toxic substance called haemozoin, which is responsible for the chill and high
fever.
4 If a patient is advised anti-retroviral drug, which infection is he suffering from? Name the causative
organism.
ANSWER
AIDS caused by the Human Immuno deficiency Virus
5. Many microbial pathogens enter the gut of humans along with food. Name the physiological barrier that
protects the body from such pathogens?
ANSWER
Physiological barriers : Acid in the stomach and saliva in the mouth.
SHORT ANSWER-2MKS
1. Define innate immunity. Name and explain the category of barrier which involves macrophages. 2
OR
Name and explain the type of barrier of innate immunity system which involves macrophages.
Ans: Innate immunity is non-specific type of defence, that is present at the time of birth
and is inherited from parents. Cellular barrier/phagocytosis involves macrophages. The
macrophages form an important component of this type of immunity. At the site of infection,
the blood flow increases and so does the cellular components. Macrophages and neutrophils
engulf themicrobes and cellular debris whenever they happen to enter the body.
2.Name and explain the two types of immune responses in humans.
2MAns: The two types of immunity are active immunity and passive immunity.
Active immunity: immunity developed in the host body due to production of antibodies in
response to antigens.
Passive immunity: when ready-made antibodies are directly given to protect the bodyagainst
foreign agents.
Or
The two types of immunity are humoral immunity and secondary immunity
Humoral immunity: Immune responses given by antibodies found in the blood.Cell-
mediated immunity: T- lymphocytes mediate this immunity.
Or
The two types of immunity are primary immunity and secondary immunity.
Primary immunity: when our body encounters a pathogen for the first time producesprimary
response.
Secondary immunity: subsequent encounter with the same pathogen generates highly
intensified secondary response.

3 Explain the role of the following in providing defence against infection in human body: 3M

(i) Histamines (ii) Interferons (iii) B-cells

Ans: (i) Histamines: these are chemicals which cause inflammatory responses.

Ii] Interferons: these are glycoproteins which protect non-infected cells from furtherviral
infection
Iii] B-cells: these produce proteins called antibodies in response to pathogens into theblood to
fight with them.
4. What are allergens? How do human beings respond to them? 2M

Ans: Allergens are allergy causing agents or substances which act as weak antigens andcause
inappropriate and excessive immune response. Allergen being a harmless substance initiates
antibody formation which in turn binds to the receptors on the mast cell. The mast cell releases
chemicals such as histamines, which in turn cause inflammation of the mucous membranes.
Many anti-allergic treatments suppress mast cells or neutralize histamines.
5. Explain the response initiated when a dose of vaccine is introduced into the human body. 2M

Or
Write the events that take place when a vaccine for any disease is introduced into the human
body. Ans: the vaccine contains proteins of pathogen or inactivated/weakened pathogen. When a
K.V.S.MUMBAI REGION
dose of vaccine is introduced into the body, it behaves as an antigen and the body produces
antibodies in response to the antigen. This response generates active immunity. The antibodies
thus produced will neutralize the pathogenic agents duringactual infection. The vaccines also
generate memory B- and T- cells that recognize the pathogen quickly on subsequent exposure
and overwhelm the invaders with a massive production of antibodies.
6. Define auto-immune disease. Give two examples. 2M

Ans: auto-immune disease refers to the disease when body’s immune system due togenetic
or other unknown reasons starts destroying self-cells and molecules, e.g., rheumatoid
arthritis and systemic lupus.
7. Name any two carcinogens? Also explain the term malignancy.
Ans -UV rays and 5-bromouracil. 2M
8. How does spleen act as a lymphoid organ? Explain
2
MAns: the spleen is a large bean-shaped organ. It mainly contains lymphocytes and phagocytes.
It acts as a filter of the blood by trapping blood-borne microorganisms.
Spleen also has a large reservoir of erythrocytes.
9. (a) Explain the property that prevents normal cells from becoming cancerous. 2M
(b) All normal cells have inherent characteristic of becoming cancerous. Explain

Ans: (a) by the contact inhibition property, normal cells do not change into cancerous cells.
B]Several genes called cellular oncogenes (c-onc) or proto-oncogenes in normal cells which,
when activated under certain conditions, could lead to oncogenic transformationof cells leading to
cancer.
10. Name the blank spaces a, b, c and d in the table given below: 2M

SNo Name of the drug Plant source Organ system


affected
1 a Poppy plant b
2 Marijuana C d

Ans: (a) Morphine b) Central nervous system


(b) Cannabis sativa d) Cardiovascular system

11. From which plant are cannabinoids obtained? Name any two cannabinoids. Which part of the 2M
body is affected by consuming these substances?

Ans: Cannabinoids are obtained from the inflorescences of the plant Cannabissativa. Marijuana,
hashish, charas, ganja are some cannabinoids. These chemicals interact withcannabinoid
receptors of the body, mainly present in the brain. Cardiovascular system is effected adversely.
12. How does the transmission of each of following diseases take place? 2M

(a) Amoebiasis b) Malaria


(c) Ascariasis d) Pneumonia

Ans: (a) Amoebiasis: Caused by Entamoeba histolytica(protozoan parasite).


Transmission:
(i) By ingesting cysts with food and water
(ii) The cysts are carried by flies from faces to food and drinks
B]Malaria: Caused by Plasmodiumsps. Transmission:
(i)Transmitted by female Anopheles mosquito.
(ii) Female anopheles injects sporozoites along with saliva while sucking blood.
C]Ascariasis: Caused by Ascaris lumbricoides. Is transmitted through contaminatedfood and
water with Ascaris eggs.
D]Pneumonia: Caused by Streptococcus pneumonia. Transmission is through sputum,droplets
or aerosols of the patient.
13.(a) Cancer is one of the most dreaded diseases of humans. Explain ‘Contact inhibition’ 2M
and ‘Metastasis’ with respect to the disease.
K.V.S.MUMBAI REGION
B]Name the group of genes which have been identified in normal cells that could lead tocancer
and how they do so?
C]Name any two techniques which are useful to detect cancers of internal organs
D]Why are cancer patients often given α- interferon as part of the treatment?

Ans: (a) Contact inhibition is the property of normal cells in which contact with other cells inhibits
their uncontrolled growth. Metasatasis is the property in which tumor cells reach distinct sites in
the body, though blood.
B]Proto oncogenes or cellular oncogenes. These genes when activated under certaincondition
could lead to oncogenic transformation of the cells.
C]Biopsy/ radiotherapy/CT/MRI
D]2-interferon activities immune system and destroys the tumor.

14. Why does a doctor administer tetanus anti toxin and not a tetanus vaccine to a child injured 2M
in a road side accident with a bleeding wound? Explain.
Or
Why a person with cuts and bruises following an accident is administered tetanus antitoxin?
Give reasons.
Ans: tetanus is caused by a microbe which has a deadly and fast action. Action of vaccineis slow
and this delay may become fatal therefore anti toxin’s are administered which neutralize the effect
of bacterial toxin.
15. Why an immuno suppressive agent is taken after an organ transplant? 2M

Ans: Our immune system is capable to differentiate between self and non self cells/tissues.The
graft is a non-self tissue which may be rejected by our immune system. So to prevent the rejection,
immuno suppressants are taken after the transplant.

SHORT ANSWER -3Mks


1 Name any two organisms that are responsible for ringworms in humans. Mention two
diagnostic symptoms. Name the specific parts of the human body where these organisms thrive
and explain why.
3M
Ans: Microsporum/Trichophyton/Epideromophyton.
Symptoms: Dry/scaly lesion on skin/nails/scalp/intense itching (any two)
These thrive in body groin/between toes, thrive better in heat/moisture/perspiration.
2 Identify a,b,c and d in the following table: 3M
Sno Name of the Name of the causal Specific organ orits
human disease bacteria/virus part affected

(i) Typhoid Salmonella typhi A


(ii) Common cold B C
(iii) Pneumonia Streptococcus pneumoniae D

Ans: (a) small intestine (b) Rhino virus


(b)Nose and respiratory passage (d) alveoli of lungs

K.V.S.MUMBAI REGION
3 Draw a well labeled diagram of an antibody molecule.
3

MAns:

4. Write the source and the effect on the human body of the following drugs: 3M

(i) Morphine (ii) Cocaine (iii) Marijuana

Ans: (i) Morphine: it is obtained from poppy plant papaver somniferum. It binds to specificopioid
receptors present in central nervous system and gastrointestinal tract.
(i) Cocaine: it is obtained from coca plant Erythroxylum coca. It interferes with
thetransport of the neurotransmitter dopamine.
( i) Marijuana: it is obtained from Cannabis sativa. It affects the
cardiovascularsystem of the body.
5. What are the various public health measures which you would suggest as safeguard against 3M
infectious disease?

Ans: The common preventive measures are as follows:


(i) Education: people should be educated about communicable diseases to
protectthemselves from such diseases.
(ii) Isolation: the infected person should be isolated to minimize the spread of infection
(iii) Vaccination: people should get vaccination on time to avoid infection
(iv) Sanitation: the Sanitation condition should be improved to avoid infection
frompolluted water, contaminated food etc.
(v) Eradication of vectors: the breeding places of vectors should be destroyed
andadult vectors should be killed by suitable methods
(vi) Sterilization: the patient’s surroundings and articles of use should be
completelysterilized so as to reduce the chances of infection.
6. (a) Name the stage of plasmodium that gains entry into the human body 3M
(b) Trace the stages of plasmodium in the body of female Anopheles after its entry
(c) Explain the cause of periodic recurrence of chill and high fever during malarial attack in humans

Ans: (a) plasmodium enters the human body as sporozoites

(b) When a female Anopheles mosquito bites an infected person, the parasites enter the
mosquito’s body and undergo further development. The parasites multiply within them
toform sporozoites that are stored in salivary glands until their transfer to human body
K.V.S.MUMBAI REGION
(c) The rupture of RBCs release a toxic substance called haemozoin, which
isresponsible for the chill and high fever recur.

Q 15. A person has been diagnosed to be HIV+. 3M

(i) Name the test which the person has undergone.


(ii) Write the test which of the pathogen involved and describe its structure.
(iii) Which particular cells of this person are likely to get destroyed?
Ans: (i) ELISA – Enzyme Linked Immuno-Sorbent Assay.
Human immuno deficiency Virus
Structure of HIV--
*It has an envelope enclosing the genetic material which is single stranded RNA.
*The envelope has spikes with receptors capable of recognizing antigen receptorsfound on
helper T-cells.
*Helper T-Cells.
LONG ANSWER-5MKS
1.Identify and name the disease in which the patient’s cells lose the property of contact inhibition.
State its possible causes and explain any three methods to accurately detect the pathological and physiological changes
that take place due to the disease in living tissues.
ANSWER
Disease: Cancer
Probable Causes: Physical/ Environmental- Exposure to X – rays/ gamma rays/ UV rays; Chemicals/Nicotine in tobacco/
other carcinogens Biological- Viral oncogenes/ Mutations Detection and diagnosis:
1. Cancer detection is based on biopsy and histopathological studies of the tissue; blood and bone marrow tests for
increased cell counts in the case of leukemias. In biopsy, a piece of the suspected tissue cut into thin sections is stained
and examined under microscope (histopathological studies) by a pathologist.
2. Techniques like radiography (use of X-rays), CT (computed tomography) and MRI (magnetic resonance imaging) are
very useful to detect cancers of the internal organs. Computed tomography uses X-rays to generate a three
dimensional image of the internals of an object. MRI uses strong magnetic fields and non-ionising radiations to
accurately detect pathological and physiological changes in the living tissue.
3. Antibodies against cancer-specific antigens are also used for detection of certain cancers. 4. Techniques of molecular
biology can be applied to detect genes in individuals with inherited susceptibility to certain cancers. (any three
methods)
2. A patient had tested positive to ELISA Test. Identify the disease and the pathogen responsible, give reasons for the
reduced/ weak immunity of the patient and trace the path, spread and effects of this pathogen in the human body.

K.V.S.MUMBAI REGION
ANSWER
Disease: AIDS (Acquired ImmunoDefeciency Syndrome)
Pathogen: Human Immuno deficiency virus (HIV).
Reason: Due to decrease in the number of helper T lymphocytes, the person starts suffering

Important You tube link to understand contents--


https://youtube.com/playlist?list=PLMQhn0MTk6aLevo-Yw3wDG-rW6XtaIWMd

K.V.S.MUMBAI REGION
CHAPTER- 10
Microbes in Human Welfare
Keywords
LAB lactic acid bacteria
Fermenters Large vessels used to culture microorganisms
Antibiotics Substance produced by microorganism, that is detrimental to other microorganisms
Sewage Sewage is a type of wastewater that is produced by a community of people
STPs Sewage treatment plants
Flocs masses of bacteria associated with fungal filaments form mesh like structures
BOD biochemical oxygen demand
Methanogens Methane producing bacteria
IARI Indian Agricultural Research Institute
KVIC Khadi and Village Industries Commission
Bio fertilizer Microorganism used as fertilizer to improve soil quality
Bio pesticide Living organisms used as pesticide
Mycorrhiza Association between fungus and root of plants
Biocontrol use of biological methods for controlling plant diseases and pests
IPM Integrated pest management
MICROBES IN HOUSEHOLD PRODUCTS

LAB- Lactobacillus group of bacteria is called lactic acid bacteria (LAB).


The LAB produces lactic acid which coagulates and partially digests the milk proteins. This results
in the formation of curd.
Curd is rich in vitamin B12 and it checks the disease-causing microbes in the stomach.
Fermentation property – Microorganisms are widely used in the formation of dosa, idli, and bread.
The puffed-up appearance of dough is due to the production of CO2 gas.
In bread formation, baker’s yeast (Saccharomyces cerevisiae) is used.
The fermentation of palm sap is used to make traditional drinks like a toddy.
In cheese formation, specific microbes are used-
The large holes in ‘Swiss cheese’ are due to the production of a large amount of CO2 by a
bacterium named Propionibacterium sharmanii.
The ‘Roquefort cheese’ is ripened by growing specific fungi on them.

MICROBES IN INDUSTRIAL PRODUCTS

Beverages - wine, beer, whisky, brandy or rum. Brewer’s yeast (Saccharomyces cerevisiae) is used for
fermenting malted cereals and fruit juices, to produce ethanol.
Wine and beer are produced without distillation.
Whereas whisky, brandy and rum are produced by distillation of the fermented broth.

ANTIBIOTICS
Antibiotics are chemical substances, which are produced by some microbes and can kill or retard the growth of other
(disease-causing) microbes.
• Alexander Fleming discovered the first antibiotic Penicillin from Penicillium notatum.
• Ernest Chain and Howard Florey established the full potential of Penicillin as an effective antibiotic.
• Antibiotics have greatly improved our capacity to treat deadly diseases such as plague, whooping cough,
diphtheria, and leprosy, etc.
CHEMICALS, ENZYMES AND OTHER BIOACTIVE MOLECULES

Microorganism Name Product


Fungus Aspergillus niger Citric acid
Bacterium Acetobacter aceti Acetic acid
Bacterium Clostridium butylicum Butyric acid
K.V.S.MUMBAI REGION
Bacterium Lactobacillus Lactic acid
Yeast Saccharomyces cerevisiae Ethanol
Lipases are used in detergent formulations and are helpful in removing oily stains from the laundry.
Bottled fruit juices bought from the market are clearer as compared to those made at home. This is because the
bottled juices are clarified by the use of pectinases and proteases.
Product Microorganism Use
Streptokinase Streptococcus clot buster
Cyclosporin A Trichoderma polysporum immunosuppressive agent
Statins Monascus purpureus Blood-cholesterol-lowering agents.

MICROBES IN SEWAGE TREATMENT


This municipal wastewater is also called sewage. Sewage) contains a large amount of organic matter and pathogenic
microbes, human excreta etc. Before disposal, hence, sewage is treated in sewage treatment plants (STPs)
Sewage treatment is done in two stages- primary treatment and secondary biological) treatment
Primary Treatment
• This involves the physical removal of particles from the sewage through filtration and sedimentation.
• The effluent from the primary settling tank is taken for secondary treatment.
Secondary treatment (Biological treatment)

• The primary effluent is passed into large aeration tanks where it is constantly agitated mechanically and air is
pumped into it.
• This allows vigorous growth of useful aerobic microbes as flocs.
• These microbes consume the major part of the organic matter in the effluent. This significantly reduces the
BOD.
• Once the BOD of sewage is reduced significantly, the effluent is then passed into a settling tank where the
bacterial ‘flocs’ are allowed to sediment. This sediment is called activated sludge.
• A small part of the activated sludge is pumped back into the aeration tank to serve as the inoculum.
• The remaining major part of the sludge is pumped into large tanks called anaerobic sludge digesters. Here
anaerobic bacteria produce biogas (methane, hydrogen sulphide, and carbon dioxide).
• The effluent from the secondary treatment plant is generally released into natural water bodies like rivers and
streams.
MICROBES IN THE PRODUCTION OF BIOGAS
• Biogas is (containing methane) produced by the microbial activity of methanogen on cow dung and cellulosic
waste. Cow dung is a common source of methanogens e.g. Methanobacterium.
• The biogas plant consists of a concrete tank (10-15 feet deep) in which bio-wastes are collected and the slurry
of dung is fed.
• The Biogas plant has A floating cover placed over the slurry (which keeps on rising as the gas is produced in
the tank) and an outlet (supply biogas to nearby houses).
• The spent slurry is removed through another outlet and may be used as fertilizer. The biogas thus produced is
used for cooking and lighting.
MICROBES AS BIOCONTROL AGENTS
Biological control of pests and diseases
Biocontrol refers to the control of pests by microorganisms.

60
• The organic farmer, therefore, works to create a system where the insects that are sometimes called pests
are not eradicated, but instead are kept at manageable levels by a complex system of checks and balances
within a living and vibrant ecosystem.
• The use of biocontrol measures will greatly reduce our dependence on toxic chemicals and pesticides.
Ladybird aphids
Dragonflies mosquitoes
Bacillus thuringiensis Insects caterpillar
Trichoderma Several plant pathogens
Baculoviruses e.g. NPV insects
MICROBES AS BIOFERTILISERS
Bio-fertilizers are organisms that enrich the nutrient quality of the soil.

Rhizobium in the root nodule of leguminous plants fixes atmospheric nitrogen


Bacteria into organic forms, which is used by the plant as a nutrient. Azospirillum and
Azotobacter are also used as nitrogen-enriching biofertilizers.

Many species of fungus form a symbiotic association with the roots of plants
Fungi (Mycorrhiza). e.g.- Genus Glomus form mycorrhiza. The fungal mycelium absorbs
phosphorus from the soil and passes it to the plant. Such plants also show
resistance to root-borne pathogens and tolerance to salinity and drought.

Cyanobacteria- These blue-green algae may fix atmospheric nitrogen. E.g.


BGA Anabaena, Nostoc, Oscillatoria etc. Blue-green algae also add organic matter to
the soil and increase its fertility
.

IMPORTANT QUESTIONS
Very Short Answer Type Questions/ MCQ
1- Nostoc is economically very important due to its following feature-
a. N2 fixing bacteria
b. N2 fixing cyanobacteria
c. Mycorrhizae
d. None of these
Ans: B
2- Bacillus thuringiensis is widely used as:
a. Insecticide
b. Weedicides
c. Rodenticide
d. None of the above
Ans: A
3- Which one is used as a biocontrol agent-
a. Bacillus
b. NPV
c. Trichoderma
d. All of these
Ans: D
4- Full potential of Penicillin is established by which scientist?
a. Ernest Chain
b. Howard Florey
c. Both A and B
d. Only A
Ans: C
5- What is common in penicillin and ampicillin-

a. Antibiotic
b. Antibody
c. Immunoglobin
d. Antiseptic
Ans: A
6- Which organism is involved in curd formation? Mention the process of curd formation.
Ans: Lactic acid bacteria are used in the formation of curd.
The LAB produces lactic acid which coagulates and partially digests the milk proteins.
7- Swiss cheese has large holes. Give the reason. Also, mention the name of the microorganism responsible for
this.
Ans: the large holes in ‘Swiss cheese’ are due to the production of a large amount of CO2 by a bacterium named
Propionibacterium sharmanii.
8- Write the scientific name of baker’s yeast and brewer’s yeast.
Ans: Saccharomyces cerevisiae
9- What Nucleopolyhedra viruses (NPV) are being used nowadays?
Ans: NPV is used as a biocontrol agent for insect control
10- Name a biological product that is used to remove stains from clothes.
Ans: Lipase
11- List an immune immunosuppressive agent and also, mention the name of the source organisms.
Ans: Cyclosporin A, source organism: Trichoderma polysporum
12- Why bottled juice looks clearer than homemade juice.
Ans: Proteases and pectinases are used which digest pectin and make juice clearer.
13- Why is the secondary treatment of water in sewage treatment plants called biological treatment?
Ans: In this treatment Organic wastes of sewage water are decomposed by certain microorganisms in the
presence of water.
14- Name the plant whose sap is used in making Toddy. Mention the process involved in it.
Ans: Palm tree, Fermentation process
Short Answer Types Questions
1- Expand BOD. Mention its significance in the sewage treatment plant.
Ans: BOD refers to the amount of oxygen that would be consumed if all the organic matter in one liter of water
were oxidized by bacteria. The sewage water is treated till the BOD is reduced. The greater the BOD of waste
water, the more is polluted.
2- Alexander Fleming observed that in the presence of Penicillium notatum a particular species “A” can’t grow.
Give the reason and also identify “A”.
Ans: “A” is Staphylococci bacteria.
• ‘A’ is unable to grow because the chemical Penicillin (now called as antibiotic) is released by Penicillium
notatum
3- Give two examples each of distilled and non-distilled beverages.
Ans: Wine and beer (without distillation), whisky, and brandy (distillation) of the fermented broth
4- Name the type of association that genus Glomus exhibits with the higher plant. How it is beneficial for
plants?
• Ans: Mycorrhiza
• The fungal mycelium absorbs phosphorus from the soil and passes it to the plant. Such plants also
show resistance to root-borne pathogens and tolerance to salinity and drought.
5- From which organism we can obtain a clot buster? Write its use.
Ans: clot buster is obtained from Streptococcus. It is used for removing clots from the blood vessels of patients
who have undergone myocardial infarction.
6- How Flocs are formed during sewage treatment. Mention its application.
• Ans: Flocs are mesh-like structures containing aerobic bacteria and fungal mycelium.
• These are forms in aerobic tanks when organic matter is abundant.
• Flocs digest organic matter and reduce the BOD of sewage water.
7- Why is sewage water treated until the BOD is reduced? Give a reason.at which stage of sewage treatment
BOD reduces significantly.
• Ans: The greater the BOD of sewage water more is its polluting potential. So, the sewage water is
treated, till its BOD is reduced to reduce the organic matter present in it.
• At the secondary level during biological treatment flocs reduces BOD significantly.
8- A farmer is advised to add a culture of microorganisms in the soil before sowing the crop. Name two
microorganisms that can be helpful for farmers. How are these microbes useful to the crop?
Ans: bio-fertilizers: Rhizobium, Cyanobacteria
These microorganisms enrich the nutrient quality of the soil.
They can fix atmospheric nitrogen into organic forms, which is used by plants as a nutrient.
9- Why are biofertilizers or biopesticides preferred to chemical fertilizers or pesticides?
Ans: They do not cause any pollution.
They do not spoil the quality of the soil.
Biopesticides are target specific.
They are less expensive and are biodegradable.
10- Write the chemical nature of biogas. Name one organism which produces biogas. Name the institutions that
developed technology of biogas production in India.
Ans: Chemical nature of biogas – CH4 , CO2 , H2.
Organism: Group of bacteria known as methanogens. E.g. Methanobacterium
Indian Agricultural Research Institute (IARI) and Khadi and Village Industries Commission
CASE-BASED QUESTIONS:
1.Read the following and answer any four questions from (i) to (v) given below:
Antibiotics: An antibiotic is an organic substance which is produced by a microorganism which inhibits the
growth of other microbes. The very first antibiotic was produced using a fungus by Sir Alexander Fleming in
1929, known as penicillin from fungus Penicillium notatum which is also known as wonder drug. It has no
adverse effects on humans but kills gram positive type bacteria. Streptomycin is another one produced from
Streptomyces griseus, effective against gram negative bacteria. Some other antibiotics produced are
chloromycetin, tetramycin and aureomycin. Various animal and human diseases which are not cured by
other drugs are effectively cured by aureomycin. Another mostly used antifungal product isolated from fungi
is Griseofulvin. It is isolated from mycelium of penicillium. It is fungistatic, i.e., diminishes the growth by
interfering in the formation of fungal walls, it has dermatological uses in ringworms and athlete’s foot
disease. Some drugs obtained from fungi also work as immunosuppressants, for example cyclosporin A.
Psilocybin is a compound found in fungi such as Psilocybe semilanceata and Gymnopilus junonius, which
have been used for their hallucinogenic properties by various cultures for thousands of years.
(i) Antibiotics are the most effective on:
(a) Bacteria (b) Virus (c) Fungi (d) None of these
(ii) The yield of the antibiotic depends upon:
(a) age of the inoculum (b) the pH of the medium
(c) composition of the medium (d) all of the above
(iii) In Penicillium chrysogenum, the maximum antibiotic production occurs during the:
(a) second phase (b) third phase (c) first phase (d) in all three phases
(iv) What type of side effect is most commonly observed in beta-lactam antibiotics?
(a) Hearing loss (b) Aplastic anaemia (c) Allergic reaction (d) Yellowing of teeth
(v) Assertion: Discovery of the first antibiotic was a chance discovery.
Reason: Penicillin was the first antibiotic.
(a) Both A and R are true and the R is the correct explanation of the A.
(b) Both A and R are true but R is not the correct explanation of the A.
(c) A is true but R is false.
(d)A is false but R is true.
Ans. (i) (a) Bacteria (ii) (d) all of the above (iii) (a) The second phase (iv) (c) Allergic reaction. (v) (b) Both A and
R are true but R is not the correct explanation of A.

2. Biogas is produced by anaerobic digestion with methanogens or anaerobic organisms, who digest
material inside a closed system, or perform fermentation of biodegradable materials. This closed system is
called an anaerobic digester, bio-digester or a bioreactor. Proper use of waste material can be achieved
through biogas. The complex material is digested and is acted upon by several methanogenic bacteria.
(i) Write about the microbial activity during production of a biogas.
(ii) Enlist any four advantages of it.
Ans. (i) Biogas is produced by anaerobic digestion with methanogens or anaerobic organisms, which digest
material inside a closed system or perform fermentation of biodegradable materials. Biogas is primarily
methane (CH4) and carbon dioxide (CO2) and may have small amounts of hydrogen sulphide (H2S), moisture
and siloxanes. The gases such as methane, hydrogen and carbon monoxide (CO) can be combusted or
oxidised with oxygen. This energy release allows biogas to be used as a fuel; it can be used for any heating
purpose, such as cooking. It is used in a gas engine to convert the energy in the gas into electricity and heat.
(ii) Advantages of using Biogas:
(a) It is an eco-friendly source of energy.
(b) Biogas generation reduces soil and water pollution.
(c) The process of producing biogas yields organic manure
(d) It is a healthy cooking alternative as it is low cost technology and economic for developing nations.

ASSERTION AND REASON TYPE QUESTIONS


In the following questions a statement of assertion followed by a statement of reason is given.
Choose the correct answer out of the following choices.
(a) Assertion and reason both are correct statements and reason is correct explanation for assertion. (b) Assertion and
reason both are correct statements but reason is not correct explanation for assertion.
(c) Assertion is correct statement but reason is wrong statement.
(d) Assertion is wrong statement but reason is correct statement.
1. Assertion : Chemical fertilisers are more preferable than biofertilizers.
Reason : Chemical fertilisers are expensive and hazardous to the environment.
2. Assertion : A small part of activated sludge is pumped back into aeration tank.
Reason : It serves as inoculum.
3. Assertion : Streptococcus thermophilus increases nutritional value of milk.
Reason : Curd and yoghurt have higher vitamin content than milk.
4. Assertion : For organ transplantation Cyclosporin A needs to be injected to the patient.
Reason : Cyclosporin A inhibits activation of T-cells and interferons.
5. Assertion : Primary treatment of sewage is also called bilogical treatment.
Reason : Primary sewage treatment depends only on density of materials in sewage.
6. Assertion : Acetic acid production involves both aerobic and anaerobic processes.
Reason : First alcohol is produced from glucose by aerobic process which is followed by production of acetic by
anaerobic process.
7. Assertion : Baculoviruses control growth of many insects and arthropods.
Reason : Lady bird and Trichoderma are used as biocontrol agents.
8. Assertion : Integrated Pest Management (IPM) uses different pest control methods which are ecofriendly.
Reason : Bacillus thuringiensis kill larvae of certain insects.
9. Assertion : Biogas is produced by anaerobic digestion of biomass by methanogenic bacteria.
Reason : Biogas is made up of methane entirely and is the most eco friendly fuel.
10. Assertion : In mycorrhiza the fungus symbiont absorbs phosphorus for the plant.
Reason : Mycorrhiza is a symbiotic association of fungus with roots of higher plants
Answers 1. (d) 2. (a) 3. (a) 4. (a) 5. (d) 6. (c) 7. (b) 8. (b) 9. (c) 10. (b)
Long Answer Type Questions
1- Explain the process of secondary sewage treatment.
Ans: •The primary effluent is passed into large aeration tanks where it is constantly agitated mechanically
and air is pumped into it.
•This allows vigorous growth of useful aerobic microbes as flocs. These microbes consume the organic matter
in the effluent. This significantly reduces the BOD.
•Once the BOD of sewage is reduced significantly, the effluent is then passed into a settling tank where the
bacterial ‘flocs’ are allowed to sediment. This sediment is called activated sludge.
•The major part of the sludge is pumped into anaerobic sludge digesters. Here anaerobic bacteria produce
biogas (methane, hydrogen sulphide and carbon dioxide).
•The effluent from the secondary treatment plant is generally released into natural water bodies like rivers
and streams.
2- (i) Draw a typical biogas plant.
(ii) Describe how biogas is obtained from the activated sludge?
Ans: (i) Fig. 10.8, Page 186, NCERT
(ii) Biogas formation from activated sludge:
• Major portion of activated sludge is pumped into anaerobic sludge digesters.
• Here, anaerobic bacteria digest the organic material of the sludge.
• During this digestion, the methanogen bacteria produce biogas (methane, carbon dioxide hydrogen
sulphide)
3- Organic farmers use microbes to decrease the use of chemical pesticides. By giving five examples, Explain
how can this be accomplished.
Ans: Organic farmer believes that the use of biocontrol measures will greatly reduce our dependence on
toxic chemicals and pesticides.
Ladybird Aphids
Dragonflies Mosquitoes
Bacillus thuringiensis Insects caterpillar
Trichoderma Several plant pathogens
Baculoviruses, NPV Insects
4- Microbes may be very useful for human beings. Give at least five examples in favour of your answer.
Ans: Microbes are the major components of biological systems on this earth. They may be very useful for
mankind in the following ways-
• As biofertilizers- Rhizobium, Cyanobacteria, and Azotobacter are goof nitrogen-fixing organisms
• Biopesticides- Trichoderma, NPV, Dragonfly, and Bacillus thuringiensis are good biopesticides.
• Sewage treatment and Biogas formation- Aerobic bacteria and methanogens
• Antibiotics- Penicillum notatum produces Penicillin
• Food and beverages – LAB- curd formation, (Saccharomyces cerevisiae) in the bakery and brewers industry
5- Various bioactive molecules and chemicals are produced from microorganisms by using modern
technology. Give a brief account of them
Ans:
Microorganism Name Product
Fungus Aspergillus niger Citric acid
Bacterium Acetobacter aceti Acetic acid
Bacterium Clostridium butylicum Butyric acid
Bacterium Lactobacillus Lactic acid
Yeast Saccharomyces cerevisiae Ethanol
Lipases are used in detergent formulations and are helpful in removing oily stains
from the laundry.
Bottled fruit juices bought from the market are clearer as compared to those made
at home. This is because the bottled juices are clarified by the use of
pectinases and proteases.
Product Microorganism Use
Streptokinase Streptococcus clot buster
Cyclosporin A Trichoderma polysporum immunosuppressive agent
Statins Monascus purpureus Blood-cholesterol lowering
agents.
CHAPTER 11
Key words BIOTECHNOLOGY: PRINCIPLES AND PROCESSES
Biotechnology ‘The integration of natural science and organisms, cells, parts thereof, and
molecular analogues for products and services.
Recombinant DNA Desired DNA + Vector DNA
Origin of replication which is responsible for initiating replication
Plasmid autonomously replicating circular extra-chromosomal DNA
Restriction enzymes Cut DNA at specific locations
Vector Transfer DNA/r-DNA to the host
Cloning Obtaining multiple copies of rDNA
Exonucleases Exonucleases remove nucleotides from the ends of the DNA
Endonucleases Cuts at a specific position in palindromic sequence within the DNA
Palindromic sequences The same reading frame on opposite polarity strand of DNA
ampR , tet R Ampicillin and Tetracycline resistant gene
DNA ligases Ligate adjacent nucleotides
Electrophoresis Separation of charged particles under the influence of electric current.
Elution Obtaining DNA from agarose gel
Downstream processing It is the separation and purification of the product.
PRINCIPLES OF BIOTECHNOLOGY
Genetic engineering: Techniques to alter the chemistry of genetic material (DNA and RNA).
Bioprocess engineering: Maintenance of sterile (microbial contamination-free) conditions to enable the
growth of only the desired microbe/eukaryotic cell in large quantities for the manufacture of
biotechnological products like antibiotics, vaccines, enzymes, etc.

Three basic steps in genetically modifying an organism-

TOOLS OF RECOMBINANT DNA TECHNOLOGY

Restriction enzymes
• A restriction enzyme is a type of nuclease enzyme that cleaves DNA at specific sites, producing DNA
fragments with a known sequence.
• The nuclease enzyme may be – exonucleases and endonucleases.
• Example of restriction endonuclease- Eco RI
• Nomenclature of restriction endonuclease –
E Genus, Escherichia

The palindromic sequence of EcoRI co Species, coli

R Strain

I Order of isolation
PROCESS OF r-DNA FORMATION

ELECTROPHORESIS
• This technique is used to separate cleaved DNA sequences as bands based
on their size.
• The DNA samples are loaded into wells on an agarose gel.
• When electricity is passed, being negatively charged the DNA molecules
move toward the positive charge end.
• DNA fragments can be visualized by using Ethidium bromide under UV
lamp.
CLONING VECTORS
Cloning vectors are utilized to insert desired DNA into another host cell and create multiple copies of the same. The
main properties of cloning vectors are-
1- Origin of replication (ori): This is responsible for replication. This sequence is also responsible for controlling
the copy number of the linked DNA.
2- Selectable marker: It helps in identifying and eliminating non-transformants. These may be chemical markers
(ampicillin, chloramphenicol, tetracycline, or kanamycin) and phenotypic marker (beta-galactosidase).
3- Cloning sites: It is the site where desired DNA is inserted into the vector.
Selection of transformants and recombinants

Vectors for cloning genes in plants and animals


Agrobacterium tumifaciens – This plant pathogen is now used as a cloning vector.
Retroviruses – these are used as vectors in animals.
Competent Host (For Transformation with Recombinant DNA)
Method Explanation
Divalent cation Treating a cell with a specific concentration of a divalent
cation, such as calcium, increases the cell’s efficiency to take
DNA.
Heat and shock incubating the host cell under heat and then ice, creates
pores on the wall through which DNA enters the bacterium.
Micro-injection r- DNA is directly injected into the nucleus of an animal cell

Biolistic/ Gene gun method plant cells are bombarded with high-velocity micro-particles
of gold or tungsten coated with DNA.
Disarmed pathogen vectors Infect host cell and transfer r- DNA into them

Isolation of the Genetic Material (DNA)

Chemical Used for Chemical Used for


Lysozyme To break the bacterial cell wall. Cellulase To break the plant cell wall
Chitinase To break the fungal cell wall. Ribonuclease Removes RNA
Protease Removes proteins (histones Chilled ethanol Precipitate DNA
associated with DNA).
Amplification of Gene of Interest using PCR-
• Artificial method (invitro) of DNA cloning.
• The specific enzyme Taq Polymerase a type of DNA
Polymerase used is isolated from Thermus aquaticus
bacteria. The enzyme can withstand very high
temperatures.
Steps involved in PCR –

BIOREACTOR
• It is the cylindrical vessel of large volumes (100-1000 litres) in which culture can be processed to obtain desired
products by using downstream processing.
• In bioreactors, r-hosts are provided with optimal conditions (nutrient, temperature, pH, substrate, salts,
vitamins, oxygen) for culture recombinant cells.
Bioreactors are of two types-

Simple stirred tank bioreactor- cylindrical Sparged stirred-


or with a curved base to facilitate the tank bioreactor-
mixing of the reactor contents and the sterile air is
stirrer facilitates even mixing and oxygen sparged through
availability throughout the bioreactor the reactor

IMPORTANT QUESTIONS
Very Short Answer Type Questions
1- The cutting of DNA at a specific location became possible with –
a. Restriction enzymes
b. Ligases
c. Both a and b
d. None of these
Ans: b
2- Electrophoresis is a technique used in laboratories in order to separate
a. DNA
b. Lipoprotein
c. Lactose
d. All of these
Ans: a
3- The role of antibiotics in genetic engineering is chiefly as-
a. Ori C
b. Selectable markers
c. Both a and b
d. None of these
Ans: b
4- joins two DNA fragments together via a phosphodiester bond between two nucleotides.
a. DNA Polymerase
b. Primase
c. DNA Ligase
d. DNA topoisomerase
Ans: c
5- Site of origin controls replication in circular plasmid DNA
a- Ori
b- Selectable marker
c- rop
d- all of these
Ans: a
6- Selectable markers define biotechnology as given by the European Federation of Biotechnology (EFB).
Ans: ‘The integration of natural science and organisms, cells, parts thereof, and molecular analogues for
products and services.
7- Why alien DNA is linked with the origin of the replication sequence?
Ans: Alien DNA is linked with the origin of replication, so that, this alien piece of DNA can replicate and
multiply itself in the host organism.
8- Write the recognition sequence/ cleavage site of Eco RI.
Ans:

9- Nowadays says which matrix is used in gel electrophoresis? From which source one can get gel used in
electrophoresis?
Ans: Nowadays the most commonly used matrix is agarose. The main source of this is seaweed.
10- Which dye is used to visualize DNA bands on electrophoresis?
Ans: Ethidium bromide-stained gel exposed to UV light
11- Give examples of any four antibiotics which are used as selectable markers in the cloning vector.
Ans: ampicillin, chloramphenicol, tetracycline or kanamycin
12- Why it is considered that a single recognition site in a vector is better than multiple recognition?
Ans: The presence of more than one recognition site within the vector will generate several
fragments, which will complicate gene cloning.
13- Why are proteases added while isolating the DNA?
Ans: to remove histone proteins from DNA
14- What is T- DNA? Write about its importance in genetic engineering.
Ans: it is a piece of DNA of Agrobacterium tumefacient. T-DNA transform normal plant cells into a tumour
cell.
15- Name one widely used artificial vector and one natural vector used to make GMOs.
Ans: pBR322, Retrovirus
Short Answer Type Questions

1- Write nomenclature method of restriction endonuclease EcoRI.


Ans: E -Genus, Escherichia, co - Species, coli, R- Strain, I- Order of isolation
2- Explain the one advantage of using Agrobacterium in genetic engineering.
Ans: Agrobacterium tumefaciens is a bacterial plant pathogen. It infects plants and causes crown gall disease.
This disease is induced by Ti plasmid (tumour-inducing plasmid). The Ti plasmid incorporates T-DNA into the
host plant cell.
3- What are two core techniques that enabled the birth of modern biotechnology?
Ans: Genetic engineering: Techniques to alter the chemistry of genetic material (DNA and RNA).
Bioprocess engineering: Maintenance of sterile (microbial contamination-free) for optimal growth of desired
microbes.
4- What do you understand by the term selectable marker? Give two examples of selectable markers.
Ans: A selectable marker is a gene which is used in the selection of transformants and recombinants.
Example: phenotypic marker (antibiotic-resistant gene like ampR, tetR) and genotypic marker (beta-
galactosidase)
5- List three basic steps required in the creation of GMOs (genetically modifying an organism).
Ans: (i) identification of DNA with desirable genes;
(ii) introduction of the identified DNA into the host;
(iii) maintenance of introduced DNA in the host and transfer of the DNA to its progeny.
6- Draw a typical agarose gel electrophoresis showing the migration of DNA fragments and label any four parts.
Ans: Fig11.3, page 198, NCERT
7- What are restriction enzymes? From where one can isolate it? How many linear DNA fragments will be
produced when a circular plasmid is digested with a restriction enzyme having two sites?
Ans: Restriction enzymes are a type of nuclease enzyme that can cut DNA fragments.
One can isolate it mainly from bacteria and a few other microorganisms.
Four fragments will produce.
8- Who constructs the recombinant DNA? Which organism does he use for this purpose?
Ans: Cohen and H. Boyer constructed the first r- DNA using antibiotic-resistance genes present on the plasmid.
He used the bacterium Salmonella typhimurium.
9- What features make plasmid a good cloning vector?
Ans: Plasmids are low molecular weight extrachromosomal genetic material, it has an origin of replication
centre, self-replication, have cleavage site for restriction endonucleases, selectable markers and are easily
available.
10- (i) Name the enzymes that are used for the isolation of DNA from bacterial and fungal cells for rDNA
technology.
(ii) Why is enzyme cellulase used for isolating genetic material from plant cells and not from animal cells?
Ans: (i) Lysozyme for bacterial cells and chitinase for the fungal cell.
(ii) Cellulase is used to break plant cell walls, animal cells lack a cell wall therefore cellulose will not work
on animal cells.
ASSERTION AND REASONING QUESTIONS
These questions consist of two statements each, printed as Assertion and Reason.
While answering these questions you are required to choose any one of the following four
responses.
A .If both Assertion and Reason are true and the Reason is correct explanation of the
Assertion.
B. If both Assertion and Reason are true but the Reason is not a correct explanation of the
Assertion.
C. If Assertion is true but the Reason is false.
D. If both Assertion and Reason are false .
QUESTIONS-
1. Assertion(A): Exonucleases remove nucleotides from the ends of DNA.
Reason(R): Endonucleases make cuts at specific positions within the DNA.
2. Assertion(A): The cut pieces of DNA are linked with plasmid DNA.
Reason(R): Plasmid DNA fails to act as Vectors.
3. Assertion(A): Recombinant DNA technology has become successful because of the
presence of restriction endonucleases in eukaryotic cells.
Reason(R): Restriction endonucleases cut the DNA molecule to form blunt ends.
4. Assertion(A): Genetic engineering overcomes the drawbacks of traditional hybridisation.
Reason(R): Genetic engineering involves creation of a recombinant DNA and introduce the
desirable genes into target organisms.
5. Assertion(A): Asexual reproduction preserves the genetic information.
Reason(R): Sexual reproduction permits variation.
6.. Assertion (A): Enzyme application in industry is enhanced by its immobilization.
Reason (R): Immobilization provides protection to enzymes without affecting their activity.
7. Assertion (A): Restriction endonucleases are also called ‘molecular scissors’.
Reason (R): When fragments generated by restriction endonucleases are mixed, they join
together due to their sticky ends .
ANSWERS:
1. B 2. C 3.D 4.A 5.B 6.A 7. B
CASE STUDY BASED -1
Read the following text and answer the following questions on the basis of the same:
The term “Biotechnology” refers to the use of living organisms or their products to
modify human health and their human environment. For example, ‘test tube’
programmes, synthesis of a gene or correcting a defective gene are all part of the
biotechnology. The basis of the modern biotechnology is genetic engineering and
maintenance of sterile conditions. Genetic engineering is the techniques that alter
the chemistry of genetic material i.e., DNA and RNA, then this genetic material is
introduced intohost organisms, which alter the phenotype of the host organism.
1) Discovery of----- molecule made genetic engineering possible.
(a) Restriction exonuclease (b) Restriction endonuclease
(b) (c) Ribozyme (d) DNA polymerase Ans b.
2) The recognition sequence of the first restriction enzyme isolated was base
pair long.
(a) Four (b) Five (c) Six (d) Two Ans c.
3) The specific DNA sequence where EcoRI cuts is:
(a) GATTCG (b) GAATTC (c) GTTCAA (d)
TTCCAAAns b
4) The cutting of DNA at specific locations became possible with the discovery of:
(a) Ligases (b) Restriction enzyme (c) Probes (d)
Selectable markerAns b
5) DNA fragments are:
(a) Positively charged (b) Negatively charged
(c) Neutral (d) Either positively or negatively charged depending
on their size
Ans b
CASE STUDY BASED -2
Restriction endonuclease was isolated for the first time by W. Arber in 1962 in
bacteria. Restriction endonucleases cut the DNA duplex at specific points
therefore they are also called as molecular scissors or biological scissors. Three
types of restriction endonuleases are Type 1. Type II and Type III but only Type II
restriction endonucleases are used in recombinant DNA technology. Restriction
endonuclease EcoR I recognizes the base sequence GAATTC in DNA duplex and
cut strands between G and A.
(i) Only type II restriction enzymes are used in gene manipulation because
(a) ATP is not required for cleaving (b) it consists of three different subunits
(c) it makes cleavage or cut in both the strands of DNA molecule
(d) both (a) and (c).
Ans-I) (d)
(ii) Which of the following ions are used by restriction endonucleases for restriction?
(a) Mg ions (b) Mn ions (c) Na ions (d) K ions
Ans- ii)(a)
(iii) Restriction endonuclease was isolated for the first time in a
(a) plant cell (b) animal cell (c) prokaryotic cell (d) germinal cell.
Ans- iii)(c)
(iv) Restriction endonucleases are also called as molecular or biological scissors because
(a) they cleave base pairs of DNA only at their terminal ends
(b) they cleave one or both the strands of DNA
(c) they act only on single stranded DNA
(d) none of these.
Ans- iv)(b)
(v) Select the option that correctly states the working action of restriction endonuclease
EcoR 1 on DNA sequence GAATTC.
(a) 5-GAATTC-3′ 3-CTTAAG-5' (b)5′-GAATTC-3′ 3-CTTAAG-5
(c)5'-GAATTC-3′ 3-CTTAAG-5 (d) 5-GAATIC-3 3-CTTAAG-5
Ans- v)(b)
Long Answer Type Questions

1- (i) Explain the correct sequential step of the polymerase chain reaction.
(ii) Which enzyme is used for PCR and why?
(iii) What s the source of that enzyme?
Ans: (i) the correct steps are
• Denaturation- separation of ds DNA into ss DNA at high temperature (92℃).
• Annealing- attachment of primers towards 3’ end of both strands.
• Extension- To polymerization on primer to form new ds DNA.
(ii) Enzyme used is Taq DNA Polymerase.
(iii) Source of the enzyme is the bacteria Thermus aquaticus.
2- What is Bioreactor? What are the advantages of Stirred tank Bioreactor? Show diagrammatically a simple
Stirred tank Bioreactor.
Ans: Bioreactors are large vessels in which raw materials are biologically converted into specific biological
products.
Stirred tank Bioreactor provides optimal conditions (temp., pH etc), it also has an agitation system and foam
control system for better growth.
Fig. 11.7 (a), page 204, NCERT
3- Diagrammatically represent the steps in the formation of recombinant DNA by the action of restriction
endonuclease enzyme – EcoRI.
Ans: Fig. 11.1 page 196 NCERT
4- What is genetic engineering? Explain briefly the different steps involved in genetic engineering technology.
Ans: Genetic engineering is a technique of modification/alteration of the genome and its application for human
welfare.
Steps:
• Isolation of genetic material containing the desired gene.
• Cleavage of the sequence of DNA containing the gene of interest and vector gene with the same
restriction endonuclease.
• Amplification of gene using PCR.
• Formation of r- DNA by ligating vector DNA and gene of interest
• Using gene transfer technology transfer of r- DNA into the host cell.
5- Explain any four methods of vector-less gene transfer and one method which involve vectors.
Ans:
Method Explanation
Vector less Method
Divalent cation Treating a cell with a specific concentration of a divalent cation,
such as calcium, increases the cell’s efficiency to take DNA.
Heat and shock incubating the host cell under heat and then ice, create pores
on the wall through which DNA enters the bacterium.
Micro-injection r- DNA is directly injected into the nucleus of an animal cell
Biolistic/ Gene gun plant cells are bombarded with high-velocity micro-particles of
method gold or tungsten coated with DNA.
Vector mediated
Disarmed pathogen Infect host cell and transfer r- DNA into them
CHAPTER 12
BIOTECHNOLOGY AND ITS APPLICATION
KEYWORDS
GMOs these are genetically modified living organisms like plants,
Genetically modified organisms animals and microorganisms.
Biopesticide The pesticide of biological origin
PCR Polymerase chain reaction
ELISA Enzyme-linked immunosorbent assay
Bt Bacillus thuringiensis
Transposons These are mobile genetic elements that replicate via an RNA
intermediate.
Biopiracy Use of bioresources with proper authentication

THREE CRITICAL RESEARCH AREAS OF BIOTECHNOLOGY:

APPLICATION IN AGRICULTURE

ADVANTAGES OF GENETIC MODIFICATION IN PLANTS:

PEST RESISTANT PLANTS


• They act as bio-pesticide.
• It reduces the need for insecticides.
• E.g. Bt cotton, Bt corn, rice, tomato, potato, soybean, etc.
Bt Cotton:

• Some strains of bacteria Bacillus thuringiensis have proteins that kill insects like coleopterans (beetles),
lepidopterans (tobacco budworm, armyworm) & dipterans (flies, mosquitoes).
• B. thuringiensis forms Bt toxin. Bt toxin is a crystalline insecticidal protein,
• When an insect ingests the toxin, it becomes active due to alkaline pH in the midgut of the insect and creating
pores. This causes the death of the insect.
• Bt toxin genes were isolated from bacteria B. thuringiensis and incorporated into crop plants such as cotton by
using genetic engineering.
CRY Genes

Cry genes code Bt toxin. Cry genes are of the following types-
cryIAc & cryIIAb control cotton bollworms cryIAb controls corn borer
NEMATODE RESISTANCE IN TOBACCO PLANTS

RNAi (RNA Interference)/ RNA Silencing

•Nematode - Meloidogyne incognita


•Plant- Tobacco
•Vector used- Agrobacterium
•A nematode infects the roots of tobacco plants.
•The process of RNA interference (RNAi) involves silencing of a specific mRNA due to a complementary dsRNA
(double-stranded m RNA) molecule that binds to and prevents translation of the mRNA, also referred to as
RNA silencing.
• The source of this complementary RNA could be from infection by viruses having RNA genomes or transposons.
• Using Agrobacterium vectors, nematode-specific genes were introduced into the host plant such that it
produced both sense and anti-sense RNA in the host cells.
• Two RNA’s being complementary to each other forming a dsRNA that initiated RNAi and thus, silenced the
specific mRNA of the nematode.
ADVANTAGES OF GENETIC MODIFICATION IN PLANTS

• It makes crops more tolerant to abiotic stresses (cold, drought, salt, heat etc.).
• Pest-resistant crops reduce the use of chemical pesticides.
• It reduces post-harvest losses.
• It increases plant mineral usage efficiency (it prevents soil fertility's early exhaustion).
• It enhances the nutritional value of food. E.g. Golden rice (Vitamin A enriched rice).
• To create tailor-made plants to supply alternative resources (starches, fuels, pharma etc)
APPLICATIONS IN MEDICINE

Genetically Engineered Insulin

• Insulin from the pancreas of animals (cattle & pigs) causes allergy or other types of reactions.
• Human insulin consists of two short polypeptide chains (chain A & chain B) that are linked by disulphide bridges
• Another chain C peptide is removed during the maturation process
• In 1983, Eli Lilly (an American company) prepared two DNA sequences corresponding to A & B chains of human
insulin and introduced them in plasmids of E. coli to produce insulin chains.
• Chains A & B were combined by creating disulphide bonds to form human insulin (Humulin).
GENE THERAPY

• Gene therapy is a method that allows the correction of a faulty gene by a correct and functional gene.
• First evidence - 1990 to a 4-year-old girl with adenosine deaminase (ADA) deficiency which is caused due to
the deletion of the gene for adenosine deaminase. ADA enzyme is crucial for the immune system to function.

Lymphocyte culture - Lymphocytes from the blood of the patient are grown in a culture outside
the body and a functional ADA cDNA (using a retroviral vector) is then introduced into these
lymphocytes using a retroviral vector which is returned to the patient. It is not a permanent
method as lymphocytes are not immortal

Enzyme replacement- in this functional ADA is given to the patient by injection. This required
periodic infusion of injection.

Bone marrow transplantation- If the ADA gene from marrow cells is introduced into cells at
early embryonic stages, it could be a permanent cure.
MOLECULAR DIAGNOSIS

PCR (Polymerase Chain Reaction) A very low concentration of a bacteria or virus can be detected
by amplification of their nucleic acid by PCR.
To detect HIV in suspected patients.
To detect gene mutations in suspected cancer patients.
ELISA (Enzyme-Linked Immuno- It is based on antigen-antibody interaction.
Sorbent Assay) To detect HIV- AIDS
Autoradiography To find out mutated genes

STEM CELL TECHNOLOGY

Stem Cells are present in the inner cell mass of the embryo, and bone marrow. It has the ability to develop in all types
of tissues and organs. These cells are also involved in the development, growth, and repair of the organism. These
properties are used to treat many diseases such as heart disease, kidney disease, type-I diabetes, arthritis, muscular
dystrophy, etc.

TRANSGENIC ANIMALS

Transgenic animals are animals with a modified genome. Their genome has been manipulated by using genetic
engineering technology.
Examples: Transgenic plant- Bt Cotton, Bt brinjal
Transgenic animals – rats, rabbits, pigs, sheep, cows and fish
Benefits of transgenic animals:


ETHICAL ISSUES
Ethical standards are required to evaluate the morality of all human activities, and to validate GM research. For this
purpose, Govt of India established GEAC (Genetic Engineering Approval Committee).

BIO-PIRACY

It is use of bio-resources by multinational companies and other organizations without proper authorisation from the
countries and people concerned without compensatory payment developed and used by farmers and indigenous
people of a country.

Important Question
Very short answer type questions / MCQ
1- Genetically engineered bacteria have been used in commercial production of
a. Thymine
b. Testosterone
c. Human insulin
d. All of these
Ans: c
2- Biopiracy is
a. Unauthorized use without permission
b. Authorized use without permission
c. None of these
d. All of these
Ans: a
3- The process of RNA interference has been used in the development of plants resistant to _
a. RNAi b- DNAi c- Insertional inactivation d-None of these
Ans: a
4- ELISA is based on-
a. Ag- IFN interaction
b. Ag- Ab interaction
c. DNA- RNA interaction
d. mRNA – DNA interaction
Ans: b
5- How human protein (α-1-antitrypsin) is used in the medical field.
a. To treat diabetic
b. To remove clot
c. Immunosuppressive agent
d. To treat emphysema
Ans: d
6- Cry gene is extensively used in genetic engineering to develop transgenic plants like cotton. Name the
source organism from which one can get the cry gene.
Ans: Bacillus thuringiensis
7- Which organism badly affects the crop of tobacco?
Ans: Nematode-Meloidogyne incognita
8- Human proinsulin has three polypeptide chains. Name all these chains and which chain is not present in
mature insulin.
Ans: chain A, B and C. Chain C are not present in mature insulin.
Ans: to treat emphysema
9- What is GEAC? Write its role.
Ans: GEAC (Genetic Engineering Approval Committee), which will make decisions regarding the validity of GM
research and the safety of introducing GM organisms to public services.
10- By giving one example clarify that GMOs enhance the nutritional value of food.
Ans: Golden rice, i.e., Vitamin ‘A’ enriched rice.
11- Which vector is used to create a pest-resistant tobacco plant?
Ans: Agrobacterium vectors
12- Specify one consequence of adenosine deaminase (ADA) deficiency.
Ans: adenosine deaminase (ADA) is crucial for the immune system to function
13- Write any two roles of PCR in molecular diagnosis technique.
Ans: HIV detection, identification of mutated gene.
Short answer type questions

1- How is a probe used in molecular diagnostics?


Ans: A probe is a ss DNA or RNA used to search for its complementary sequence in a sample genome.
By the process of hybridization and autoradiography, the probes can be identified.
2- What is biopiracy?
Ans: use of bio-resources by multinational companies and other organisations without proper authorisation
from the countries and people concerned without compensatory payment.
3- Developed nations are exploiting the bioresources of under-industrialised nations. Justify the statement
with a suitable example.
Ans: 27 varieties of Basmati are grown in India. There is a reference to Basmati in ancient texts, folklore and
poetry, as it has been for centuries. In 1997, an American company got patent rights on Basmati rice through
the US Patent and Trademark Office. This allowed the company to sell a ‘new’ variety of Basmati, in the US and
abroad.
4- Transgenic animals can be used in various ways. How could be a transgenic mouse helpful?
Ans: Transgenic mice are developed for testing the safety of vaccines before they are used on humans.
example: polio vaccine.
5- How crystal protein acts in Bt Cotton?
Ans: Cry gene of Bacillus thuringiensis is introduced in the cotton plant to form insect resistant plant. When
insects feed on the cotton plant the inactive crystal protein becomes active due to the alkaline pH in the midgut
of insects. The protein creates pores in the midgut and ultimately insects die.
6- Give two examples of biological products obtained by using genetic engineering?
Ans: human protein (α-1-antitrypsin) is used to treat emphysema. Rosie cow produced 2.4 g/ The milk
contained the human alpha-lactalbumin and was nutritionally a more balanced product for human babies than
natural cow-milk
Long Answer Type Questions
1- Plants bacteria, fungi and animals whose genes have been altered by manipulation are called Genetically
Modified Organisms (GMO). GM plants have been useful in many ways. Give at least five examples in
support of the statement.
Ans:
• made crops more tolerant to abiotic stresses (cold, drought, salt, heat).
• reduced reliance on chemical pesticides (pest-resistant crops).
• helped to reduce post-harvest losses.
• increased efficiency of mineral usage by plants
• enhanced nutritional value of food, e.g., golden rice, i.e., Vitamin ‘A’ enriched rice
2- What are the advantages of insulin obtained from genetic engineering? Explain the process of its
formation.
Ans: Insulin obtained from genetic engineering is cast effective and shows no adverse effect on the body.
Insulin has polypeptide chains A and B.
The chain A and B is synthesized in the different host using r—DNA technology.
These chains are isolated and joined with the help of a disulphide bond.
3- Explain different processes of Gene Therapy.
Ans: • Lymphocyte culture - Lymphocytes from the blood of the patient are grown in a culture outside the
body and a functional ADA cDNA (using a retroviral vector) is then introduced into these lymphocytes using a
retroviral vector which is returned to the patient. It is not a permanent method as lymphocytes are not
immortal.
•Enzyme replacement- in this functional ADA is given to the patient by injection. This required periodic infusion
of injection.
•Bone marrow transplantation- If the ADA gene from marrow cells is introduced into cells at early embryonic
stages, it could be a permanent cure.
4- What is RNA Silencing? How is this strategy used to create pest-resistant plants?
Ans: RNA silencing involves silencing specific mRNA. This causes a stoppage of translation.
This is achieved in the bacco plant against nematode Meloidogyne incognita.
In RNAi complementary ds RNA is produced against specific mRNA.
Using the Agrobacterium vector, nematode-specific genes were introduced into the host plant. The
introduction of DNA was such that it produced both sense &anti-dense RNA in the host cell.
These two RNA’s being complementary to each other formed a ds RNA that initiated RNAi.
5- What are transgenic animals? Enlist any four reasons for their production.
Ans: Transgenic animals or GMOs are those organisms whose genetic material has been altered by using
genetic engineering techniques.
Reason of their production
• To study the effect of genes on normal physiology and development
• To study diseases like cancer, cystic fibrosis, rheumatoid arthritis and Alzheimer’s
• To obtain biological products. E.g. human protein (α-1-antitrypsin)
• For study of Vaccine and chemical testing and safety
ASSERTION/ REASONING
A. If both Assertion & Reason are true & reason is the correct explanation of Assertion
B. If both Assertion & Reason are true & reason is not the correct explanation of Assertion
C. If Assertion is true & Reason is false
D. If both Assertion & Reason are false.
i) Assertion: ELISA is widely used for the detection of infectious diseases like AIDS.
Reason: ELISA is a very sensitive and selective test and needs a very small amount of reagents.
Ans. B. If both Assertion & Reason are true & reason is not the correct explanation of Assertion
ii) Assertion: Vaccination is also called preventive inoculation.
Reason: Vaccine prevents the formation of antibodies inside the body.
Ans. C. If Assertion is true & Reason is false
iii) Assertion: DNA fingerprinting involves identifying differences in some specific regions in DNA sequence.
Reason: In repetitive DNA sequences, a small stretch of DNA is repeated many times.
Ans. A. If both Assertion & Reason are true & reason is the correct explanation of Assertion
iv) Assertion: Genetic engineering can overcome the drawbacks of traditional hybridisation.
Reason: Genetic engineering can create desired DNA sequences to meet specific requirements.
Ans. A. If both Assertion & Reason are true & reason is the correct explanation of Assertion
v) Assertion: In recombinant DNA technology, human genes are often transferred into bacteria (prokaryotes)
or yeast (eukaryotes).
Reason: Both bacteria and yeast multiply very fast to form huge populations which express the desired gene.
Ans. A. If both Assertion & Reason are true & reason is the correct explanation of Assertion
vi) Assertion: Bt toxin gene has been cloned from bacteria and expressed in plants to provide resistance from
insects without the need of insecticides.
Reason: Bt toxin is produced from bacterium Bacillus thuringiensis.
Ans. A. If both Assertion & Reason are true & reason is the correct explanation of Assertion
vii) Assertion: Interferons are a type of antibodies produced by cells infected with bacteria.
Reason: Interferons stimulate inflammation at the site of injury.
Ans. D. If both Assertion & Reason are false.
viii) Assertion: Biopiracy is the practice of commercially exploiting naturally occurring biochemical or genetic
material especially by obtaining patents that restricts its future use while failing to pay fair compensation to the
community from which it originates.
Reason: US patented turmeric and neem which is a case of biopiracy.
Ans. B. If both Assertion & Reason are true & reason is not the correct explanation of Assertion
ix) Assertion: Transgenic animals are used to study the physiology and development of an organism.
Reason: Transgenic animals are specifically designed to allow the study of regulation of genes.
Ans. A. If both Assertion & Reason are true & reason is the correct explanation of Assertion
x) Assertion: Agrobacterium tumifacious is popular in genetic engineering because this bacterium is
associated with the roots of all cereals and pulse crops
Reason: A gene incorporated in the bacterial chromosome gets automatically transferred into crop with which
the bacteria (prokaryote) is associated.
Ans. D. If both Assertion & Reason are false.
xi) Assertion: Humulin is more effective than the insulin produced by conventional methods.
Reason: Humulin is absorbed rapidly in the blood than the conventionally produced insulin.
Ans. A. If both Assertion & Reason are true & reason is the correct explanation of Assertion
xii)Assertion: Organisations like GEAC are necessary to monitor GM researches and test the safety of
introducing GM organisms for public services.
Reason: GM researches can have unpredictable results which can be disastrous when genetically modified organisms
are introduced into the ecosystem.
xiii) Assertion: The antibiotics produced by Streptomyces species have great commercial applications.
Reason: Some life saving antibiotics like Penicillin and polymixin -B are produced by Streptomyces
Ans. C. If Assertion is true & Reason is false

CCTCASE BASED-1
i) Name the pest that destroys the cotton bolls.
a) nematode b) bollworms c) grasshopper d) plasmid Ans. b)bollworms
ii) Insect pest resistant Bt Cotton was developed by a)
somaclonal variation b) micropropagation c) transgenic technology d) somatic hybridization
Ans. c) transgenic technology
iii) Which one is wrong in relation to transgenic Bt cotton plant
a) Crop yield loss due to attack by Bacillus thuringiensis is reduced.
b) Crop yield loss due to attack by lepidopteran insect pest is reduced.
c) Use of chemical pesticides in the cotton field is minimised.
d) Better quality cotton is produced.
Ans. a) Crop yield loss due to attack by Bacillus thuringiensis is reduced.
iv) In Bt cotton, a transgenic plant, Bt refers to
a) Biotechnology b) Bacillus thuringiensis c) Botanical d) Beta protein
Ans. b) Bacillus thuringiensisv) Bt cotton is not
a) a GM plant b) insect resistant c) is transgenic crop d) resistant to all pesticides
Ans. d) resistant to all pesticides
CASE BASED-2
The treatment of genetic disorder by manipulating genes is called gene therapy. However gene therapy has
not yet given any proven relief to patients. Any genetic disorder caused by a single defective allele can be
theoretically set right by replacing or supplementing the defective allele with a normal functional allele
through rDNA. The genetic
disease being investigated ranges from sickle cell anaemia to severe combined immuno deficiency (SCID).
The SCID patient has a defective gene for the enzyme adenosine deaminase (ADA). He/ she lacks functional
T lymphocytes and therefore cannot fight the infecting pathogens. Lymphocytes are extracted from the bone
marrow of patients and a normal functional copy of human gene coding for ADA is introduced into these
lymphocytes with the help of a retrovirus. These treated cells are re-introduced into the bone marrow of
patients. The lymphocytes produced by these cells contain functional ADA gene and reactivate the immune
system of the patients for life.
i) The full form of SCID is
a) systemic combined immuno disease b) syndrome containing immuno disease
c) severe combined immuno deficiency d) severe combined immuno disease
Ans. c) severe combined immuno deficiency
ii) The first clinical gene therapy was done for treatment of
a) AIDS b) Cancer c) Cystic fibrosis d) SCID
Ans. d) SCID
iii) ADA is an enzyme which is deficient in a person with genetic disorder SCID. Its full form is
a) Adenosine deoxyaminase b)Adenosine deaminase
c) Aspartate deaminase d) Arginine deaminase
Ans. b)Adenosine deaminase
iv) Which kind of therapy was given in 1990 to a four year old girl with ADA deficiency?
a) gene therapy b) chemotherapy
c) immunotherapy d) Radiation therapy
Ans. a) gene therapy
v) Patients with ADA deficiency- SCID patients lack
a) B lymphocytes b) Macrocytes c) Osteocytes d) T lymphocytes
Ans. d) T lymphocytes
CASE BASED -3
Its human nature, it seems, to resist change and fear the unknown. So, it is no surprise that genetic
engineering of food and feed crops raised many doubts in the minds of many consumers. Farmers and
agricultural scientists have been genetically engineering the foods we eat for centuries through breeding
programs. In addition to traditional crossbreeding, agricultural scientists have used radiation and chemicals to
induce gene mutations in edible crops in attempts to achieve desired characteristics. The fears of GMOs are
still theoretical, like the possibility that insertion of one or a few genes could have a negative impact on other
desirable genes naturally present in the crop. But many positive instances of GMO have also been reported.
Golden Rice, Iron rich Spinach, Bt-cotton are such examples. Ethical standards are required to evaluate the
morality of all such genetic modification activities. GEAC makes all the decisions regarding the validity of
GM research and the safety of introducing GMOs for public services.
i) Farmers and agriculture Scientists are using
a. Conventional breeding methods
b. Use of radiations and chemicals to induce gene mutations
c. Both (i) and (ii)
d. Recombinant DNA Techniques
Ans. C .Both (i) and (ii)
ii) Golden Rice is enriched in
a. Carbs b. B-carotene c. Fats d. Minerals
Ans. b. B-carotene
iii) GEAC stands for
a. Genetic Engineering Approval Committee
b. Genetic Environment Approval Committee
c. Genetic Engineering Action Committee
d. Genome Engineering Action Committee
Ans. a.Genetic Engineering Approval Committee
iv) Cultivation of Bt cotton has been much in the news. The prefix "Bt" means
a. Barium treated cotton seeds
b. Carrying an endotoxin gene from Bacillusthuringiensis.
c. Produced by biotechnology method
d. Bigger thread variety of cotton with tensile strength.
Ans. b.Carrying an endotoxin gene fromBacillusthuringiensis.
v) Assertion: GMOs have been useful in making crops tolerant to abiotic stresses, reduced reliance on
chemical pesticides and to enhance nutritional value of food.
Reason: Genetic modification has been used to create tailor-made plants.a. Both assertion and reason are true
and reason is the correct explanation of assertion.
b. Both assertion and reason are true but reason is not the correct explanation of assertion.
c. Assertion is true but reason is false.
d. Both assertion and reason are false
Ans. b.Both assertion and reason are true but reason is not the correct explanation of assertion.

CASE BASED -4 5 marks


In the given picture one is a diseased tobacco plant whereas one is a disease free plant
i) Name the process through the above plant is made disease free
a) RNAi b) Gene therapy c) Antigen antibody interaction d) none of the above
Ans. a) RNAi
ii) In this process, vector used is
a) Retrovirus b) Plasmid c) Bacteriophage d) Agrobacterium
Ans. d) Agrobacterium
iii) This method is a type of __________ mechanism found in all the _________
a) Defence, prokaryotes b) Cellular defence, Eukaryotes
c) Artificially induced, Eukaryotes d) Artificially induced, Prokaryotes
Ans. b) Cellular defence, Eukaryotesiv) The causative agent of the disease is
a) bacteria b) virus c) nematode d) none of the above
Ans. c) nematode
v) How is the pathogen specific gene silenced in this mechanism?
a) through ds DNA b) through ss DNA c) through ds RNA d) through ssRNA
Ans. c) through ds RNA
Chapter-13
Organisms and Populations
ORGANISMS AND POPULATION – MASTER CARD
GIST CONTENT
Population Single individuals of any species, live in a group in defined
geographical area, compete for similar resources, potentially
interbreed, constitute a population.
Example: Lotus plant in a pond, Cormorants in a wetland, rats in
abandoned dwelling, bacteria in a culture plate.
Population Birth rate, Death rate, Male- Female sex ratio contribute to
attributes population attributes
Age pyramid Population at any given time is composed of individuals of different
ages. The age distribution ( per cent individuals of a given age or a
age group) is plotted for the population, the resulting structure is
called an age pyramid.
Example: Expanding, Stable , Declining
Population The density of a population in a given period changes due to four
Growth basic processes,namely (i) Natality (ii) Mortality (iii) Immigration
(iv) Emigration
Population It is the number of individuals in a population per unit area,
density volume or size in relation to some unit of space. Population density
in a given habitat changes due to four basic process; If population
density is N at time t, then its density at time t+1 is ;
Nt+1 = Nt + [(B+I) – (D+E)]
N=population density, t= time, B= Birth rate(Natality), I =
immigration, D= Death rate (Mortality), E= Emigration
Examples: The tiger census in our national parks and tiger reserves
is often based on pug marks and fecal pellets.
Growth Populations have characteristic patterns of growth with time. They
models are known as Growth models.
(i) Exponential growth :Plentiful availability of resources results
in the exponential growth
(Nt = N0ert, dN/dT = rN)
(ii) Logistic growth : A population growing in a habitat with
limited resources shows logistic growth
(dN/dT = rN((K-N)/K) )
Population It refers to the interaction between different populations.
Interactions There are various modes of population interaction and are as
follows;
a) Mutualism-Both of the partners will get benefitted
Ex: Lichens
b) Competition-Both will get harmed
Ex: Monkeys
c) Predation-One partner will get benefitted other will get
harmed
Ex: Lion attacking Deer
d) Commensalism- One partner is benefitted and the other is
neutral. Ex: Orchid on mango tree
e) Amensalism-One partner is harmed (Detrimental) and the
other is neutral. Ex: Penicillium notatum inhibits the growth
of bacteria

KEY POINTS
Population Population is defined as the total number of individuals of
a species in a specific geographical area, which can interbreed
under natural conditions to produce fertile offspring and
function as a unit of biotic community.
Characteristics of 1. Population Density = Number of individuals of a species
population: per unit area or volume.
2. Natality : Per capita births in the population
3. Mortality : Per capita deaths in the population
4. Sex ratio : Ratio of male and female individuals
in the population
5. Age distribution : Percent individuals of a given
age or age group
Age Pyramid If the age group is plotted for the population, the resulting structure
is the age pyramid.
(i) Expanding :Maximum number of pre-reproductive
individuals. It is not ideal for a population
(ii) Stable : Number of pre-reproductive and reproductive
individuals is almost same.Post reproductive individuals
are fewer.It is ideal for a population.
(iii) Declining : Number of post reproductive individuals are
high. It is not ideal for a population.

Population The density of a population in a given period changes due to four


Growth basic processes, namely (i) Natality (ii) Mortality (iii)
Immigration (iv) Emigration
N(t + 1)= Nt + [(B + I) – (D + E)]

Population density will increase if the number of births plus the


number of immigrants (B + I) is more than the number of deaths
plus the number of emigrants (D + E), otherwise it will decrease.
Growth models Growth of population takes place according to availability of food,
habit condition and presence of other biotic and abiotic factors.
(a) Exponential growth (b) Logistic growth

a) Exponential Plentiful availability of resources results in the exponential


growth growth (Nt = N0ert ,dN / dt = (b – d) × N
Let (b – d) = r, then dN / dt = rN )
b) Logistic A population growing in a habitat with limited resources shows
growth logistic growth

Population Interspecific interactions arise from the interaction of two different


Interactions species. It could be detrimental(-),beneficial(+)or neutral(0).
Examples for commensalism
1. Sucker fish with shark
2. Cattle egret and grazing animals
3. Orchid growing on mango tree
4. Barnacles and whales

Examples for Mutualism :


1. Honey bees and the flowers
2. Flowers and insects
3. Nectar feeding birds and flowers
4. Lactobacillus and Humans
5. Cellulose digesting bacteria inside the caecum of
ruminants
6. Lichens
7. Rhizobium in the root nodules of leguminous plants
8. Sea-anaemone and clown fish
9. Mycorrhizal association with the roots of Glomus or
conifers.
10. Crocodile and crocodile bird
11. Sea anaemone and Hermit crab
12. Mediterranean orchid- sexual deceit for
pollination- appears as female bee

Examples for Amensalism


1. Penicillium notatum and Staphylococcus bacteria

Examples for competition


1. Flamingo & fish compete for zooplankton
2. Feeding efficiency of a species reduce due to other species
even if resources are plenty – Abingdon tortoise.

Examples for Predation


1. Lion and Tiger attacks on herbivores for food
2. Insectivorous plants

Examples for Parasitism


1. Plasmodium in humans
2. Leech and animals
3. Ascaris and humans
4. Louse,Ratflea,Tick,Itch mite in humans
Gause’s CompetitiveIt states that two closely related species competing for the same
Exclusion resources cannot co-exist indefinitely and the competitively
Principle inferior one will be eliminated eventually by the superior one.
Resource The phenomenon in which species facing competition might evolve
partitioning mechanisms that promote coexistence rather than exclusion.
POPULATIONS
A population is a group of organisms of one species that can interbreed and live in a particular geographical area.
POPULATION ATTRIBUTES

Natality Mortality Age Pyramid


(per capita birth) (per capita death) (age distribution of the population)

Age pyramid may be the following types:-


➢ Expanding
➢ Stable
➢ Declining
POPULATION GROWTH

• Population density is the average number of individuals in a population per unit of area
• Density of a population in a given habitat during a given period, fluctuates due to changes in four basic
processes, two of which (Natality and immigration) contribute to an increase in population density and two
(mortality and emigration) to a decrease.

If N is the
population density at time t, then its density at time t +1 is
Nt+1 = Nt + [(B + I) – (D + E)]
GROWTH MODELS
(i) Exponential growth
• It can occur if natural resources are unlimited and there is no
completion between species.
• Increase or decrease in N during a unit time period t (dN/dt) will be-3
dN/dt = (b – d) × N
Let (b–d) = r,
then dN/dt = rN
r = ‘intrinsic rate of natural increase

• ‘r’ is very useful in assessing the impacts of any biotic or abiotic factor on population growth.
• Another formula for exponential population growth is-
N= Noert
(ii) Logistic growth
• In nature the resources are limited and this leads to competition between individuals.
• Since resources for growth for most animal populations are finite and become limiting sooner or later, the
logistic growth model is considered a more realistic one.
• In nature, a given habitat has enough resources to
support a maximum possible number, beyond which
no further growth is possible; this is called as carrying
capacity (K).
• The logistic growth cure has- lag phase (initial phase),
phases of acceleration and deceleration and finally an
asymptote, when the population density reaches the carrying capacity.
• A plot of N in relation to time (t) results in a sigmoid curve. This type of population growth is called
• Verhulst-Pearl Logistic Growth and is described by the following equation:
POPULATION INTERACTIONS
PREDATION
• Predators feed on prey and by doing so they act as ‘conduits’ for energy transfer across trophic levels and keep
prey populations under control.
• In the absence of natural predators the prey species could achieve very high population densities and cause
ecosystem instability. Predators also help in maintaining species diversity in a community, by reducing the
intensity of competition among competing prey species.
The prickly pear cactus introduced into Australia (in 1920s) spread very rapidly into millions of hectares
of rangeland. It can be controlled only after introducing the natural predator moth of that cactus
In the rocky intertidal communities (American Pacific Coast) when predator starfish were removed from
an enclosed intertidal area, more than 10 species of invertebrates became extinct within a year, because
of interspecific competition.
Prey species have evolved various defenses to lessen the impact of predation-
Animals: Camouflage in frogs, highly distasteful Monarch butterfly
Plants: Thorns (Acacia, Cactus, highly poisonous cardiac glycosides (Calotropis), alkaloids (nicotine,
caffeine, quinine, strychnine, opium, etc.)
COMPETITION
Competition is best defined as a process in which the fitness of one species (measured in terms of its ‘r’ the intrinsic
rate of increase) is significantly lower in the presence of another species.
(i) Interspecific competition: competition between two different species
(ii) Intraspecific competition: competition between two same species
South American lakes, visiting flamingoes and resident fishes compete for their common food, the
zooplankton in the lake.
The Abingdon tortoise in Galapagos Islands became extinct within a decade after goats were introduced
on the island, apparently due to the greater browsing efficiency of the goats.

• Connell’s elegant field experiments showed that on the rocky sea coasts of Scotland, the larger and
competitively superior barnacle Balanus dominates the intertidal area, and excludes the smaller barnacle
Chathamalus from that zone.
• Gause’s ‘Competitive Exclusion Principle’ – it states that two closely related species competing for the same
resources cannot co-exist indefinitely and the competitively inferior one will be eliminated eventually.
• Resource partitioning- if two species compete for the same resource, they could avoid competition by
choosing, for instance, different times for feeding or different foraging patterns. MacArthur showed that five
closely related species of warblers living on the same tree were able to avoid competition and co-exist due to
behavioral differences in their foraging activities.
PARASITISM
It is the interaction where one species (parasite) depends on the other species (host) for food and shelter, the host is
harmed.
Ectoparasite: feeds on the external surface of the host. Examples – head lice on humans, ticks on dogs, Cuscuta
Endoparasite: Take shelter within the body of the host organism. Examples – Liver fluke, Plasmodium

Parasites evolved special adaptations Loss of unnecessary sense organs, presence of adhesive organs or suckers
to cling on to the host, loss of digestive system and high reproductive capacity.
Brood parasitism- In this one bird (e.g. cuckoo) lays its eggs in the nest of another bird (e.g. crow) and lets that
other bird to incubate them.

COMMENSALISM
In this one species benefit and the other is neither harmed nor benefited.
Examples: An orchid growing as an epiphyte on a mango branch, Barnacles growing on the back of a whale, the
cattle egret and grazing cattle, Sea anemone, and clown fish

MUTUALISM
It is an interaction in which both the interacting species are benefited
Examples: Lichen (fungi and algae), Mycorrhiza (fungi and roots of higher plants), Pollination of plants by insects,
Mediterranean orchid- sexual deceit for pollination- appears as a female bee

AMENSALISM
In this one species is harmed and the other species is neither harmed nor benefited.

Example. Fungus Penicillium secrete Penicillin which kills the bacteria but there is no benefit to the fungus
IMPORTANT QUESTIONS?
Very Short Answer Type Questions
1- Penicillium placed with streptococcus will show which type of population interaction?
a. Commensalism
b. Amensalism
c. Parasitism
d. None of these
Ans: b
2- An orchid growing as an epiphyte on a mango branch shows which type of population interaction?
a. Commensalism
b. Amensalism
c. Parasitism
d. None of these
Ans: a
3- If ‘+’ sign is positive interaction and ‘-‘ sign is detrimental, then which type of population interaction will be
shown by ‘+’ ‘-‘

a. Commensalism
b. Amensalism
c. Parasitism
d. None of these
Ans: c
4- Sea anemone, and clown fish show-
a. Commensalism
b. Amensalism
c. Parasitism
d. None of these
Ans: a
5- In this one bird (e.g. cuckoo) lays its eggs in the nest of another bird (e.g. crow) and lets that other bird to
incubate them
a. Commensalism
b. Amensalism
c. Brood Parasitism
d. None of these
Ans: c
6- Define competitive release in reference to the population interaction.
Ans: Competitive release is the mechanism in which the species expands its distribution range when the
competitor species is removed.
7- What is emigration? How does it affect population density?
Ans: It is the outward movement of some individuals of the species from a local population during the time
period under consideration.
It decreases the population density.
8- What are two parameters by which population can be measured?
Ans: Biomass and Number
9- Name the orchid plant which undergoes sexual deceit phenomenon.
Ans: Mediterranean orchid Ophrys
10- The given age of pyramid represents –
Ans: stable population
11- Why do certain exotic species (like Parthenium in India) become too much invasive in certain geographical
areas?
Ans: because those areas do not have natural predators of weed
12- In which specific condition ‘J-shaped’ population growth curve may be observed?
Ans: when there are unlimited natural resources available, and no competition takes place.
Short answer Type questions

1- 8 individuals in a laboratory population of 80 fruit flies died in a week. Calculate the death rate.
Ans: Death rate= no of individual die/ total number
= 8/80 = 10
2- Fill the ‘a’ and ‘b’ with appropriate population attributes-

Ans: A= Natality or Immigration B= Mortality or Emigration

3- Give an example for each of these:


a) Chemical defense agent b) Predator animal c) Migratory animal d) Camouflaged animal
Ans: a- alkaloids b- Lion c- Bird d- Frog
4- Mention the attributes which a population has but not an individual organism.
Ans: Natality, Mortality, Sex ratio, age groups.
5- List any two examples of defense mechanisms in plants against herbivorous predators.
Ans. i) Thorn in Acacia.
ii) Cardiac glycosides in Calotropis
Long Answer Type Questions
1- (i) Discuss the role of predators in an ecosystem.
(ii) What is brood parasitism? Give an example
Answer: (i) Predators feed on prey and by doing so they act as ‘conduits’ for energy transfer across trophic
levels and keep prey populations under control. In the absence of natural predators, the prey species could
achieve very high population densities and cause ecosystem instability. Predators also help in maintaining
species diversity in a community, by reducing the intensity of competition among competing prey species.
(iii) Phenomenon in which one (parasitic) bird species lays its eggs in the nest of another bird species.
Cuckoo lays her egg in crow’s nest.
2- (i)How does age distribution help in the study of the population?
(ii) How does an age pyramid, for the human population at a given point of time helps the policymakers in
planning the future?
(iii) Draw the different types of age pyramid.
Ans: (i)The relative abundance of the organisms of various age groups in the population is called the age
distribution of the population. Age may be grouped into pre-reproductive, post-reproductive and reproductive
With regard to age distribution, there are three kinds of populations:
Expanding, stable and declining.
(ii) The shape of the age pyramid reflects the growth status of the population. Thus age pyramid for the human
population at a given time helps policymakers in planning for the future.
(iii) Fig 13.4, page 227, NCERT
3- (i)Compare exponential and logistic growth curves by giving a diagram only.
(ii) Write a mathematical equation to determine population density in logistic growth.
(iii) Mention the significance of the ‘r’ value.
Ans: (i) Fig 13.6 page 230 NCERT
(ii) Nt = N 0 ert
Nt = Population density after time t
N0 = Population density at time zero
r = intrinsic rate of natural increase
e = the base of natural logarithms (2.71828)
(iii) ‘r’ is very useful in assessing the impacts of any biotic or abiotic factor on population growth.
4- What is ‘sexual deceit’? Explain by giving a suitable example.
Ans: one petal of orchid flower resembles as female bee in size, colour and marking.
The male bee is attracted to what it perceives as a female, ‘pseudocopulates’ with the flower, During this bee
is dusted with pollen from the flower.
When this same bee ‘pseudocopulates’ with another flower, it transfers pollen to it and thus, pollinates the
flower.
5- What are the different types of interaction in a habitat? Explain with the help of chart.
Ans: fig 13.1, page 232, NCERT

ASSERTION AND REASON QUESTIONS


In the following questions a statement of assertion followed by a statement of reason is given. Choose
the correct answer out of the following choices.
(a) Assertion and reason both are correct statements and reason is correct explanation for assertion.
(b) Assertion and reason both are correct statements but reason is not correct explanation for
assertion.
(c) Assertion is correct statement but reason is wrong statement.
(d) Assertion is wrong statement but reason is correct statement.
1. Assertion : Species are groups of potentially interbreeding natural populations which are isolated
from other such groups.
Reason : Distinctive morphological features are displayed due to reproductive isolation.
2. Assertion : Leaf butterfly and stick insect show mimicry to dodge their enemies.
Reason : Mimicry is a method to acquire body colour blending with the surroundings.
3. Assertion : Small sized animals are rarely found in polar regions.
Reason : Small sized animal have larger surface area relative to their volume and have to spend energy
to generate body heat.
4. Assertion : A stable population is depicted by bell-shaped age pyramid.
Reason : The proportion of individuals in reproductive age group is higher than those in pre
reproductive age group.
5. Assertion : Plant-animal interactions do not generally involve co-evolution of the mutualist
organisms.
Reason : Evolution of plants and animals go side by side.
6. Assertion : Predators are organisms which feed on other individuals.
Reason : Prey species have evolved various defences to lessen the impact of predation.
7. Assertion : Population pyramid (graphically) depicts the rate at which population will grow in future.
Reason : A triangular population pyramid depicts population size is stable.
8. Assertion : Epiphytes growing on branches of the tree exhibit commensalism.
Reason : In commensalism on organism benefits from the association while the other has no effect.
9. Assertion : Coral reefs are found in regions of West Bengal and Andhra Pradesh.
Reason : Coral reef require low fresh water inflow, high salinity and optimal temperature to propagate.
10. Assertion : Verhulst-Pearl Logistic growth curve is sigmoid in nature.
Reason : A population growing in habitat with limited resources shows an initial lag phase, followed by
acceleration and deceleration and finally an asymptote.
Answers 1. (b) 2. (a) 3. (a) 4. (c) 5. (d) 6. (b) 7. (c) 8. (a) 9. (d) 10. (b

MULTIPLE CHOICE BASED QUESTIONS


1. Which one of the following is not a parasitic adaptation?
(a) Development of adhesive organs (b) Loss of digestive organs
Sl.No Population Sl.No Example
Interaction

1 Predation A Cuscuta and Hedge plants


2 Commensalism B Balanus and Chthamalus
3 Parasitism C Cactus and moth
4 Competition D Orchid and Mango
(c) Loss of reproductive capacity (d) Loss of unnecessary sense organs
2. In a growing population of a country,
(a) pre-reproductive individuals are more than the reproductive individuals
(b) reproductive individuals are less than the post-reproductive individuals
(c) reproductive and pre-reproductive individuals are equal in number
(d) pre-reproductive individuals are less than the reproductive individuals.
3. Asymptote in a logistic growth curve is obtained when
(a) K = N (b) K > N (c) K < N (d) the value of ‘r’ approaches zero
4. Select the correct statement.
(a) In a population, number of births is different from birth rate
(b) A sigmoid growth curve is depiction of exponential growth
(c) In a logistic growth curve the asymptote is beyond the carrying capacity
(d) ‘r’ is equal to the difference between number of births and number of deaths in a population.
5. Which of the following is correct?
(a) Population change = (Birth + immigration) – (death + emigration)
(b) Population change = (Birth + immigration) + (death + emigration)
(c) Population change = (Birth + emigration) + (death – immigration)
(d)Population change = (Birth – immigration)– (death + emigration)
6. Which is correctly labelled with respect to the given diagram?
(a) B : Logistic curve (b) C : Carrying capacity
(c) C : Exponential curve (d) A : Carrying capacity
7. Between which among the following, the relationship is not an example of commensalism?
(a) Orchid and the tree on which it grows
(b) Cattle Egret and grazing cattle
(c) Sea Anemone and Clown fish
(d) Female wasp and fig species
8. If ‘+’ sign is assigned to beneficial interaction,‘–’ sign to detrimental and ‘O’ sign to neutral
interaction, then the population interaction represented by ‘+’ ‘–’ refers to
(a) mutualism (b) amensalism (c) commensalism (d) parasitism.
Answer: The option D is correct
9. Match the following.
(a) 1-C, 2-D, 3-A, 4-B (b) 1-D, 2-C, 3-B, 4-A
(c) 1-A, 2-C, 3-B, 4-D (d) 1-C, 2-D, 3-B, 4-A
10. All the following interactions are mutualism, except
(a) Plant and animal relation for pollination
(b) Association of algae and fungi in lichens
(c) Association of cattle egret and grazing cattle
(d) Association of fungi and roots of higher plants in mycorrhiza.
11. Gause’s principle of competitive exclusion states that
(a) no two species can occupy the same niche indefinitely for the same limiting resources
(b) larger organisms exclude smaller ones through competition
(c) more abundant species will exclude the less abundant species through competition
(d) competition for the same resources exclude species having different food preferences
12. Which one of the following population interactions is widely used in medical science for the
production of antibiotics?
(a) Commensalism (b) Mutualism (c) Parasitism (d) Amensalism
13. Which of the following statements is false regarding predators?
(a) Predators keep prey populations under control.
(b) Predators help in maintaining species diversity in a community.
(c) If a predator is not efficient, then the prey population would become extinct.
(d) Herbivores (predators) have a greater advantage since the plants cannot run away to avoid
predation.
14. Which one of the following causes population explosion?
(a) Decrease in infant mortality rate and increase in death rate
(b) Decrease in death rate, maternal mortality rate and infant mortality rate
(c) Decrease in infant mortality rate and decrease in the number of people in reproductive age
(d) Decrease in death rate and increase in maternal mortality rate
CASE BASED MCQ’s:
15 Case I: Read the following passage and answer the questions from 46 to 50 given below. Growth of
a population with time shows specific and predictable patterns. Two types of growth patterns of
population are exponential and logistic growth. When resources in the habitat are unlimited each
species has the ability to realise fully its innate potential to grow in number. Then the population
grows in exponential fashion. When the resources are limited growth curve shows an initial slow rate
and then it accelerates and finally slows giving the growth curve which is sigmoid.
(i) Which of the following statement is incorrect?
(a) Exponential growth occurs in organism such as lemmings.
(b) Logistic growth is more realistic.
(c) Exponential growth has two phases lag and log.
(d) In logistic growth, population passes well beyond the carrying capacity of ecosystem.
(ii) Which of the following equations correctly represents the exponential population growth curve?
(a) dN/dt = rN (b) dN/dt=rN (K- N/K)
(c) Nt = N0ert (d) Both (a) and (c)
(iii) Which of the following equations correctly represents Verhulst-Pearl logistic growth?
(a) dN/dt = rN(K-N/K) (b) dN/dt = rN/K
(c) dN/dt = N(K-N)/K (d) dN/dt = r(K-N)/K
(iv) The population growth is generally described by the following equation:
dN/dt = rN(K-N/K) What does ‘r’ represent in the given equation?
(a) Population density at time ‘t’ (b) Intrinsic rate of natural increase
(c) Carrying capacity (d) The base of natural logarithm
(v) Study the population growth curves (A and B) in the given graph and select the incorrect option
(a) Curve ‘A’ shows exponential growth, represented by equation dN/dt=rN.
(b)Curve ‘B’ shows logistic growth, represented by equation dN/dt = r(K-N)/K
(c) Exponential growth curve is considered as more realistic than the logistic growth curve.
(d) Curve ‘A’ can also be represented by equation Nt = N0ert
16. Case II :The organism has various alterations for coping with extremes environment. Some able to
respond through certain physiological adjustments while others do so behaviourally. These responses
are their adaptations. Many adaptations have evolved over a long evolutionary time and are
genetically fixed. Many desert plants have a thick cuticle on their leaf surfaces and have their stomata
arranged in deep pits to minimise water loss through transpiration. In the polar seas, aquatic
mammals like seals have a thick layer of fat (blubber) below their skin that acts as an insulator and
reduces the loss of body heat. Some organisms possess adaptations that are physiological which
allows them to respond quickly to a stressful situation.
(i) Adaptation maybe
a) behavioural
b) morphological
c) physiological
d) all of these
(ii)Opuntia has spine-like leaves which help in
a) reducing the rate of transpiration
b) increasing the rate of transpiration
c)increasing the rate of photosynthesis
d) reducing the rate of photosynthesis
(iii)Mammals from colder climates generally have shorter ears and limbs to minimise heat loss. This is
called
a) Allen’s rule
b) Berger’s rule
c) Borge’s rule
d) Powell’s rule
(iv) In the absence of an external source of water, the kangaroo rat in North American deserts is
capable of meeting all its water requirements through
a) its internal fat oxidation
b) through concentrating its urine
c) none of these
d) both (a) and (b)
ASSERTION AND REASON QUESTIONS:
For the below questions, two statements are given-one labelled Assertion and the other labelled
Reason. Select the correct answer to these questions from the codes (a), (b), (c) and (d) as given
below.
(a) Both assertion and reason are true and reason is the correct explanation of assertion.
(b) Both assertion and reason are true but reason is not the correct explanation of assertion.
(c) Assertion is true but reason is false.
(d) Assertion is false but reason is true.
17. Assertion : Sigmoid growth curve occurs when resources are unlimited.
Reason: Exponential growth curve occurs when resources are abundant.
Answer: The option D is correct
18. Assertion : The prickly pear cactus introduced into Australia in early 1920s caused havoc by
spreading rapidly into millions of hectares of land range.
Reason: When certain exotic species are introduced into a geographical area, they become
invasive and start spreading fast because the invaded land does not have the natural predators.
19. Assertion :Natality increases both population density and population size.
Reason :Natality increases the number of individuals in an area by births.
20. Assertion: Fig species and wasp have a tight one to one relationship .
Reason: Angiosperms and insects are coevolved to perform a plant
pollinator interaction.
21. Assertion: Predation and parasitism are considered to be negative
interactions.
Reason : Parasites and predators limit the population.
22. Assertion : Pathogenic bacteria do not multiply well in the soil
Reason : Antibiotic producing fungi and bacteria are common in the soil.
23.Assertion :Leguminous plants grow well in N2-deficient soil.
Reason :They need little Nitrogen
24.Assertion :Biological control of pests benefits environment
Reason :It does not cause pollution
Answer Key
1 2 3 4a 5a 6d 7d 8 9 10 a
c a a d a
11 12 13 14 15 15 15 15 15 16 (i)
a d d b ( ( ( (iv ( d
i i i ) v
) i i )
d ) i b
d ) c

a
16 16 16 17 18 19 20 21 22 23 c
( ( (iv)d d a a a b a
i i
i i
) i
a )
a
24
a
Chapter-14
Ecosystem
Ecosystem
An ecosystem is the functional unit of nature where living organisms (biotic component) interact among themselves
and also with the surrounding physical environment( Abiotic component).
Ecosystem- Structure and Functions Ecosystem

Terrestrial ecosystem forest, grassland, desert


Aquatic ecosystem ponds, lakes, river estuary
Components of ecosystem

Biotic Non-living component Light, Water, Soil, atmospheric gases, etc


Abiotic Living component Producer, consumer, decomposers
The components of the ecosystem that are seen as a functional unit are-
(i)Productivity (ii) Decomposition (iii) Energy flow (iv) Nutrient cycling

Productivity The rate of biomass production is GPP- It is the rate of production of organic matter
called productivity. during photosynthesis.
Unit -gm–2 yr –1 or (kcal m–2 ) yr –1 GPP minus respiration losses (R), is the net
Primary production is the primary productivity (NPP).
amount of biomass or organic NPP = GPP - R
matter produced per unit area NPP- is the available biomass for the consumption
over a time period by plants to heterotrophs
during photosynthesis. Secondary productivity is defined as the rate of
Unit - (gm–2) or energy (kcal m–2) formation of new organic matter by consumers.
Decomposition
• Breakdown of complex organic matter into inorganic substances like carbon dioxide, water and nutrients
• Detritus- Dead plant remains and dead remains of animals.
Steps of Decomposition- Fragmentation, leaching, catabolism, humification and mineralization
Fragmentation of Detritivores (e.g., earthworms) break down detritus into smaller particles. This process
Detritus is called fragmentation
Leaching Water-soluble inorganic nutrients go down into the soil horizon and get precipitated as
unavailable salts.
Catabolism Bacterial and fungal enzymes degrade detritus into simpler inorganic substances
Humification Accumulation of a dark coloured amorphous substance called humus
Mineralization Degradation of humus microbes and release of inorganic nutrients in the soil
Factors affecting rate of Decomposition

Chemical The decomposition rate will be slow when detritus is rich in lignin and chitin and the rate
composition increases when detritus is rich in nitrogen and water-soluble substances like sugars.
Climatic Warm and moist environment favour decomposition and low temperatures and anaerobiosis
conditions inhibit decomposition.
Energy Flow

• All living organisms are dependent on their food on


producers, directly or indirectly.
• There is a unidirectional flow of energy from the sun
to producers and then to consumers.
• Photosynthetically active radiation (PAR) is
responsible for the synthesis of food by plants.
• Transfer of energy follows the 10 percent law that is
only 10 percent of the energy is transferred to each trophic level from the lower trophic level.
Food chain

• Consumers obtain their food from autotrophs (plants).


• Food chain is the flow of energy from one trophic level to another trophic level.
• Trophic level: Based on the source of their nutrition or food, organisms occupy a specific place in the food
chain that is known as the trophic level. E.g. producer, herbivore, primary carnivore, secondary carnivore

• Food chains are of two types- Grazing food chain (GFC) and detritus food chain (DFC)

GFC Energy flows from producers to consumers.


DFC Begins with dead organic matter. It is made up of saprotrophs/ decomposers
(heterotrophic organisms like fungi and bacteria).

Food web
The natural interconnection of the food chain forms the food web.
Significance of food web:
(1) Food webs permit alternative foods.
(2) They ensure a better chance of survival of an organism, in case any of
its food sources happens to be scarce
(3) More complex food web means a more stable ecosystem

Ecological Pyramids
• Pyramid is the graphical representation of an ecological
parameter (number, biomass, energy) sequence-wise in various
trophic levels of a food chain with producers at the base and herbivores in the middle and carnivores at the
top tiers.
• It can be upright, inverted, or spindle-shaped.

Three common ecological pyramids are

Pyramids of • Represent the number of individuals per unit area at various trophic levels with
number a producer at the base.
• It is generally upright.
• A pyramid of numbers in the case of a big tree is generally inverted because the
number of insects feeding on that tree generally exceeds in number.
Pyramids of • Represent the biomass in various trophic levels.
biomass • The pyramid of mass is upright except in the aquatic food chain involving short-
lived plankton.
• A pyramid of biomass in the sea is generally inverted.
Pyramids of • Give s graphic representation of the amount of energy trapped by different
energy trophic levels per unit area.
• It is always upright, and can never be inverted, because when energy flows from
a particular trophic level to the next trophic level, some energy is always lost as
heat at each step e.g in feeding, digestion, assimilation and respiration
IMPORTANT QUESTIONS
Very Short Answer Type Questions
1- Which one is an example of a manmade ecosystem.
a- Aquarium
b- Crop field
c- Aquaculture pond
d- All of these
Ans: d
2- During ecological succession
a. Speciation in the new area
b. The gradual and predictable change in species composition occurs in a given area
c. Reduction of a pioneer community
d. Bare land with no vegetation
Ans: b
3- Energy transfer takes place from one trophic level to another –
a- 10 % b- 20 % c- 50 % d- depends on the trophic level
Ans: A
4- Which of the following is called as a detrivore?
a. An animal feeding on an animal
b. A plant feeding on an insect
c. An animal feeding on insect
d. An animal feeding on decaying organic matter
Ans: d
5- Root hairs are absent in-
a. Xerophytes
b. Hydrophytes
c. Lithophytes
d. Mesophytes
Ans: b
6- Why decomposition occurs at a faster rate in the tropics?
Ans: high humidity and temperature favour the rapid rate of decomposition
7- How do decomposers like fungi obtain their food?
Ans: Decomposers release their enzymes to decompose dead and decaying remains of pants and animals
absorb the simple inorganic substances.
8- Name the trophic level occupied by secondary consumers & tertiary consumers.
Ans. Third trophic level by secondary consumer and fourth trophic level by tertiary consumer
9- What is the shape of the pyramid of biomass in the sea? Why?
Ans: Inverted, because the biomass of fishes is too much as compared to very smaller phytoplankton.

Sort answer types questions

1- Describe stratification by citing three examples.


Ans: Vertical distribution of different species occupying different levels is called stratification. For example,
trees occupy the top vertical strata of a forest, shrubs are the second and grasses occupy the bottom layers.
2- What are the four basic functional components of an ecosystem?
Ans: (i) Productivity; (ii) Decomposition; (iii) Energy flow (iv) Nutrient cycling.
3- Give two examples of autotrophic components in the food chain of the aquatic ecosystem.
Ans: phytoplankton, some algae
4- Which metabolic process causes a reduction in gross primary productivity? Explain
Ans: A considerable amount of GPP is utilized by plants in respiration.
Gross primary productivity minus respiration losses (R), is the net primary productivity (NPP). GPP – R = NPP
5- Give an account of energy flow in an ecosystem.
Ans: The flow of energy in an ecosystem is unidirectional.
The Sun is the ultimate source of energy. It is used in the photosynthesis process to make food in autotrophs.
This energy is transferred from autotrophs to the next trophic level. Only 10% of energy is transferred to next
trophic level, the rest amount is lost as heat.

Very Long Answer Type Questions

1- Describe steps in the process of decomposition of detritus in DFC (detritus food chain).
Ans: Fragmentation of Detritus - Detritivores (e.g., earthworms) break down detritus into smaller particles.
Leaching- Water-soluble inorganic nutrients go down into the soil horizon and get precipitated as unavailable
salts.
Catabolism- Bacterial and fungal enzymes degrade detritus into simpler inorganic substances
Humification- Accumulation of a dark-coloured amorphous substance called humus
Mineralization- Degradation of humus microbes and release of inorganic nutrients in the soil.
2- Provide a diagrammatic representation of the decomposition cycle in a terrestrial ecosystem.
Ans: Fig. 14.1 page 244, NCERT
3- What is meant by the ecological pyramid? Distinguish between upright & inverted pyramids. Explain with
the help of a diagram.
Ans: The graphic representation of the trophic structure of a food chain is known as the ecological pyramid.
In the upright pyramid, the number or biomass of producers is more in comparison to consumers in an
ecosystem.
In an inverted pyramid, the number/biomass of producers is less as compared to consumers.
Fig. 14.4 (a, b, c) page 248 NCERT
4- (i)How does the rate of decomposition affected by abiotic factors in an ecosystem?
(ii) Construct a pyramid of biomass starting with phytoplankton.
Ans: (i) Decomposition rate will be slow when detritus is rich in lignin and chitin and rate increases when
detritus is rich in nitrogen and water-soluble substances like sugars.
Warm and moist environment favour decomposition and low temperature and anaerobiosis inhibit
decomposition.
(ii)

5- Describe the components of an ecosystem.


Ans: i) Abiotic components – Inorganic substances (P, N, K, C, H etc.) b) Organic substances (Protein,
carbohydrates, lipids), Climatic (water, air, soil, temperature etc.)
ii) Abiotic components-
Producers (make their own food like plants)
Consumers (Depend on producers for food. Consumers are of the following types-
i) Primary consumers - herbivores e.g. deer, cow
ii) Secondary consumers – feed on primary consumers like cats, fox
iii) Tertiary consumers- feed on secondary consumers like Lion
Decomposers: - Decompose dead and decaying objects like fungi, bacteria
Assertion and Reason-
Directions: In the following questions, a statement of assertion is followed by a
statement of reason. Mark the correct choice as:
(a) If both Assertion and Reason are true and Reason is the correct explanation of Assertion.
(b) If both Assertion and Reason are true but Reason is not the correct explanation of Assertion.
(c) If Assertion is true but Reason is false.
(d) If both Assertion and Reason are false.
1. Assertion : Net primary productivity is gross primary productivity minus respiration.
Reason : Secondary productivity is produced by heterotrophs.
Ans 1-a-If both Assertion and Reason are true but Reason is not the correct explanation of
Assertion
2. Assertion :A network of food chains existing together in an ecosystem is known as food
web.
Reason: An animal like kite can not be a part of a food web.
Ans-c- If Assertion is true but Reason is false.
3. Assertion : Pyramid of energy may be upright or inverted.
Reason: Only 20%ofenergygoestonexttrophic level.
Ans3-d- If both Assertion and Reason are false.
4. Assertion: The pyramid of energy is always upright.
Reason: Maximum number of autotrophs is present in the pyramid of energy.
Ans4-c- If Assertion is true but Reason is false.
5. Assertion: Ecosystem can be defined as the community of organism together with the
environment in which they live.
Reason :It includes both biotic and abiotic factors.
Ans5-a- If both Assertion and Reason are true and Reason is the correct explanation of
Assertion.
6. Assertion : Food web consists of several food chains.
Reason: Food web decreases the stability of an ecosystem.
Ans-6- c- If Assertion is true but Reason is false.
7. Assertion :The conversion of productivity at nexttrophiclevelis10%.
Reason : Energy is lost in the respiration process.
Ans7- a- If both Assertion and Reason are true and Reason is the correct explanation of
Assertion.
8. Assertion :A big fish eats small fish which eats water fleas and water fleas in turn eat
phytoplankton. In this chain, water fleas are primary consumers.
Reason : Secondary consumers will be phytoplankton.
Ans8- c- If Assertion is true but Reason is false.
9. Assertion: Despite occupying about70% ofthe earth’s surface. Oceans are a low
productivity ecosystem.
Reason: In aquatic ecosystems, productivity is limited by light that decreases with
increasing water depth.
Ans9-
a- If both Assertion and Reason are true and Reason is the correct explanation of
Assertion.
10. Assertion: During the process of photosynthesis amount of organic matter synthesized
by producers per unit time and per unit area is referred to as net primary productivity.
Reason: Due to abundance of sunlight. Primary productivity is usually high and sustained
throughout the year in temperate areas.
Ans10-d- If both Assertion and Reason are false.
11. Assertion- The annual net primary productivity of the whole biosphere is
approximately 170 billiontons (dry weight) of organic matter.
Reason- Majority of this is contributed by the oceans as they occupy
larger area of earth.
Ans-c
12. Assertion-In a terrestrial ecosystem, detritus food chain is a major conduit for
energy flow.
Reason: Solar energy is the direct source of energy supply in the detritus food chain.
Ans-c
13. Assertion- Ecosystem is the structural and functional unit of biosphere consisting of
abiotic and bioticcomponents which interact with each other and maintain a
balance in nature.
Reason: In an ecosystem, energy and matter are continuously exchanged between living
and non-livingcomponents.
Ans- a
14. Assertion-The rate of decomposition is controlled by chemical composition of detritus.
Reason-Decomposition is largely an oxygen
requiring process.
Ans-b
15. Assertion-Primary productivity varies in different types of ecosystems.
Reason: Primary productivity depends on a variety of environmental factors, availability
of nutrients andphotosynthetic capacity of plants.
Ans- a
Chapter-15
Biodiversity and its Conservation
BIODIVERSITY
• Biodiversity term is popularized by Edward Wilson.
• Biodiversity is the sum total of flora and fauna in a geographical area.

TYPES OF BIODIVERSITY

How Many Species are there on Earth and How Many in India?
Some facts

PATTERNS OF BIODIVERSITY
a) Latitudinal gradients
• Species diversity decreases as we move away from the equator towards the poles.
Country location Bird species
Colombia near the equator 1,400
New York 41° N 105
Greenland at 71° N 56
India Mostly tropical 1200
• Biodiversity in the amazon forest (South America) as the greatest biodiversity on earth.
Plant 40000 Mammal 427
Fish 3000 Reptile 378
Bird 1300 Invertebrate 125000
Tropics harbor more species than temperate or polar areas.
• Tropical latitudes have remained relatively undisturbed for millions of years.
• Tropical environments are less seasonal and more constant and predictable.
• More solar energy available in the tropics results in higher productivity.
Species-Area relationships
• Alexander von Humboldt observed that within a region species richness increased with increasing
explored area, but only up to a limit.
• On a logarithmic scale, the relationship is a straight line described by the equation
• log S = log C + Z log A (Where, S= species, A= Area, Z= slope of the line, C =Y- intercept)
Rivet popper hypothesis
• Proposed by Paul Ehrlich.
He proposed that every rivet of an airplane is like species of an ecosystem. Loss of rivets (key species that perform
major ecosystem functions) may cause loss of ecosystem as well as biodiversity.
LOSS OF BIODIVERSITY
• IUCN (International Union for Conservation of Nature) documented the Red List which reported the extinction
of 784 species (including 338 vertebrates, 359 invertebrates, and 87 plants) in the last 500 years.
• Example of recent extinction: the dodo (Mauritius), quagga (Africa), thylacine (Australia), Steller’s Sea Cow
(Russia) and three subspecies (Bali, Javan, Caspian) of tiger.

CONSEQUENCES OF BIODIVERSITY LOSS-


• Decline in plant production
• Lowered resistance to environmental perturbations, drought, and flood.
• Increased variability in ecosystem processes such as productivity, water use, and pest and disease cycles.

CAUSES OF BIODIVERSITY LOSSES

‘The Evil Quartet’ (The four


major causes)

BIODIVERSITY CONSERVATION
Reason for conservation

Narrowly utilitarian Biodiversity provides direct economic benefits from nature like food, firewood, fibers,
medicinal plants, industrial products, etc
Broadly Utilitarian Biodiversity plays a major role in ecosystem services like the production of Oxygen
during photosynthesis, pollination without a natural pollinator, and pleasure from
nature are priceless.
Ethical for conserving biodiversity relates to what we own to millions of plants, animals and
microbes species with whom we share this planet.
HOW DO WE CONSERVE BIODIVERSITY?
In situ (on-site) conservation Ex situ (off-site) conservation
 conserving species in their natural  Animals and plants are taken out of their
habitats and environment natural habitat and placed in special
 Ex- biosphere reserves, national parks, settings to protect them.
wildlife sanctuaries, sacred groves  Ex- Zoological parks, Botanical Gardens,
wildlife safari parks and Cryopreservation
HOT SPOTS
• Biodiversity hotspots are the regions with very high levels of species richness and high degree of endemism.
• Three hotspots – Western Ghats and Sri Lanka, Indo-Burma and Himalaya
SACRED GROVES
• Socially, culturally, or religiously important place, protected by local people.
• Examples- Khasi and Jaintia Hills (Meghalaya), Aravalli Hills (Rajasthan)
CONVENTIONS ON BIODIVERSITY
• The Earth Summit- Rio de Janeiro in 1992, World Summit - Johannesburg, South Africa in 2002
RAMSAR SITE

• Ramsar Sites are wetland sites designed of international importance under the Ramsar Convention.
• These wetlands are protected under strict guidelines of the Ramsar Convention on Wetlands.
➢ The main objectives of the Ramsar Convention are:
• To ensure the wise use of all their wetlands. The wise use of wetlands means; maintaining the ecological
character of a wetland.
• designate appropriate wetlands for the list of Wetlands of International Importance (the “Ramsar List”) and
guarantee their effective management.
• To cooperate worldwide on transboundary wetlands, shared wetland systems and shared species.
➢ India currently has 54 sites designated as Ramsar sites.

IMPORTANT QUESTIONS
Very Short Answer Questions
1- Identify the ex-situ conservation by which gametes of threatened species can be preserved.
a- Zoological Park
b- Wildlife sanctuary
c- Sacred grooves
d- Cryopreservation
ANS: D
2- Ramsar sites designed of international importance are-
a- Oceans
b- Aquatic bodies
c- Wetlands
d- All of these
Ans: c
3- Aravalli Hills are well known for
a- Religious site
b- ASI sites
c- Mangrove
d- Sacred grooves
Ans: d
4- Examples of onsite biodiversity conservations are
a- biosphere reserves
b- national parks
c- wildlife sanctuaries
d- all of these
Ans: d
5- western ghats have more amphibians than eastern ghat represents-
a- species diversity
b- genetic diversity
c- ecological diversity
d- global diversity
Ans: a
6- List some important conventions held at the international level for biodiversity conservation.
Ans: The Earth Summit- Rio de Janeiro in 1992, World Summit - Johannesburg, South Africa in 2002
7- Make your statement in favor of India as a major mega biodiversity country.
Ans: India has only 2.4 % of the world’s land area but its biodiversity is 8.1 %. Therefore India is in one of the
12 mega-diversity countries of the world.
8- Give an example of any two alien species that become a threat to indigenous plant species.
Ans: Parthenium, water hyacinth (Eichhornia)
9- How exotic species differ from endemic species.
Ans: Exotic species are derived into a geographical area from any other geographical area. Endemic species
are native species restricted to a particular geographical area.

Short Answer Type Questions

1- (i) Hot spots of biodiversity show high degree of endemism. Explain the statement.
(ii) Give three examples of hot spots.
Ans: (i) species confined to a region and not found anywhere else. Example: Polar beer
(ii) Western Ghats and Sri Lanka, Indo-Burma and Himalaya
2- It is very difficult to study the pattern of biodiversity of prokaryotes. Give reason.
Ans: The problem is that conventional taxonomic methods are not suitable for identifying microbial species
and many species are simply not culturable under laboratory conditions.
3- What are evil quarters responsible for loss of biodiversity?
Ans- Habitat loss, over exploitation, Alien species invasion, co-extinction
4- Give four examples of recent extinction due to biodiversity loss and over exploitation.
Ans: Example of recent extinction: the dodo, quagga, thylacine, Steller’s Sea Cow
5- Explain co-extinction and its consequence with a suitable example.
Ans: Co-extinction refers to the disappearance of species with extinction of another species of plant or animal.
E.g. Plant-pollinator mutualism, In absence of pollinators the plant cannot survive.
6- What could have triggered mass extinctions of species in the past?
Ans: Glaciation, melting of snow, the eruption of large volcanoes, earthquakes, movement of continents, large
meteorites falling on the earth, drought, etc.
7- Describe sixth extinction. How it is different from previous episodes? What is major concern of ecologist
about sixth extinction?
Ans: In past five episode of mass extinction takes place. In present sixth Extinction’ is in progress.
The current species extinction rates are estimated to be 100 to 1,000 times faster than in the pre-human times.
Ecologists warn that if the present trends continue, nearly half of all the species on earth might be wiped out
within the next 100 years
8- “Amazonian rain forest in south America has the greatest bio-diversity on earth”. Justify the statement.
Ans: Amazonian rain forest in south America has the greatest biodiversity on earth; it harbors about40000
species of plants, 1,25,000 species of insects, 3000 species of fishes, 427 of amphibians, 378 of reptiles, 1300
of birds & 427 of mammals.
9- Provide examples of the followings –
(i)Over exploitation (ii) Alien species invasion (iii) Co extinction
Ans: (i) extinction of Steller’s Sea cow, passenger pigeon
(ii) Alien species invasion- Nile perch introduced into Lake Victoria (East Africa) causes extinction more
than 200 species of cichlid fish in the lake.
(iii) Co extinction- plant-pollinator mutualism where extinction of one invariably leads to the extinction of
the other.
10- What is the special about tropics that might account for their greater biodiversity?
Ans: (i) Tropical latitudes have remained relatively undisturbed for millions of years.
(ii) Tropical environments are less seasonal and more constant and predictable.
(iii) More solar energy available in the tropics that results in higher productivity.

Very Long Answer Type Questions


1- Identify the type of biodiversity with the help following statements
(i) Variation in number Amphibians of eastern ghat and western ghat
(ii) Alpine meadow has greater ecosystem diversity than Scandinavian country
Class XII Biology Concept Maps & Important Diagrams
(iii) Rauwlofia vomitaria in different Himalaya range
(iv) Thousands of species of Rice
(v) Desert, coral reefs and mangroves in Indian sub-continent
Ans: (i) species diversity (ii) and (v) ecological diversity (iii) and (iv) (genetic diversity
2- (i) Represent global biodiversity of plants, invertebrates and vertebrates by giving suitable pie charts.
(ii) As per IUCN data how many species were become extinct in recent past?
Ans: (i) fig 15.1 , [page 260 NCERT
(ii) 784 species (including 338 vertebrates, 359 invertebrates and 87 plants)
3- Who proposed Species-Area relationships? Explain by giving graphical representation.
Ans: Species area relationship was given by Alexander Von Humboldt
According to this within a region species richness increased with increasing explored area, but only up to a
limit.
On a logarithmic scale, the relationship is a straight line described by the equation log S
= log C + Z log A
Where, S= species, A= Area, Z= slope of the line, C =Y- intercept. Fig.
15.2, page 262 NCERT
4- (i) Why is there a need to conserve biodiversity?
(ii) Name and explain any two ways that are responsible for the loss of biodiversity.
Ans: (i) The narrowly utilitarian- Biodiversity provides direct economic benefits from nature like food,
firewood, fibres, medicinal plants, industrial products etc.
The Broadly Utilitarian- Biodiversity plays a major role in ecosystem services like productions of Oxygen during
photosynthesis, pollination without natural pollinator, pleasure from nature are priceless.
Ethical- for conserving biodiversity relates to what we own to millions of plants, animals and microbes species with
whom we share this planet.
(ii) Over-exploitation- Many species extinctions in the last 500 years were due to overexploitation by humans. For
example- Steller’s Sea cow, passenger pigeon.
Alien species invasions- Some of alien species turn invasive and cause decline or extinction of indigenous
species. E.g. The Nile perch introduced into Lake Victoria in east Africa led eventually to the extinction of more
than 200 species of cichlid fish in the lake. Invasive weeds species like carrot grass (parthenium), Lantana and
water hyacinth causing threats to indigenous species.
5- What are the different approaches for biodiversity conservation in India?
Ans: There are two major approaches for conservation of biodiversity: -
i) In-situ conservation - it is protecting of the endangered species in the natural habitat.
Biosphere Reserves, National Park, wildlife Sanctuaries, Sacred Grooves are example of In-situ conservation India
has 14 biosphere reserves, 90 national parks and 448 wildlife sanctuaries.

ii) Ex- situ Conservation- It is the process of protecting of the species rearing them under human care. It includes
Botanical Garden, zoological park. Cryopreservation is the modern method of ex-situ preservation where
gametes can be stored at very low temperature.
CASE BASED TYPE QUESTIONS
15. Indian Board of Wild life (IBWL) was constituted in 1952.The main function of IBWL is the promotion of public
involvement in wildlife and its preservation in harmony with natural and human environment.India is rich in
biodiversity. I also promotes the collaboration with international organizations on wildlife research, management
and training. It advise the government on matters relating to wildlife conservation and preservation,giving or
reserving clearances to projects in and around national parks and other protected areas.Since 1955,a wild life
week (first week of October) is being observed to educate the people about the importance and need of
conservation of wild life.As per Wild life Protection Act (1972),Hunting and trade of the products of useful animal
wild life should be regulated.No person shall enter a sanctuary with any weapon except with the previous
permission in writing of the Chief Wild Life Warden or the authorised officer.National wild life Action plan was
introduced in 1983-84 for rehabilitation of endangered species by captive breeding.Methods of Wildlife
Conservation,Habitat management, Establishment of the protected area, Rehabilitation of Endangered species,
Captive breeding programme, Mass education, Promulgation of laws:
(i) Wild life conservation week is celebrated in
K.VS..MUMBAI REGION
Class XII Biology Concept Maps & Important Diagrams
(a) August (b) October (c)September (d) December
(ii) Hunting and trade of the animal products banned under
(a) Wild life Protection act ,1952
(b) Wild life Protection act ,1964
(c) Wild life Protection act ,1972
(d) Wild life Protection act ,1984
(iii) Rehabilitation of Endangered species can be done in
(a) In-situ conservation
(b) Ex-situ conservation
(c) Both a & b
(d) None of these
(iv) Identify Biodiversity hot spot in India
(a) Western Ghats (b) Assam (c) Meghalaya (d) Sikkim
Assertion-Reason Questions In the following questions a statement of assertion followed by a statement of reason
is given. Choose the correct answer out of the following choices.
(a) Assertion and reason both are correct statements and reason is correct explanation for assertion.
(b) Assertion and reason both are correct statements but reason is not correct explanation for assertion.
(c) Assertion is correct statement but reason is wrong statement.
(d) Assertion is wrong statement but reason is correct statement.
1. Assertion : A network of food chains existing together in an ecosystem is known as a food web.
Reason : An animal like kite cannot be part of a food web.
2. Assertion : In open water zone upto the depth to which light can penetrate is called photic zone.
Reason : The photic zone contains autotrophs.
3. Assertion : In a food chain the members of the successive trophic levels are fewer.
Reason : Number of organisms at any trophic level is independent of the availability of organisms which serve as
food at the lower level.
4. Assertion : Vertical distribution of different species occupying different levels is called stratification.
Reason : Trees occupy top vertical strata, shrub the second, herbs and grasses occupy the bottom layers.
5. Assertion : Primary succession takes very long time.
Reason : Soil is absent at the time of beginning of primary succession.
6. Assertion : Pyramid of energy is always upright.
Reason : When energy flows from a particular trophic level to the next trophic level, some energy is always lost at
heat at each step.
7. Assertion : Ecological succession occurs when older communities of plants and animals are replaced by newer
communities.
Reason : The natural process of replacement of one vegetation community in a given habitat by the other
vegetation community.
8. Assertion : The decomposers feed on detritus, or decaying organic matter, derived from all levels.
Reason : At each level of energy flow in the food web, energy is lost to respiration.
9. Assertion : In a terrestrial ecosystem, detritus food chain is the major conduit for energy flow. Reason : Solar
energy is the direct source for energy supply in a detritus food chain.
10. Assertion : Net primary productivity is gross primary productivity minus respiration.
Reason : Secondary productivity is produced by heterotrophs.
Answers 1. (c) 2. (b) 3. (d) 4. (b) 5. (a) 6. (a) 7. (a) 8. (b) 9. (c) 10. (b

K.VS..MUMBAI REGION
Class XII Biology Concept Maps & Important Diagrams

Chapter 2. Sexual Reproduction in Flowering Plants

Fig: LS of a flower Fig: (a) A typical stamen; (b) three–dimensional cut section of an anther

K.VS..MUMBAI REGION
Class XII Biology Concept Maps & Important Diagrams

Fig: (a) Transverse section of a young anther; (b) Enlarged view of one microsporangium showing wall layers

Fig: A mature dehisced anther

Fig: Pollen Grain Tetrad Fig: stages of a microspore maturing into a pollen grain

K.VS..MUMBAI REGION
Class XII Biology Concept Maps & Important Diagrams

Fig: A. Hibiscus pistil. B. Multicarpellary, syncarpous pistil of Papaver Fig: Anatropous Ovule
C. A multicarpellary, apocarpous gynoecium of Michelia

Fig: Parts of the ovule showing a large megaspore mother cell, a dyad and a tetrad of megaspores

Fig: 2, 4, and 8-nucleate stages of embryo sac and a mature embryo sac & A diagrammatic representation ofthe
mature embryo sac.

K.VS..MUMBAI REGION
Class XII Biology Concept Maps & Important Diagrams

Fig: Cleistogamous flowers Fig: A wind-pollinated plant showing compact


inflorecence and well exposed stamens

Fig: Pollination by water in Vallisneria Fig: L.S. of pistil showing path of pollen tube growth

Fig: An egg apparatus showing entry


of pollen tube into a synergid Fig: Discharge of male gametes into a synergid and
the movements of the sperms, one into the egg
and the other into the central cell.

K.VS..MUMBAI REGION
Class XII Biology Concept Maps & Important Diagrams

Fig: Fertilised embryo sac showing Fig: Stages in embryo development in a dicot.
zygote and Primary Endosperm Nucleus (PEN).

Fig: A typical dicot embryo Fig: L.S. of an embryo of grass (monocot)

Fig: Structure of some seeds Fig: False fruits of apple and strawberry

K.VS..MUMBAI REGION
Class XII Biology Concept Maps & Important Diagrams

Chapter 3. Human Reproduction

K.VS..MUMBAI REGION
Class XII Biology Concept Maps & Important Diagrams

Fig: Male Pelvis showing Reproductive System Fig: Male Reproductive System

Fig: Seminiferous Tubules Fig: Female Pelvis showing Reproductive System

Fig: Female Reproductive System Fig: Mammary Gland


Sequence of milk conduction in mammary glands: Mammary alveoli → mammary tubules → mammary
duct → mammary ampulla → lactiferous duct.

Fig: Spermatogenesis Fig: Sperm


K.VS..MUMBAI REGION
Class XII Biology Concept Maps & Important Diagrams

Fig: Oogenesis Fig: Ovum surrounded by few sperms


3 membranes on ovum: Plasma membrane, Zona pellucida, Corona radiata

Fig: Sectional view of a seminiferous tubule(enlarged) Fig: Structure of ovary

MENSTRUAL CYCLE (REPRODUCTIVE CYCLE)


Phases Days Main events
1. Menstrual phase 1-5th day Menstrual flow (bleeding).

o Primary follicles → Graafian follicles.


2. Follicular (Proliferative) phase 5-13th day
o Proliferation of ruptured uterine endometrium.
LH surge → rupture of Graafian follicle →
3. Ovulatory phase 14th day
ovulation.
Corpus luteum forms → progesterone →
4. Secretory (Luteal) phase 15-28th day
endometrium maximum vascular, thick and
soft

Fig: Menstrual Cycle

K.VS..MUMBAI REGION
Class XII Biology Concept Maps & Important Diagrams

FERTILIZATION AND IMPLANTATION

Zygote → cleavage → morula (8-16 blastomeres) → blastocyst → embryo

Fig: Transport of ovum, Fertilisation, passage of growing embryo through fallopian tube & Implantation

Fig: Blastocyst
A. Inner cell mass: Becomes embryo. B. Trophoblast: Gives nourishment to inner cell mass. Also, it is
attached to endometrium.

Fig: The Human foetus within the uterus

PARTURITION: Signals from foetus & placenta → mild uterine contractions (fetal ejection
reflex) → oxytocin from pituitary → stronger uterine muscle contractions → further secretion of
oxytocin
→ Parturition (giving birth).

K.VS..MUMBAI REGION
Class XII Biology Concept Maps & Important Diagrams

Chapter 4. Reproductive Health

WHO: World Health Organization, RCH: Reproductive & Child Health Care Programme
MMR: Maternal Mortality Rate, IMR: Infant Mortality Rate.

Fig: Female Condom Fig: Male Condom

Fig: Cu-T Fig: Implants

Fig: Vasectomy (Male) Fig: Tubectomy

K.VS..MUMBAI REGION
Class XII Biology Concept Maps & Important Diagrams

Chapter 5. Principles of Inheritance and Variation

Contrasting Traits
7 Characters Dominant Recessive
1. Stem height Tall Dwarf
2. Flower colour Violet White
3. Flower position Axial Terminal
4. Pod shape Inflated Constricted
5. Pod colour Green Yellow
6. Seed shape Round Wrinkled
7. Seed colour Yellow Green

Fig: Steps in making a cross in Pea


K.VS..MUMBAI REGION
Class XII Biology Concept Maps & Important Diagrams

Fig: Monohybrid Cross

Fig: A Punnett square used to understand a typical monohybrid cross conducted by Mendel between true-
breeding tall plants and true-breeding dwarf plants.

K.VS..MUMBAI REGION
Class XII Biology Concept Maps & Important Diagrams

Fig: Test Cross

Fig: Dihybrid Cross

Fig: Incomplete Dominance in Snapdragon

K.VS..MUMBAI REGION
Class XII Biology Concept Maps & Important Diagrams

Table: Co-dominance (Multiple Allelism)


Blood types
Alleles from parent 1 Alleles from parent 2 Genotype of offspring
(phenotype)
IA IA IA IA A
IA IB IA IB AB
IA i IAi A
IB IA IA IB AB
IB IB IB IB B
IB i IBi B
i i ii O

Fig.: Male and Female Drosophila melanogaster

Fig: Linkage: Results of two dihybrid crosses conducted by Morgan. Cross A shows crossing between gene y and w;
Cross B shows crossing between genes w and m. Here dominant wild type alleles are represented with (+) sign in
superscript Note: The strength of linkage between y and w is higher than w and m.

Fig: Sex Determination by Chromosomal differences

K.VS..MUMBAI REGION
Class XII Biology Concept Maps & Important Diagrams

Fig: Symbols used in the human Pedigree Analysis Fig: (A) Autosomal dominant trait (E.g., Myotonic
dystrophy) & (B) Autosomal recessive trait (E.g. Sickle-cell anaemia)

Fig: Micrograph of the red blood cells and the amino acid composition of the relevant portion of b-chain of
haemoglobin: (a) From a normal individual; (b) From an individual with sickle-cell anaemia

K.VS..MUMBAI REGION
Class XII Biology Concept Maps & Important Diagrams

Chapter 6. Molecular Basis of Inheritance

Fig.: A Polynucleotide chain

K.VS..MUMBAI REGION
Class XII Biology Concept Maps & Important Diagrams

Fig.: Double Stranded Polynucleotide Chain & DNA Double Helix

Fig: Nucleosome Fig.: Hershey-Chase Experiment (Blender Experiment):


Nucleosomes condense → chromatin → chromosome.

Fig.: Messelson & Stahl’s Experiment:

K.VS..MUMBAI REGION
Class XII Biology Concept Maps & Important Diagrams

Fig.: Replication fork

Fig.: Structure of a transcription unit

Fig.: Process of Transcription in Bacteria Fig.: Process of Transcription in Eukaryotes

Fig.: tRNA-the adapter molecule Fig.: Translation

K.VS..MUMBAI REGION
Class XII Biology Concept Maps & Important Diagrams

Fig: The Lac Operon

Fig: Human Genome Project

Fig.: DNA Fingerprinting

K.VS..MUMBAI REGION
Class XII Biology Concept Maps & Important Diagrams

Chapter 8. Human Health & Diseases

Fig: Stages in the life cycle of Plasmodium Fig: Antibody Molecule


K.VS..MUMBAI REGION
Class XII Biology Concept Maps & Important Diagrams

Fig: Inflammation due to Elephantiasis and ringworm affected area of the skin

Fig: Lymph Nodes Fig: Replication of retrovirus

Fig: Structure of Morphine & Opium poppy Fig: Cannabinoid Molecule & Leaves of Cannabis sativa

Fig: Flowering Branch of Datura

K.VS..MUMBAI REGION
Class XII Biology Concept Maps & Important Diagrams

Chapter 10. Microbes in Human Welfare

Microbes Group Uses


1. Acetobacter aceti Bacterium Acetic acid
2. Aspergillus niger Fungus Citric acid
3. Azospirillum Bacterium Nitrogen fixation, biofertilizer
4. Azotobacter Bacterium Nitrogen fixation, biofertilizer
5. Bacillus thuringiensis (Bt) Bacterium Biocontrol of butterfly caterpillar
Biocontrol of insects and other
6. Baculoviruses
Virus arthropods. Used in Integrated
(mainly nucleopolyhedrovirus)
Pest
Management (IPM).
7. Clostridium butyilicum Bacterium Butyric acid
8. Cyanobacteria (blue green algae) Bacteria Nitrogen fixation, biofertilizer
9. Lactobacillus (LAB) Bacterium Lactic acid, milk to curd
10. Methanobacterium (methanogens) Bacterium Biogas (CH4) production i.e., source of
energy
11. Monascus purpureus Fungus (a yeast) Statins (blood cholesterol
loweringagents)
Fungi (E.g., Glomus) +
12. Mycorrhiza Biofertilizer
plant
Penicillin (First antibiotic discovered by
13. Penicillium notatum Fungus (mould)
Alexander Fleming).
14. Penicillium roqueforti Fungus Roquefort cheese
15. Propionibacterium shermanii Bacterium In Swiss cheese formation
16. Rhizobium Bacterium Nitrogen fixation, biofertilizer
17. Saccharomyces cerevisie (Baker’s Production of beverages, bread
Fungus
yeast or Brewer’s yeast) byfermenting dough, ethanol etc.
18. Streptococcus Bacterium Streptokinase (a clot buster)
19. Trichoderma polysporum Fungus Cyclosporine A (An immunosuppressive
agent)
20. Trichoderma sp Fungus Biocontrol

K.VS..MUMBAI REGION
Class XII Biology Concept Maps & Important Diagrams

Fig: Steps of Sewage treatment

Fig: Secondary Treatment Fig: Aerial view of Sewage Plant

Fig: Biogas Plant

K.VS..MUMBAI REGION
Class XII Biology Concept Maps & Important Diagrams

Chapter 11. Biotechnology-Principles & Processes

Fig: Steps in formation of recombinant DNA by action of restriction endonuclease enzyme - EcoRI

K.VS..MUMBAI REGION
Class XII Biology Concept Maps & Important Diagrams

Fig: Representation of recombinant DNA technology

2. Cloning Vector

Fig: A typical agarose gel electrophoresis Fig: E. coli Cloning Vector pBR322
showing migration of undigested (lane 1) and
digested set of DNA fragments (lane 2 to 4)

Fig: DNA that separates out can Fig: PCR


be removed by spooling

K.VS..MUMBAI REGION
Class XII Biology Concept Maps & Important Diagrams

Fig: Simple stirred-tank bioreactor Fig: Sparged stirred-tank bioreactor


through which sterile air bubbles are sparged

K.VS..MUMBAI REGION
Class XII Biology Concept Maps & Important Diagrams

Chapter 12. Biotechnology & its Applications

Fig: Maturation of pro-insulin into insulin Fig: Eli Lilly Method for production of Humulin.

K.VS..MUMBAI REGION
Class XII Biology Concept Maps & Important Diagrams

Fig: Cotton Boll: (a) destroyed by bollworms;


(b) a fully mature cotton boll Fig: Host plant-generated dsRNA triggers protection against
nematode infestation: (a) Roots of a typical control plants; (b)
transgenic plant roots 5 days after deliberate infection of nematode
butprotected through novel mechanism.

Fig: Gene Therapy

K.VS..MUMBAI REGION
Class XII Biology Concept Maps & Important Diagrams

Chapter 13. Organisms & Populations

K.VS..MUMBAI REGION
Class XII Biology Concept Maps & Important Diagrams

Fig: representation of organismic response Fig: age pyramids for human population

Fig: 4 Processes affecting Population Density Fig: Population growth curve


a. when responses are not limiting (exponential plot)
b. when responses are limiting (logistic plot), K is
carrying capacity.
Table: Population Interactions

Fig: Insect Pollination and Pseudo Copulation

K.VS..MUMBAI REGION
Class XII Biology Concept Maps & Important Diagrams

Chapter 15. Biodiversity & Conservation

Fig: Global biodiversity: proportionate number of species of major taxa of plants, invertebrates and vertebrates

K.VS..MUMBAI REGION
Class XII Biology Concept Maps & Important Diagrams

Fig: Showing species area relationship. Note that on log scale the relationship becomes linear

K.VS..MUMBAI REGION
Class XII Biology Concept Maps & Important Diagrams

KVS SAMPLE QUESTION PAPER


Sub-Biology(044)
Time: 3 hours Maximum Marks: 70
General Instructions:
I. All questions are compulsory
II. The Question Paper has fivesections and 33 questions. All questions are compulsory.
III. Section –A has 16 questions of 1 marks each. Section B has 5 questions of two marks each. Section-C has 7
questions of 3 marks each. Section-D has 2 case based questions of 4marks each and Section E has 3 questions
of 5 marks each.
IV. There is no overall choice. However internal choice have been provided in some questions. A student has to
attempt only one of the alternatives in such questions.
V. Wherever necessary, neat and properly labelled diagram should be drawn.
Section A

Q.No Questions Marks


1 Name the cells which fuses during triple fusion: 1
a.Two Polar nuclei and male gamete
b. One polar nucleus, male gamete and female gamete
c.Two Synergid and male gamete.
d. Two antipodals and male gamete.
2 Chemically gene is a: 1
a. DNA
b. DNA + Histone Protein
c. DNA + Non Histone Protein
d. DNA + Histone Protein + Non Histone Protein
3 Name the female gametophyte of angiosperm: 1
a. Egg
b. Integuments
c. Embryo sac
d. Ovary
4 Fill in the missing blank „a‟ and „b‟ exhibiting the route of sperm transport. 1
Seminiferous tubule→---- ------→Vasa Efferentia →Epididymis ----- ---→Urethra a) i-Vas
i ii

deferens ii- Rete testis


b) i- Rete testis ii- Vas deferens
c) i- Lobules ii- urinary bladder
d) i- Vas deferens ii- Lobules
5 Biotic components of an ecosystem include: 1
a) Producers, consumers and decomposers
b) Producers and consumers
c) Producers only
d) Consumers only

K.VS..MUMBAI REGION
Class XII Biology Concept Maps & Important Diagrams

6 Which hormone is a milk ejecting hormone? 1


a) Prolactin
b) Oxytocin
c) Estrogen
d) Progesterone
7 Self- Pollination is fully ensured if : 1
a) The flower is bisexual
b) The style is longer than filament
c) The flower is cleistogamous.
d) The flower is unisexual
8 Identify the type of inheritance from following pedigree: 1

a) X linked recessive inheritance


b) Y linked Inheritance
c) Autosomal Dominant inheritance
d) None of the above
9 Night blindness can be prevented by use of : 1
a) Golden Rice
b) Flavr Savr tomato
c) Bt Brinjal
d) None of the above
10 Which of the following Pyramid can never be inverted? 1
a) Pyramid of Biomass
b) Pyramid of Number
c) Pyramid of Energy
d) None of the above
11 Antibody abundant in Colostrum is:
a) IgA
b) IgG
c) IgD
d) IgM
12 The given Karyotype below shows the following syndrome:

K.VS..MUMBAI REGION
Class XII Biology Concept Maps & Important Diagrams

a) Turner‟s Syndrome
b) Down‟s Syndrome
c) Klienfelter‟s Syndrome
d) Criminal Syndrome

Questi on No 13 to 16 consist of two statements- Assertion (R) and Reason (R).Answer these que stions
selecti ng the appropriate option given below:

a. Both assertion and reason are true and reason is correct explanation of assertion

b. Both assertion and reason are true, but reason is not the correct explanation of assertion.

c. Assertion is true but reason is false.

d. Assertion is False but Reason is true

13 Assertion: DNA is known as blue print of living organism. 1


Reason :Type of protein or enzyme formed in body depends on type of DNA carried by
Organism.

14 Assertion:Agrobacterium is used to deliver foreign gene in animals. Reason:Agrobacterium has 1


Ti plasmid.

15 Assertion: In birds, sex is determined by male bird. 1


Reason: Female bird produces two type of genetically different egg containing either Z or W
sex chromosome.

16 Assertion:Value of Gross Primary Productivity is greater Net Primary Productivity. `1


Reason:Net Primary productivity minus Respiration loss is Gross Primary Productivity.
Section B

17 State any two features of artificial plasmid pBR322 which make it fit to use as a cloning vector. 2

18 Name the special enzyme which is obtained from bacteria Thermus aquaticus and used 2

in PCR. Also write its significance.

K.VS..MUMBAI REGION
Class XII Biology Concept Maps & Important Diagrams

19 While playing in ground one of your friend is bitten by snake. He is taken to a nearby hospital 2
where he is administered with an injection to counter snake venom.
Name the type of immunisation in the above case and explain the benefit of this type of
immunisation in above case.
20 How does a small amount of curd added to fresh milk convert it into curd? Mention any one 2
advantage of curd over milk.
21 What do you mean by flocs? State their importance in biological treatment of water. 2

Section C
22 3
How do Darwin finches illustrate adaptive radiation?
Or

Comment on the similarity of forearm of man and forelimb of birds stating its significance in
evolution.
23 What do you mean by IVF?Write the steps of IVF. 3
24 What are transgenic animals? Enlist any four areas where they can be used. 3
25 Explain briefly the following: 3
a) Bioreactor
b) Bt cotton
c) Cloning of Gene
26 3
a) Explain Hardy Weinberg Principle.
b) The frequency of two alleles in a gene pool is 0.19 (A) and 0.81(a). Assume that the
population is in Hardy-Weinberg equilibrium.
Calculate the percentage of heterozygous individuals in the population.

27 Explain the process of microsporogenesis with the help of diagram. 3


28 Explain the following cause regarding loss of biodiversity. 3
a) Habitat fragmentation
b) Introduction of Alien Species
c) Co extinction
Section D
Q.no 29 and 30 are case based questions. Each question has subparts with internal choice in one subpart.
29 Read the following regarding population interaction and answer the questions that follow: 4
An ecological community consist of all the populations of all the different species that live
together in a particular area. Interactions between different species in a community are called
interspecific interactions. Different type of interspecific interaction have different effects on
two participant which may be positive(+),negative(-) or neutral(0) The main type of
interspecific interactions include

K.VS..MUMBAI REGION
Class XII Biology Concept Maps & Important Diagrams

competition,Predation,mutualism,commensalism andparasitism.
In most case, many species share a habitat and the interaction between them play a major role
in regulating population growth and abundance
a) Name the interaction where one species is benefitted while other is not affected. b)
Give an example of mutualism.
c) Explain the kind of interaction built by cuscuta growing on hedge plant.
d) The Abingdon tortoise in Galapagos Islands became extinct within a decade after
goats were introduced on the island. Explain the cause for this effect. Or
Explain Mac Arthur “Resource Partitioning” theory of competition.

30 Read the following regarding genetics of ABO blood groupand answer any four questions from 4
16(i) to 16(v) given below:

ABO blood groups are controlled by single gene I.The plasma membrane of the red blood cells
has sugar polymer (antigen) that protrude from its surface and the kind of sugar is controlled
by the gene. The gene I has three alleles IA,IB and i.The alleles IA and IB produce a different form
of the sugar while allele idoes not produce any sugar. When individual carries allele IA and IB
then both are expressed and RBC of the individual has both the antigen A and B.The blood
group of such individual is AB. IA and IB are dominant over i.
a) What do you mean by allele?
b) What is the difference between incomplete dominance and complete dominance?
c) A child has A blood group. What can be the possible genotype for that Blood group?
d) If a cattle with black coat is crossed with white coat then F1 hybrid possess both black
and white colour in patches known as roan coat. What will be the phenotypic ratio on
inbreeding the F1.Show it in a checker board?
Or
In a paternity case, a single mother claimed that a certain man was the father of her
baby. The man denied it, claiming that her current boyfriend was the father. The court
ordered a blood test (much cheaper than DNA testing) to see if he could be ruled out
as the father. The mother was Type O and the baby was Type O. The man was Type
AB. Is it possible that he was the father? Why or why not?

Section E
31 Explain oogenesis and show its various stages by flowchart or diagram. Or 5
Explain the process of spermatogenesis and show its various stages by flowchart or diagram.

32 Explain the life cycle of malarial parasite stating its various stages in human and 5
female anopheles.
Or
What is the cause of cancer at cellular level? Name the technique in which a piece of tissue is
cut from the organ suspected for cancer. What are the different strategies used for the
treatment of cancer.

K.VS..MUMBAI REGION
Class XII Biology Concept Maps & Important Diagrams

33 With reference to lac operon answer the following questions. 5

a) What do you mean by operon?


b) What is the role of repressor protein in lac operon?
c) What is the role of promoter?
d) Name the inducer in lac operon and also state its role.
e) Which enzyme is responsible for breaking lactose into glucose and galactose .Also
state which gene of lac operon synthesize this enzyme?

Or

Explain the process of transcription in eukaryotes. Write any two difference in transcription of
Eukaryote and Prokaryotes.

K.VS..MUMBAI REGION
Class XII Biology Concept Maps & Important Diagrams

BIBLIOGRAPHY
1- CBSE (https://www.cbse.gov.in/)
2- NCERT ( https://ncert.nic.in/)
3- Google Chrome images
4- KVS STUDY MATERIALS 2023-24 Prepared by ZIET Chandigarh.
5- Study Materials prepared by PGTs of Various KVS Regions

You might also like